Download as pdf or txt
Download as pdf or txt
You are on page 1of 201

Solutions Manual for College Algebra:

Concepts Through Functions 3rd


Sullivan III 0321925726
Visit to download the full and correct content document: https://testbankbell.com/dow
nload/solutions-manual-for-college-algebra-concepts-through-functions-3rd-sullivan-iii
-0321925726/
More products digital (pdf, epub, mobi) instant
download maybe you interests ...

Precalculus Concepts Through Functions A Right Triangle


Approach to Trigonometry 3rd Edition Sullivan Solutions
Manual

http://testbankbell.com/product/precalculus-concepts-through-
functions-a-right-triangle-approach-to-trigonometry-3rd-edition-
sullivan-solutions-manual/

College Algebra 10th Edition Sullivan Solutions Manual

http://testbankbell.com/product/college-algebra-10th-edition-
sullivan-solutions-manual/

Solution Manual for Precalculus: Concepts Through


Functions, A Unit Circle Approach to Trigonometry 4th
Edition by Sullivan

http://testbankbell.com/product/solution-manual-for-precalculus-
concepts-through-functions-a-unit-circle-approach-to-
trigonometry-4th-edition-by-sullivan/

Solution Manual for College Algebra, 11th Edition,


Michael Sullivan

http://testbankbell.com/product/solution-manual-for-college-
algebra-11th-edition-michael-sullivan/
Test Bank for Precalculus: Concepts Through Functions,
A Unit Circle Approach to Trigonometry 4th Edition by
Sullivan

http://testbankbell.com/product/test-bank-for-precalculus-
concepts-through-functions-a-unit-circle-approach-to-
trigonometry-4th-edition-by-sullivan/

Fundamentals of Statistics Sullivan III 4th Edition


Solutions Manual

http://testbankbell.com/product/fundamentals-of-statistics-
sullivan-iii-4th-edition-solutions-manual/

Solutions Manual to accompany Algebra for College


Students 3rd edition 9780136129080

http://testbankbell.com/product/solutions-manual-to-accompany-
algebra-for-college-students-3rd-edition-9780136129080/

College Algebra 7th Edition Blitzer Solutions Manual

http://testbankbell.com/product/college-algebra-7th-edition-
blitzer-solutions-manual/

College Algebra Lial 11th Edition Solutions Manual

http://testbankbell.com/product/college-algebra-lial-11th-
edition-solutions-manual/
Solutions Manual for College Algebra: Concepts
Through Functions 3rd Sullivan III 0321925726

To download the complete and accurate content document, go to:


https://testbankbell.com/download/solutions-manual-for-college-algebra-concepts-thro
ugh-functions-3rd-sullivan-iii-0321925726/
Solutions Manual for College Algebra: Concepts Through Functions 3rd Sullivan III 0321925726

Chapter 3
Polynomial and Rational Functions
Section 3.1 12. y = 3 x 4

1. ( −2, 0 ) , ( 2, 0 ) , and ( 0,9 ) 13. ∞ ; −∞


x-intercepts: let y = 0 and solve for x
14. The value of f becomes a larger and larger
9 x 2 + 4 ( 0 ) = 36 negative number as x becomes a larger and
9 x 2 = 36 larger positive number.
x2 = 4 15. f ( x) = 4 x + x3 is a polynomial function of
x = ±2
degree 3. Standard form: f ( x) = x3 + 4 x .
y-intercepts: let x = 0 and solve for y Leading term: x3 . Constant term: 0
2
9 ( 0 ) + 4 y = 36
4 y = 36 16. f ( x) = 5 x 2 + 4 x 4 is a polynomial function of
y=9 degree 4. Standard form: f ( x) = 4 x 4 + 5 x 2 .

2. Yes; it has the form Leading term: 4 x 4 . Constant term: 0


an x n + an −1 x n −1 + ... + a1 x + a0 where each ai is a
1 − x2 1 1 2
real number and n is a positive integer.; degree 3 17. g ( x) = = − x is a polynomial
2 2 2
3. down; 4 function of degree 2.Standard form:
1 1 1
4. Local maximum 6.48 at x = −0.67; g ( x) = − x 2 + . Leading term: − x 2 .
2 2 2
Local minimum −3 at x = 2 .
10 10 1
Constant term:
2

1
−3 3 −3 3 18. h( x) = 3 − x is a polynomial function of
2
1
degree 1.Standard form: h( x ) = − x + 3 .
−10 −10 2
5. False; The zeros are the x-intercepts. 1
Leading term: − x . Constant term: 3
2
6. The point ( 5, 0 ) is on the graph and is the x-
1
intercept of g . 19. f ( x) = 1 −= 1 − x −1 is not a polynomial
x
7. smooth; continuous function because it contains a negative
exponent.
8. touches
20. f ( x) = x( x − 1) = x 2 − x is a polynomial
9. ( −1,1) , ( 0, 0 ) , and (1,1)
function of degree 2. Standard form:
10. a. r is a real zero of a polynomial function f . f ( x) = x 2 − x . Leading term: x 2 . Constant
b. r is an x-intercept of the graph of f . term: 0
c. x − r is a factor of f .
21. g ( x) = x3/ 2 − x 2 + 2 is not a polynomial
function because it contains a fractional
11. turning points
exponent.

263
Copyright © 2015 Pearson Education, Inc.

Visit TestBankBell.com to get complete for all chapters


Chapter 3: Polynomial and Rational Functions

22. h( x) = x ( )
x − 1 = x − x1/ 2 is not a 28. f ( x) = ( x − 2)5
polynomial function because it contains Using the graph of y = x5 , shift the graph
fractional exponents. horizontally to the right 2 units.
1
23. F ( x) = 5 x 4 − π x3 + is a polynomial function
2
of degree 4.Standard form:
1
F ( x) = 5 x 4 − π x3 + . Leading term: 5x 4 .
2
1
Constant term:
2

x2 − 5
24. F ( x) = = x −1 − 5 x −3 is not a polynomial
x3
function because it contains a negative exponent.

25. G ( x) = 2( x − 1) 2 ( x 2 + 1) = 2( x 2 − 2 x + 1)( x 2 + 1)
= 2( x 4 + x 2 − 2 x3 − 2 x + x 2 + 1) 29. f ( x) = x5 − 3
4 3 2
= 2( x − 2 x + 2 x − 2 x + 1) Using the graph of y = x5 , shift the graph
4 3 2
= 2x − 4x + 4x − 4x + 2 vertically, 3 units down.
is a polynomial function of degree 4.Standard
form: G ( x) = 2 x 4 − 4 x3 + 4 x 2 − 4 x + 2 . Leading
term: 2x 4 . Constant term: 2

26. G ( x) = −3x 2 ( x + 2)3 = −3 x 2 ( x3 + 6 x 2 + 12 x + 8)


= −3x5 − 18 x 4 − 36 x3 − 24 x 2
is a polynomial function of degree 5.Standard
form: G ( x) = −3 x5 − 18 x 4 − 36 x3 − 24 x 2 .
Leading term: −3x5 . Constant term: 0

27. f ( x) = ( x + 1) 4
Using the graph of y = x 4 , shift the graph
30. f ( x) = x 4 + 2
horizontally, 1 unit to the left.
Using the graph of y = x 4 , shift the graph
vertically up 2 units.

264
Copyright © 2015 Pearson Education, Inc.
Section 3.1: Polynomial Functions and Models

31. f ( x) =
1 4
x 34. f ( x) = − x 4
2 Using the graph of y = x 4 , reflect the graph
Using the graph of y = x 4 , compress the graph about the x-axis.
1
vertically by a factor of .
2

35. f ( x) = ( x − 1)5 + 2
Using the graph of y = x5 , shift the graph
32. f ( x) = 3x5
horizontally, 1 unit to the right, and shift
Using the graph of y = x5 , stretch the graph vertically 2 units up.
vertically by a factor of 3.

36. f ( x) = ( x + 2) 4 − 3
5
33. f ( x) = − x
Using the graph of y = x 4 , shift the graph
5
Using the graph of y = x , reflect the graph horizontally left 2 units, and shift vertically
about the x-axis. down 3 units.

265
Copyright © 2015 Pearson Education, Inc.
Chapter 3: Polynomial and Rational Functions

37. f ( x) = 2( x + 1)4 + 1 40. f ( x) = 3 − ( x + 2) 4 = −( x + 2) 4 + 3


Using the graph of y = x 4 , shift the graph Using the graph of y = x 4 , shift the graph
horizontally, 1 unit to the left, stretch vertically horizontally, 2 units to the left, reflect about the
by a factor of 2, and shift vertically 1 unit up. x-axis, and shift vertically 3 units up.

41. f ( x) = a ( x − (−1) ) ( x − 1)( x − 3)


1
38. f ( x) = ( x − 1)5 − 2 For a = 1 :
2
Using the graph of y = x5 , shift the graph (
f ( x) = ( x + 1)( x − 1)( x − 3) = x 2 − 1 ( x − 3) )
horizontally 1 unit to the right, compress 3 2
= x − 3x − x + 3
1
vertically by a factor of , and shift vertically
2 42. f ( x) = a ( x − (− 2) ) ( x − 2)( x − 3)
down 2 units. For a = 1 :
(
f ( x) = ( x + 2)( x − 2)( x − 3) = x 2 − 4 ( x − 3) )
= x3 − 3x 2 − 4 x + 12
43. f ( x) = a ( x − (−3) ) ( x − 0)( x − 4)
For a = 1 :
(
f ( x) = ( x + 3)( x)( x − 4) = x 2 + 3 x ( x − 4) )
3 2 2
= x − 4 x + 3 x − 12 x
= x3 − x 2 − 12 x
44. f ( x) = a ( x − (− 4) ) ( x − 0)( x − 2)
39. f ( x) = 4 − ( x − 2)5 = −( x − 2)5 + 4
For a = 1 :
Using the graph of y = x5 , shift the graph
horizontally, 2 units to the right, reflect about the
(
f ( x) = ( x + 4)( x)( x − 2) = x 2 + 4 x ( x − 2) )
x-axis, and shift vertically 4 units up. = x3 − 2 x 2 + 4 x 2 − 8 x
= x3 + 2 x 2 − 8 x
45. f ( x) = a ( x − (− 4) )( x − (−1) ) ( x − 2)( x − 3)
For a = 1 :
f ( x) = ( x + 4)( x + 1)( x − 2)( x − 3)
( )(
= x2 + 5x + 4 x2 − 5x + 6 )
4 3 2 3 2 2
= x − 5 x + 6 x + 5 x − 25 x + 30 x + 4 x − 20 x + 24
= x 4 − 15 x 2 + 10 x + 24

266
Copyright © 2015 Pearson Education, Inc.
Section 3.1: Polynomial Functions and Models

46. f ( x) = a ( x − (− 3) )( x − (−1) ) ( x − 2)( x − 5) 51. a. ( )


The real zeros of f ( x) = 4 x 2 + 1 ( x − 2)3
For a = 1 : is: 2, with multiplicity three.
f ( x) = ( x + 3)( x + 1)( x − 2)( x − 5)
x 2 + 1 = 0 has no real solution.
( )(
= x 2 + 4 x + 3 x 2 − 7 x + 10 ) b. The graph crosses the x-axis at 2 (odd
4 3
= x − 7 x + 10 x + 4 x − 28 x2 3 2 multiplicity).
+ 40 x + 3x 2 − 21x + 30 c. n −1 = 5 −1 = 4
= x 4 − 3 x3 − 15 x 2 + 19 x + 30 d. The function resembles y = 4 x5 for large
2 values of x .
47. f ( x) = a ( x − (−1) )( x − 3)
For a = 1 : 52. a. The real zeros of f ( x) = 2 ( x − 3) ( x + 4)3
f ( x) = ( x + 1)( x − 3) 2 are: 3, with multiplicity one; and –4, with
(
= ( x + 1) x 2 − 6 x + 9 ) multiplicity three.

= x3 − 6 x 2 + 9 x + x 2 − 6 x + 9 b. The graph crosses the x-axis at 3 and at –4


(odd multiplicities).
= x3 − 5 x 2 + 3 x + 9
d. n −1 = 4 −1 = 3
2
48. f ( x) = a ( x − (− 2) ) ( x − 4) e. The function resembles y = 2 x 4 for large
For a = 1 : values of x .
f ( x) = ( x + 2) 2 ( x − 4)
( )
= x2 + 4 x + 4 ( x − 4) 53. a. The real zero of
2
 1 1
( )
2
3 2 2
= x − 4 x + 4 x − 16 x + 4 x − 16 f ( x) = − 2  x +  x 2 + 4 is: − , with
 2  2
3
= x − 12 x − 16 2
multiplicity two. x + 4 = 0 has no real
49. a. The real zeros of f ( x) = 3( x − 7)( x + 3) 2 solution.
are: 7, with multiplicity one; and –3, with 1
multiplicity two. b. The graph touches the x-axis at − (even
2
b. The graph crosses the x-axis at 7 (odd multiplicity).
multiplicity) and touches it at –3 (even c. n −1 = 6 −1 = 5
multiplicity).
d. The function resembles y = −2 x 6 for large
c. n −1 = 3 −1 = 2 values of x .
3
d. The function resembles y = 3x for large 2
 1 3
values of x . 54. a. The real zeros of f ( x) =  x −  ( x − 1)
 3
1
50. a. The real zeros of f ( x) = 4( x + 4)( x + 3)3 are: , with multiplicity two; and 1, with
3
are: –4, with multiplicity one; and –3,
multiplicity 3.
with multiplicity three.
1
b. The graph crosses the x-axis at –4 and at –3 b. The graph touches the x-axis at (even
(odd multiplicities). 3
multiplicity), and crosses the x-axis at 1
c. n −1 = 4 −1 = 3 (odd multiplicity).
d. The function resembles y = 4 x 4 for large c. n −1 = 5 −1 = 4
values of x . d. The function resembles y = x5 for large
values of x .

267
Copyright © 2015 Pearson Education, Inc.
Chapter 3: Polynomial and Rational Functions

55. a. The real zeros of f ( x) = ( x − 5)3 ( x + 4) 2 b. The graph touches the x-axis at 0 (even
multiplicity) and crosses the x-axis at
are: 5, with multiplicity three; and –4, with
multiplicity two. − 2 and 2 (odd multiplicities).
b. The graph crosses the x-axis at 5 (odd c. n −1 = 4 −1 = 3
multiplicity) and touches it at –4 (even
d. The function resembles y = −2 x 4 for large
multiplicity).
values of x .
c. n −1 = 5 −1 = 4
d. The function resembles y = x5 for large 60. a. The real zeros of f ( x) = 4 x( x 2 − 3) are:
values of x . − 3, 3 and 0, with multiplicity one.

( )
2
56. a. The real zeros of f ( x) = x + 3 ( x − 2) 4 b. The graph crosses the x-axis at − 3 , 3
and 0 (odd multiplicities).
are: − 3 , with multiplicity two; and 2, with
multiplicity four. c. n −1 = 3 −1 = 2

b. The graph touches the x-axis at − 3 and at d. The function resembles y = 4 x3 for large
2 (even multiplicities). values of x .
c. n −1 = 6 −1 = 5
61. The graph could be the graph of a polynomial
d. The function resembles y = x 6 for large function.; zeros: −1, 1, 2 ; min degree = 3
values of x . 62. The graph could be the graph of a polynomial.;
zeros: −1, 2 ; min degree = 4
( )( )
2
57. a. f ( x) = 3 x 2 + 8 x 2 + 9 has no real zeros. 63. The graph cannot be the graph of a polynomial.;
2 2 not continuous at x = −1
x + 8 = 0 and x + 9 = 0 have no real
solutions. 64. The graph cannot be the graph of a polynomial.;
not smooth at x = 0
b. The graph neither touches nor crosses the x-
axis. 65. The graph crosses the x-axis at x = 0 , x = 1 , and
c. n −1 = 6 −1 = 5 x = 2 . Thus, each of these zeros has an odd
multiplicity. Using one for each of these
d. The function resembles y = 3 x 6 for large multiplicities, a possible function is
values of x . f ( x) = ax( x − 1)( x − 2) . Since the y-intercept is
0, we know f (0) = 0 . Thus, a can be any
( )
3
58. a. f ( x) = − 2 x 2 + 3 has no real zeros. positive constant. Using a = 1 , the function is
f ( x) = x( x − 1)( x − 2) .
x 2 + 3 = 0 has no real solutions.
b. The graph neither touches nor crosses the x- 66. The graph crosses the x-axis at x = 0 and x = 2
axis. and touches at x = 1 . Thus, 0 and 2 each have
c. n −1 = 6 −1 = 5 odd multiplicities while 1 has an even
multiplicity. Using one for each odd multiplicity
d. The function resembles y = −2 x 6 for large
and two for the even multiplicity, a possible
values of x . function is f ( x) = ax( x − 1) 2 ( x − 2) . Since the
y-intercept is 0, we know f (0) = 0 . Thus, a can
59. a. The real zeros of f ( x) = −2 x 2 ( x 2 − 2) are:
be any positive constant. Using a = 1 , the
− 2 and 2 with multiplicity one; and 0,
function is f ( x) = x( x − 1) 2 ( x − 2) .
with multiplicity two.
67. The graph crosses the x-axis at x = −1 and x = 2
and touches it at x = 1 . Thus, −1 and 2 each

268
Copyright © 2015 Pearson Education, Inc.
Section 3.1: Polynomial Functions and Models

have odd multiplicities while 1 has an even a(3 − 4)(3 − 1)(3 + 1) 2 = 8


multiplicity. Using one for each odd multiplicity
and two for the even multiplicity, a possible a( −1)(2)(4) 2 = 8
funtion is f ( x) = a ( x + 1)( x − 1) 2 ( x − 2) . Since −32a = 8
the y-intercept is 1, we know f (0) = 1 . Thus, a = −0.25
The function is
a(0 + 1)(0 − 1)(0 − 2) = 1
f ( x) = −0.25( x − 4)( x − 1)( x + 1) 2 .
a(1)(−1)(−2) = 1
2a = 1 71. The graph crosses the x-axis at x = 0 and touches
1 it at x = −3 and x = 3 . Thus, −3 and 3 each
a=
2 have even multiplicities while 0 has an odd
1 multiplicity. Using one for each odd multiplicity
The function is f ( x) = − ( x + 1)( x − 1) 2 ( x − 2) . and two for the even multiplicity, a possible
2
funtion is f ( x) = ax( x + 3) 2 ( x − 3) 2 . We know
68. The graph crosses the x-axis at x = −1 , x = 1 , f (2) = −50 . Thus,
and x = 2 . Thus, each of these zeros has an odd
a(2)(2 + 3) 2 (2 − 3) 2 = −50
multiplicity. Using one for each of these
multiplicities, a possible funtion is a(2)(5) 2 ( −1) 2 = −50
f ( x) = a( x + 1)( x − 1)( x − 2) . We know 50a = −50
f (0) = −1 . Thus, a = −1
a(0 + 1)(0 − 1)(0 − 2) = −1 The function is f ( x) = − x( x + 3) 2 ( x − 3) 2 .
a (1)(−1)(−2) = −1
2a = −1 72. The graph touches the x-axis at x = 0 and crosses
it at x = −3 and x = −1 and x = 2 . Thus, −3
1
a=− and −1 and 2 each have odd multiplicities while
2 0 has an even multiplicity. Using one for each
1 odd multiplicity and two for the even
The function is f ( x) = − ( x + 1)( x − 1)( x − 2) .
2 multiplicity, a possible funtion is
69. The graph crosses the x-axis at x = −4 and x = 3 f ( x) = ax 2 ( x + 3)( x + 1)( x − 2) . We know
and touches it at x = −1 . Thus, −4 and 3 each
f ( −2) = 16 . Thus,
have odd multiplicities while −1 has an even
multiplicity. Using one for each odd multiplicity a( −2) 2 ( −2 + 3)( −2 + 1)( −2 − 2) = 16
and two for the even multiplicity, a possible a ( −2) 2 (1)( −1)( −4) = 16
funtion is f ( x) = a ( x + 4)( x + 1) 2 ( x − 3) . We
16a = 16
know f (1) = −8 . Thus, a =1
a(1 + 4)(1 + 1) 2 (1 − 3) = −8 2
The function is f ( x) = x ( x + 3)( x + 1)( x − 2) .
a(5)(2) 2 ( −2) = −8
−40a = −8
73. f ( x) = x 2 ( x − 3)
a = 0.2
The function is f ( x) = 0.2( x + 4)( x + 1) 2 ( x − 3) . Step 1: Degree is 3. The function resembles
y = x3 for large values of x .
70. The graph crosses the x-axis at x = 4 and x = 1
and touches it at x = −1 . Thus, 4 and 1 each Step 2: y-intercept: f (0) = 02 (0 − 3) = 0
have odd multiplicities while −1 has an even x-intercepts: solve f ( x) = 0
multiplicity. Using one for each odd multiplicity
0 = x 2 ( x − 3)
and two for the even multiplicity, a possible
funtion is f ( x) = a ( x − 4)( x − 1)( x + 1) 2 . We x = 0, x = 3
know f (3) = 8 . Thus, Step 3: Real zeros: 0 with multiplicity two, 3
with multiplicity one. The graph

269
Copyright © 2015 Pearson Education, Inc.
Chapter 3: Polynomial and Rational Functions

touches the x-axis at x = 0 and crosses Step 2: y-intercept: f (0) = (0 + 4)(0 − 2) 2 = 16


the x-axis at x = 3 .
x-intercepts: solve f ( x) = 0
Step 4: 3 − 1 = 2
0 = ( x + 4)( x − 2) 2
Step 5: Graphing by hand: x = −4, 2
Step 3: Real zeros: −4 with multiplicity one, 2
with multiplicity two. The graph
crosses the x-axis at x = −4 and
touches the x-axis at x = 2
Step 4: 3 − 1 = 2

Step 5: Graphing by hand;

74. f ( x) = x( x + 2) 2
Step 1: Degree is 3. The function resembles
y = x3 for large values of x .

Step 2: y-intercept: f (0) = 0(0 + 2) 2 = 0


x-intercepts: solve f ( x) = 0
0 = x( x + 2) 2
x = 0, − 2
76. f ( x) = ( x − 1)( x + 3) 2
Step 3: Real zeros: −2 with multiplicity two, 0 Step 1: Degree is 3. The function resembles
with multiplicity one. The graph
y = x3 for large values of x .
touches the x-axis at x = −2 and
crosses the x-axis at x = 0 . Step 2: y-intercept: f (0) = (0 − 1)(0 + 3)2 = −9
Step 4: 3 − 1 = 2
x-intercepts: solve f ( x) = 0
Step 5: Graphing by hand: 0 = ( x − 1)( x + 3) 2
x = 1, −3
Step 3: Real zeros: −3 with multiplicity two,
1 with multiplicity one. The graph
touches the x-axis at x = −3 and
crosses it at x = 1 .
Step 4: 3 − 1 = 2

Step 5: Graphing by hand;

75. f ( x) = ( x + 4)( x − 2)2


Step 1: Degree is 3. The function resembles
y = x3 for large values of x .

270
Copyright © 2015 Pearson Education, Inc.
Section 3.1: Polynomial Functions and Models

77. f ( x) = −2( x + 2)( x − 2)3


Step 1: Degree is 4. The function resembles
y = −2 x 4 for large values of x .

Step 2: y-intercept: f (0) = −2(0 + 2)(0 − 2)3 = 32


x-intercepts: solve f ( x) = 0
0 = −2( x + 2)( x − 2)3
x = −2, 2
Step 3: Real zeros: −2 with multiplicity one,
2 with multiplicity three. The graph
crosses the x-axis at x = −2 and x = 2 .
Step 4: 4 − 1 = 3 79. f ( x) = ( x + 4) 2 (1 − x)
Step 1: Degree is 3. The function resembles
Step 5: Graphing by hand; y = − x3 for large values of x .

Step 2: y-intercept: f (0) = (1 − 0)(0 + 4) 2 = 16


x-intercepts: solve f ( x) = 0
0 = (1 − x)( x + 4) 2
x = 1, −4
Step 3: Real zeros: −4 with multiplicity two,
1 with multiplicity one. The graph
touches the x-axis at x = −4 and
crosses it at x = 1 .
Step 4: 3 − 1 = 2
1
78. f ( x) = − ( x + 4)( x − 1)3
2 Step 5: Graphing by hand;
Step 1: Degree is 4. The function resembles
1
y = − x 4 for large value of x .
2
1
Step 2: y-intercept: f (0) = − (0 + 4)(0 − 1)3 = 2
2
x-intercepts: solve f ( x) = 0
1
0 = − ( x + 4)( x − 1)3
2
x = −4, 1
Step 3: Real zeros: −4 with multiplicity one,
1 with multiplicity three. The graph 80. f ( x) = (3 − x) ( 2 + x ) ( x + 1)
crosses the x-axis at x = −4 and x = 1 .
Step 4: 4 − 1 = 3 Step 1: Degree is 3. The function resembles
y = − x3 for large values of x .
Step 5: Graphing by hand;
Step 2: y-intercept: f (0) = (3 − 0) ( 2 + 0) (0 + 1)
=6
x-intercepts: solve f ( x) = 0
0 = (3 − x) ( 2 + x ) ( x + 1)
x = 3, − 2, − 1

271
Copyright © 2015 Pearson Education, Inc.
Chapter 3: Polynomial and Rational Functions

Step 3: Real zeros: 3 with multiplicity one, Step 2: y-intercept: f (0) = ( 0 − 1) (0 + 4)(0 − 3)
−2 with multiplicity one, −1 with
= −12
multiplicity one. The graph crosses the
x-axis at x = 3, −2, −1 . x-intercepts: solve f ( x) = 0
0 = ( x − 1) ( x + 4)( x − 3)
Step 4: 3 − 1 = 2
x = 1, − 4, 3
Step 5: Graphing by hand;
Step 3: Real zeros: −4 with multiplicity one,
1 with multiplicity one, 3 with
multiplicity one. The graph crosses the
x-axis at x = −4, 1, 3 .
Step 4: 3 − 1 = 2
Step 5: Graphing by hand;

81. f ( x) = ( x + 1) ( x − 2 ) ( x + 4)
Step 1: Degree is 3. The function resembles
y = x3 for large values of x .

Step 2: y-intercept: f (0) = (0 + 1) ( 0 − 2 ) (0 + 4)


= −8
x-intercepts: solve f ( x) = 0 83. f ( x) = x 2 ( x − 2)( x + 2)
0 = ( x + 1) ( x − 2 ) ( x + 4)
Step 1: Degree is 4. The function resembles
x = −1, 2, − 4 y = x 4 for large values of x .
Step 3: Real zeros: −4 with multiplicity one,
−1 with multiplicity one, 2 with Step 2: y-intercept: f ( x) = 02 (0 − 2)(0 + 2) = 0
multiplicity one. The graph crosses the x-intercepts: solve f ( x) = 0
x-axis at x = −4, −1, 2 .
0 = x 2 ( x − 2)( x + 2)
Step 4: 3 − 1 = 2 x = 0, 2, − 2
Step 5: Graphing by hand; Step 3: Real zeros: −2 with multiplicity one,
0 with multiplicity two, 2 with
multiplicity one. The graph crosses the
x-axis at x = −2 and x = −2 , and
touches it at x = 0.
Step 4: 4 − 1 = 3

Step 5: Graphing by hand

82. f ( x) = ( x − 1) ( x + 4)( x − 3)
Step 1: Degree is 3. The function resembles
y = x3 for large values of x

272
Copyright © 2015 Pearson Education, Inc.
Section 3.1: Polynomial Functions and Models

Step 5: Graphing by hand:


2
84. f ( x) = x ( x − 3)( x + 4)
Step 1: Degree is 4. The graph of the function
resembles y = x 4 for large values of x .

Step 2: y-intercept: f (0) = 02 (0 − 3)(0 + 4) = 0


x-intercept: solve f ( x) = 0
x 2 ( x − 3)( x + 4) = 0
x = 0 or x = 3 or x = −4
86. f ( x) = ( x + 2) 2 ( x − 4) 2
Step 3: Real zeros: −4 with multiplicity one,
0 with multiplicity two, 3 with Step 1: Degree is 4. The graph of the function
multiplicity one. The graph crosses the resembles y = x 4 for large values of x .
x-axis at x = 3 and x = –4, and touches
it at x = 0. Step 2: y-intercept: f (0) = (0 + 2) 2 (0 − 4) 2 = 64
Step 4: 4 − 1 = 3 x-intercept: solve f ( x) = 0
Step 5: Graphing by hand: ( x + 2) 2 ( x − 4) 2 = 0
x = −2 or x = 4
Step 3: Real zeros: −2 with multiplicity two,
4 with multiplicity two. The graph
touches the x-axis at x = −2 and x = 4 .
Step 4: 4 − 1 = 3

Step 5: Graphing by hand;

85. f ( x) = ( x + 1) 2 ( x − 2) 2
Step 1: Degree is 4. The graph of the function
resembles y = x 4 for large values of x .

Step 2: y-intercept: f (0) = (0 + 1) 2 (0 − 2) 2 = 4


x-intercepts: solve f ( x) = 0 87. f ( x) = x (1 − x ) (2 − x)
( x + 1) 2 ( x − 2) 2 = 0
Step 1: Degree is 3. The function resembles
x = −1 or x = 2
y = x3 for large values of x .
Step 3: Real zeros: −1 with multiplicity two,
2 with multiplicity two. The graph Step 2: y-intercept: f (0) = (0) (1 − 0) (2 − 0)
touches the x-axis at x = −1 and x = 2 . =0
Step 4: 4 − 1 = 3 x-intercepts: solve f ( x) = 0
0 = x (1 − x ) (2 − x)
x = 0,1, 2
Step 3: Real zeros: 0 with multiplicity one,
1 with multiplicity one, 2 with
multiplicity one. The graph crosses the
x-axis at x = 0,1, 2 .
Step 4: 3 − 1 = 2

273
Copyright © 2015 Pearson Education, Inc.
Chapter 3: Polynomial and Rational Functions

Step 5: Graphing by hand; x-axis at x = 0, and crosses it at x = −3


and x = –1.

Step 4: 4 − 1 = 3
Step 5: Graphing by hand;

88. f ( x) = x(3 − x) 2
Step 1: Degree is 3. The function resembles
y = − x3 for large values of x . 90. f ( x) = x 2 ( x − 3)( x − 1)

Step 2: y-intercept: f (0) = 0(3 − 0) 2 = 0 Step 1: Degree is 4. The graph of the function
resembles y = x 4 for large values of x .
x-intercepts: solve f ( x) = 0
0 = x (3 − x) 2 Step 2: y-intercept: f (0) = 02 (0 − 3)(0 − 1) = 0
x = 0, 3 x-intercept: solve f ( x) = 0
Step 3: Real zeros: 3 with multiplicity two, 0 0 = x 2 ( x − 3)( x − 1)
with multiplicity one. The graph x = 0, 3, 1
touches the x-axis at x = 3 and crosses
the x-axis at x = 0 . Step 3: Real zeros: 0 with multiplicity two,
3 with multiplicity one, 1 with
Step 4: 3 − 1 = 2 multiplicity one. The graph touches the
x-axis at x = 0 , and crosses it at x = 3
Step 5: Graphing by hand: and x = 1.
Step 4: 4 − 1 = 3
Step 5: Graphing by hand;

89. f ( x) = x 2 ( x + 3)( x + 1)
Step 1: Degree is 4. The graph of the function
resembles y = x 4 for large values of x .
91. f ( x) = ( x − 2) 2 ( x + 2)( x + 4)
2
Step 2: y-intercept: f (0) = (0) (0 + 3)(0 + 1) = 3 Step 1: Degree is 4. The graph of the function
x-intercept: solve f ( x) = 0 resembles y = x 4 for large values of x .
x 2 ( x + 3)( x + 1) = 0
x = 0 or x = −3 or x = −1
Step 3: Real zeros: 0 with multiplicity two,
−3 with multiplicity one, −1 with
multiplicity one. The graph touches the

274
Copyright © 2015 Pearson Education, Inc.
Section 3.1: Polynomial Functions and Models

Step 2: y-intercept: f (0) = (0 − 2) 2 (0 + 2)(0 + 4) 93. f ( x) = 5 x( x 2 − 4)( x + 3)


= 32 Step 1: Degree is 4. The graph of the function
x-intercept: solve f ( x) = 0 resembles y = 5 x 4 for large values of x .
( x − 2) 2 ( x + 2)( x + 4) = 0
Step 2: y-intercept: f (0) = 5(0)(02 − 4)(0 + 3) = 0
x = 2 or x = −2 or x = −4
x-intercept: solve f ( x) = 0
Step 3: Real zeros: 2 with multiplicity two,
−2 with multiplicity one, −4 with 5 x( x 2 − 4)( x + 3) = 0
multiplicity one. The graph touches the x = 0, −2, 2, −3
x-axis at x = 2 , and crosses it at x = −2
and x = −4 . Step 3: Real zeros: 0 with multiplicity two,
-2 with multiplicity one, 2 with
Step 4: 4 − 1 = 3 multiplicity one and -3 with multiplicity
one. The graph crosses the x-axis at
Step 5: Graphing by hand: x = 0 , x = −2 , x = 2 and x = −3 .
Step 4: 4 − 1 = 3

Step 5: Graphing by hand:

3
92. f ( x) = ( x + 1) ( x − 3)
Step 1: Degree is 4. The graph of the function
resembles y = x 4 for large values of x . 94. f ( x) = −2( x − 1)2 ( x 2 − 16)

3
Step 1: Degree is 4. The graph of the function
Step 2: y-intercept: f (0) = ( 0 + 1) ( 0 − 3) = −3 resembles y = −2 x 4 for large values of
x-intercept: solve f ( x) = 0 x .
( x + 1)3 ( x − 3) = 0
Step 2: y-intercept:
x = −1 or x = 3
f (0) = −2(0 − 1) 2 (02 − 16) = 32
Step 3: Real zeros: −1 with multiplicity three, x-intercept: solve f ( x) = 0
3 with multiplicity one. The graph
crosses the x-axis at x = –1 and x = 3. −2( x − 1) 2 ( x 2 − 16) = 0
Step 4: 4 − 1 = 3 x = 1, −4, 4
Step 3: Real zeros: 1 with multiplicity two,
Step 5: Graphing by hand; -4 with multiplicity one and 4 with
multiplicity one. The graph touches the x-
axis at x = 1 and crosses the x-axis at
x = −4 and x = 4 .
Step 4: 4 − 1 = 3

275
Copyright © 2015 Pearson Education, Inc.
Chapter 3: Polynomial and Rational Functions

Step 5: Graphing by hand: Step 4: 5 − 1 = 4

Step 5: Graphing by hand:

95. f ( x) = x 2 ( x − 2)( x 2 + 3)
Step 1: Degree is 5. The graph of the function
resembles y = x5 for large values of x .
97. f ( x) = x3 + 0.2 x 2 − 1.5876 x − 0.31752
Step 2: y-intercept: f (0) = 02 (0 − 2)(02 + 3) = 0
Step 1: Degree = 3; The graph of the function
x-intercept: solve f ( x) = 0
resembles y = x3 for large values of x .
x 2 ( x − 2)( x 2 + 3) = 0
x = 0 or x = 2 Step 2: Graphing utility
Note: x 2 + 3 = 0 has no real solution.
Step 3: Real zeros: 0 with multiplicity two,
2 with multiplicity one. The graph
touches the x-axis at x = 0 and crosses
it x = 2 .
Step 4: 5 − 1 = 4

Step 5: Graphing by hand: Step 3: x-intercepts: –1.26, –0.20, 1.26;


y-intercept: –0.31752
Step 4:

Step 5: 2 turning points;


local maximum: (–0.80, 0.57);
local minimum: (0.66, –0.99)
96. f ( x) = x 2 ( x 2 + 1)( x + 4)
Step 6: Graphing by hand
Step 1: Degree is 5. The graph of the function
resembles y = x5 for large values of x .

Step 2: y-intercept: f (0) = 02 (02 + 1)(0 + 4) = 0


x-intercept: Solve f ( x) = 0
x 2 ( x 2 + 1)( x + 4) = 0
x = 0 or x = −4
Note: x 2 + 1 = 0 has no real solution.
Step 3: Real zeros: 0 with multiplicity two,
−4 with multiplicity one. The graph Step 7: Domain: ( −∞, ∞ ) ; Range: ( −∞, ∞ )
touches the x-axis at x = 0 and crosses
it at x = −4 .

276
Copyright © 2015 Pearson Education, Inc.
Section 3.1: Polynomial Functions and Models

Step 8: Increasing on ( −∞, −0.80 ) and ( 0.66, ∞ ) ; Step 2: Graphing utility


decreasing on ( −0.80, 0.66 )

98. f ( x) = x3 − 0.8 x 2 − 4.6656 x + 3.73248


Step 1: Degree = 3; The graph of the function
resembles y = x3 for large values of x .
Step 2: Graphing utility
Step 3: x-intercepts: –3.56, 0.50;
y-intercept: 0.89
Step 4:

Step 3: x-intercepts: –2.16, 0.8, 2.16;


y-intercept: 3.73248 Step 5: 2 turning points;
Step 4: local maximum: (–2.21, 9.91);
local minimum: (0.50, 0)
Step 6: Graphing by hand

Step 5: 2 turning points;


local maximum: (–1.01, 6.60);
local minimum: (1.54, –1.70)
Step 6: Graphing by hand

Step 7: Domain: ( −∞, ∞ ) ; Range: ( −∞, ∞ )


Step 8: Increasing on ( −∞, −2.21) and ( 0.50, ∞ ) ;
decreasing on ( −2.21, 0.50 ) .

100. f ( x) = x3 − 2.91x 2 − 7.668 x − 3.8151


Step 7: Domain: ( −∞, ∞ ) ; Range: ( −∞, ∞ ) Step 1: Degree = 3; The graph of the function
Step 8: Increasing on ( −∞, −1.01) and (1.54, ∞ ) ; resembles y = x3 for large values of x .
decreasing on ( −1.01,1.54 ) Step 2: Graphing utility

99. f ( x) = x3 + 2.56 x 2 − 3.31x + 0.89


Step 1: Degree = 3; The graph of the function
resembles y = x3 for large values of x .

Step 3: x-intercepts: –0.9, 4.71;


y-intercept: –3.8151

277
Copyright © 2015 Pearson Education, Inc.
Chapter 3: Polynomial and Rational Functions

Step 4: Step 6: Graphing by hand

Step 5: 2 turning points;


local maximum: (–0.9, 0);
local minimum: (2.84, –26.16)
Step 6: Graphing by hand
Step 7: Domain: ( −∞, ∞ ) ; Range: [ −1, ∞ )
Step 8: Increasing on ( −1.12, 0 ) and (1.12, ∞ ) ;
decreasing on ( −∞, −1.12 ) and ( 0,1.12 )

102. f ( x) = x 4 − 18.5 x 2 + 50.2619


Step 1: Degree = 4; The graph of the function
resembles y = x 4 for large values of x .
Step 7: Domain: ( −∞, ∞ ) ; Range: ( −∞, ∞ )
Step 2: Graphing utility
Step 8: Increasing on ( −∞, −0.9 ) and ( 2.84, ∞ ) ;
decreasing on ( −0.9, 2.84 ) .

101. f ( x) = x 4 − 2.5 x 2 + 0.5625


Step 1: Degree = 4; The graph of the function
resembles y = x 4 for large values of x .

Step 2: Graphing utility Step 3: x-intercepts: –3.90, –1.82, 1.82, 3.90;


y-intercept: 50.2619
Step 4:

Step 3: x-intercepts: –1.5, –0.5, 0.5,1.5; Step 5: 3 turning points:


y-intercept: 0.5625 local maximum: (0, 50.26);
local minima: (–3.04, –35.30),
Step 4: (3.04, –35.30)
Step 6: Graphing by hand

Step 5: 3 turning points:


local maximum: (0, 0.5625);
local minima: (–1.12, –1), (1.12, –1)

278
Copyright © 2015 Pearson Education, Inc.
Section 3.1: Polynomial Functions and Models

Step 7: Domain: ( −∞, ∞ ) ; Range: [ −35.30, ∞ ) Step 2: Graphing utility

Step 8: Increasing on ( −3.04, 0 ) and ( 3.04, ∞ ) ;


decreasing on (−∞, −3.04) and (0, 3.04)

103. f ( x ) = 2 x 4 − π x3 + 5 x − 4

Step 1: Degree = 4; The graph of the function


resembles y = 2 x 4 for large values of
Step 3: x-intercepts: –1.47, 0.91;
x .
y-intercept: 2
Step 2: Graphing utility: Step 4:

Step 5: 1 turning point:


local maximum: (–0.81, 3.21)
Step 6: Graphing by hand
Step 3: x-intercepts: –1.07, 1.62;
y-intercept: –4
Step 4:

Step 5: 1 turning point;


local minimum: (–0.42, –4.64)
Step 7: Domain: ( −∞, ∞ ) ; Range: ( −∞, 3.21]
Step 6: Graphing by hand
Step 8: Increasing on ( −∞, −0.81) ;
decreasing on ( −0.81, ∞ )

105. f ( x) = 4 x − x3 = − x( x 2 − 4) = − x( x + 2)( x − 2)
Step 1: Degree is 3. The function resembles
y = − x3 for large values of x .

Step 2: y-intercept: f (0) = 4(0) − 03 = 0


x-intercepts: Solve f ( x) = 0
Step 7: Domain: ( −∞, ∞ ) ; Range: [ −4.64, ∞ )
0 = − x( x + 2)( x − 2)
Step 8: Increasing on ( −0.42, ∞ ) ; x = 0, −2, 2
decreasing on ( −∞, −0.42 )
Step 3: Real zeros: 0 with multiplicity one,
−2 with multiplicity one, 2 with
104. f ( x ) = −1.2 x 4 + 0.5 x 2 − 3x + 2 multiplicity one. The graph crosses the
Step 1: Degree = 4; The graph of the function x-axis at x = 0, x = −2, and x = 2.
resembles y = −1.2 x 4 for large values Step 4: 3 − 1 = 2
of x .

279
Copyright © 2015 Pearson Education, Inc.
Chapter 3: Polynomial and Rational Functions

Step 5: Graphing by hand Step 3: Real zeros: 0 with multiplicity one,


−4 with multiplicity one, 3 with
multiplicity one. The graph crosses the
x-axis at x = 0, x = −4, and x = 3.
Step 4: 3 − 1 = 2
Step 5: Graphing by hand

106. f ( x) = x − x3 = − x( x 2 − 1) = − x( x + 1)( x − 1)
Step 1: Degree is 3. The function resembles
y = − x3 for large values of x .

Step 2: y-intercept: f (0) = 0 − 03 = 0


x-intercepts: Solve f ( x) = 0
108. f ( x) = x3 + 2 x 2 − 8 x
0 = − x( x + 1)( x − 1)
x = 0, −1, 1 = x( x 2 + 2 x − 8)
Step 3: Real zeros: 0 with multiplicity one, = x( x + 4)( x − 2)
−1 with multiplicity one, 1 with Step 1: Degree is 3. The function resembles
multiplicity one. The graph crosses the y = x3 for large values of x .
x-axis at x = 0, x = −1, and x = 1 .
Step 4: 3 − 1 = 2 Step 2: y-intercept: f (0) = 03 + 2(0) 2 − 8(0) = 0
Step 5: Graphing by hand x-intercepts: Solve f ( x) = 0
0 = x( x + 4)( x − 2)
x = 0, −4, 2
Step 3: Real zeros: 0 with multiplicity one,
−4 with multiplicity one, 2 with
multiplicity one. The graph crosses the
x-axis at x = 0, x = −4, and x = 2.
Step 4: 3 − 1 = 2
Step 5: Graphing by hand

107. f ( x) = x3 + x 2 − 12 x
= x( x 2 + x − 12)
= x( x + 4)( x − 3)
Step 1: Degree is 3. The function resembles
y = x3 for large values of x .

Step 2: y-intercept: f (0) = 03 + 02 − 12(0) = 0


x-intercepts: Solve f ( x) = 0
0 = x( x + 4)( x − 3)
x = 0, −4, 3

280
Copyright © 2015 Pearson Education, Inc.
Section 3.1: Polynomial Functions and Models

109. f ( x) = 2 x 4 + 12 x3 − 8 x 2 − 48 x 5
Step 3: Real zeros: − with multiplicity one,
3 2 2
= 2 x( x + 6 x − 4 x − 24)
−1 with multiplicity one, 1 with
= 2 x  x 2 ( x + 6) − 4( x + 6)  multiplicity one. The graph crosses the
5
= 2 x( x + 6)( x 2 − 4) x-axis at x = − , x = −1, and x = 1.
2
= 2 x( x + 6)( x − 2)( x + 2)
Step 4: 3 − 1 = 2
Step 1: Degree is 4. The function resembles Step 5: Graphing by hand
y = 2 x 4 for large values of x .
Step 2: y-intercept:
f (0) = 2(0) 4 + 12(0)3 − 8(0)2 − 48(0) = 0
x-intercepts: Solve f ( x) = 0
0 = 2 x( x + 6)( x − 2)( x + 2)
x = 0, −6, 2, −2
Step 3: Real zeros: 0 with multiplicity one,
−6 with multiplicity one, 2 with
multiplicity one, −2 with multiplicity
111. f ( x) = − x5 − x 4 + x3 + x 2
one. The graph crosses the x-axis at
x = 0, x = −6, x = 2, and x = −2. = − x 2 ( x3 + x 2 − x − 1)
Step 4: 3 − 1 = 2 = − x 2  x 2 ( x + 1) − 1( x + 1) 
Step 5: Graphing by hand = − x 2 ( x + 1)( x 2 − 1)
= − x 2 ( x + 1)( x + 1)( x − 1)
= − x 2 ( x + 1) 2 ( x − 1)
Step 1: Degree is 5. The graph of the function
resembles y = − x5 for large values of x .
Step 2: y-intercept:
f (0) = −(0)5 − (0) 4 + (0)3 + (0) 2 = 0
x-intercept: Solve f ( x) = 0
− x 2 ( x + 1) 2 ( x − 1) = 0
110. f ( x) = 4 x3 + 10 x 2 − 4 x − 10 x = 0, −1, 1
= 2(2 x3 + 5 x 2 − 2 x − 5 Step 3: Real zeros: 0 with multiplicity two,
−1 with multiplicity two, 1 with
= 2  x (2 x + 5) − 1(2 x + 5) 
2
multiplicity one. The graph touches the
= 2(2 x + 5)( x 2 − 1) x-axis at x = 0 and x = −1 , and crosses
it at x = 1 .
= 2(2 x + 5)( x + 1)( x − 1)
Step 4: 5 − 1 = 4
Step 1: Degree is 3. The function resembles
y = 4 x3 for large values of x .
Step 2: y-intercept:
f (0) = 4(0)3 + 10(0) 2 − 4(0) − 10 = −10
x-intercepts: solve f ( x) = 0
0 = 2(2 x + 5)( x + 1)( x − 1)
5
x = − , −1, 1
2

281
Copyright © 2015 Pearson Education, Inc.
Chapter 3: Polynomial and Rational Functions

Step 5: Graphing by hand: 114. f ( x) = a ( x + 4)( x + 1)( x − 2)


16 = a (0 + 4)(0 + 1)(0 − 2)
16 = −8a
a = −2
f ( x) = −2( x + 4)( x + 1)( x − 2)

115. f ( x) = a ( x + 5) 2 ( x − 2)( x − 4)
128 = a (3 + 5) 2 (3 − 2)(3 − 4)
128 = −64a
a = −2
112. f ( x) = − x 5 + 5 x 4 + 4 x3 − 20 x 2
f ( x) = −2( x + 5) 2 ( x − 2)( x − 4)
= − x 2 ( x 3 − 5 x 2 − 4 x + 20)
= − x 2  x 2 ( x − 5) − 4( x − 5)  116. f ( x) = ax3 ( x + 4)( x − 2)
64 = a ( −2 ) (−2 + 4)(−2 − 2)
3
= − x 2 ( x − 5)( x 2 − 4)
= − x 2 ( x − 5)( x − 2)( x + 2) 64 = 64a
Step 1: Degree is 5. The graph of the function a =1
resembles y = − x 5 for large values of x . f ( x) = x3 ( x + 4)( x − 2)
Step 2: y-intercept:
117. a. 0 = ( x + 3) 2 ( x − 2)  x = −3, x = 2
f (0) = −(0)5 + 5(0) 4 + 4(0)3 − 20(0) 2 = 0
x-intercept: solve f ( x) = 0 b. The graph is shifted to the left 3 units so the x-
2
− x ( x + 2)( x − 2)( x − 5) = 0 intercepts would be
x = −3 − 3 = −6
x = 0, −2, 2, 5
and x = 2 − 3 = −1
Step 3: Real zeros: 0 with multiplicity two,
−2 with multiplicity one, 21 with 118. a. 0 = ( x + 2)( x − 4)3  x = −2, x = 4
multiplicity one, 5 with multiplicity
one. The graph The graph touches the b. The graph is shifted to the right 2 units so the x-
x-axis at x = 0 and crosses it at x = −2, intercepts would be
x = 2, and x = 5 . x = −2 + 2 = 0
Step 4: 5 − 1 = 4 and x = 4 + 2 = 6
Step 5: Graphing by hand:
119. a. Graphing, we see that the graph may be a
cubic relation.

113. f ( x) = a ( x + 3)( x − 1)( x − 4)


36 = a (0 + 3)(0 − 1)(0 − 4)
36 = 12a
a=3
f ( x) = 3( x + 3)( x − 1)( x − 4)

282
Copyright © 2015 Pearson Education, Inc.
Section 3.1: Polynomial Functions and Models

b. The cubic function of best fit is H ( x) =


0.3948 x3 − 5.9563x 2 + 26.1965 x − 7.4127

c. Graphing the cubic function of best fit:

c. For the decade 1961-1970, we have x = 5 .


H (5) =
0.3948(5)3 − 5.9563(5)2 + 26.1695(5) − 7.4127
≈ 24
The model predicts that about 24 major d. C (11) = 0.2156(11)3 − 2.3473(11) 2
hurricanes struck the Atlantic Basin between +14.3275(11) + 10.2238
1961 and 1970.
≈ 170.8
d. The cost of manufacturing 11 Cobalts
hour would be approximately $171,000.
e. The y-intercept would indicate the fixed
costs before any cars are made and is about
$10,200.
121. a. Graphing, we see that the graph may be a
e. For the decade 2011 to 2020 we have cubic relation.
x = 10 .
H (10) =
0.3948(10)3 − 5.9563(10)2 + 26.1695(10) − 7.4127
≈ 54
The model predicts that approximately 54
major hurricanes will strike the Atlantic
Basin between 2011 and 2020. The
prediction does not seem to be reasonable.
It appears to be too high.
120. a. Graphing, we see that the graph may be a
cubic relation. ΔT T (12) − T (9) 55 − 41 14
b. = = = ≈ 4.67
Δx 12 − 9 12 − 9 3
The average rate of change in temperature
from 9am to noon was about 4.67°F per hour.
ΔT T (21) − T (18) 57 − 64
c. = = = −2.33
Δx 21 − 18 3
The average rate of change in temperature
from 6pm to 9pm was −2.33°F per hour.
b. The cubic function of best fit is C(x) =
0.2156 x3 − 2.3473x 2 + 14.3275 x + 10.2238

283
Copyright © 2015 Pearson Education, Inc.
Chapter 3: Polynomial and Rational Functions

d. The cubic function of best fit is T ( x) =


−0.02357 x3 + 0.809 x 2 − 6.2893 x + 52.0714

At 5pm we have x = 17 .
T (17 ) = −0.02357 (17 ) + 0.809 (17 )
3 2

− 6.2893 (17 ) + 52.0714


c.
= 63.15489
The predicted temperature at 5pm is ≈ 63.2°F .
e.

f. The y-intercept is approximately 52.1°F .


The model predicts that the midnight
temperature was 52.1°F .

122. a.

d. The values of the polynomial function get


closer to the values of f. The approximations
near 0 are better than those near −1 or 1.
123. a. T (r ) = 500(1 + r )(1 + r ) + 500(1 + r ) + 500

   Deposit 3
Account value of Account value of
deposit 1 deposit 2

= 500(1 + 2r + r 2 ) + 500(1 + r ) + 500


= 500 + 1000r + 500r 2 + 500 + 500r + 500
= 500r 2 + 1500r + 1500
b.

F (0.05)
= 500(.05)3 + 2000(.05) 2 + 3000(.05) + 2000
b. ≈ 2155.06

The value of the account at the beginning


of the fourth year will be $2155.06.

284
Copyright © 2015 Pearson Education, Inc.
Section 3.1: Polynomial Functions and Models

124. The graph of a polynomial function will always b. The leading coefficient is positive because
have a y-intercept since the domain of every both ends go up.
polynomial function is the set of real numbers. c. The function appears to be symmetric about
Therefore f ( 0 ) will always produce a y- the y-axis. Therefore, it is an even function.
coordinate on the graph. A polynomial function d. The graph touches the x-axis at x = 0 .
might have no x-intercepts. For example, Therefore, x n must be a factor, where n is
f ( x) = x 2 + 1 has no x-intercepts since the even and n ≥ 2 .
equation x 2 + 1 = 0 has no real solutions. e. There are six zeros with odd multiplicity and
one with even multiplicity. Therefore, the
125. Answers will vary. minimum degree is 6(1) + 1(2) = 8 .
f. Answers will vary.
126. Answers will vary, one such polynomial is
f ( x) = x 2 ( x + 1)(4 − x)( x − 2) 2 131. Answers will vary. One possibility:
 1  3
f ( x) = −5( x − 1)3 ( x − 2)  x −   x + 
127. Answers will vary , f ( x) = ( x + 2)( x − 1) 2 and  2  5
g ( x) = ( x + 2)3 ( x − 1) 2 are two such
132. We need to put the equation in standard form.
polynomials. 5x − 2 y = 6
1 −2 y = −5 x + 6
128. f ( x) = is smooth but not continuous; 5
x y = x−3
g ( x) = x is continuous but not smooth. 2

Since we are looking for a perpendicular line, the


129. f ( x) = x3 + bx 2 + cx + d 2
new slope must be m = − .
a. True since every polynomial function has 5
exactly one y-intercept, in this case (0, d ) . 2
y − y1 = − ( x − x1 )
b. True, a third degree polynomial will have at 5
most 3 x-intercepts since the equation 2
y + 3 = − ( x − 2)
x3 + bx 2 + cx + d = 0 will have at most 3 5
real solutions. 2 4
c. True, a third degree polynomial will have at y+3= − x+
5 5
least one x-intercept since the equation
2 11
x3 + bx 2 + cx + d = 0 will have at least one y = − x−
5 5
real solution.
d. True, since f has degree 3 and the leading 133. The denominator cannot be zero so the domain
coefficient 1. is: {x | x ≠ −5}
e. False, since
−b ± b 2 − 4ac
f (− x) = ( − x ) + b ( − x ) + c ( − x ) + d
3 2
134. x =
2a
= − x 3 + bx 2 − cx + d
− (8) ± (8) 2 − 4(4)( −3)
≠ − f ( x). (unless b = d = 0) =
2(4)
f. True only if d = 0 , otherwise the statement
−8 ± 64 + 48 −8 ± 112
is false. = =
8 8
130. a. The degree will be even because the ends of −8 ± 4 7 −2 ± 7
= =
the graph go in the same direction. 8 2

285
Copyright © 2015 Pearson Education, Inc.
Chapter 3: Polynomial and Rational Functions

−2 − 7 −2 + 7 11. f ( x) = 4 x3 − 3 x 2 − 8 x + 4; c = 2
So the zeros are: ,
2 2 f (2) = 4(2)3 − 3(2) 2 − 8(2) + 4
135. 5x − 3 = 7 = 32 − 12 − 16 + 4 = 8 ≠ 0
Thus, 2 is not a zero of f and x − 2 is not a
5 x − 3 = −7 or 5 x − 3 = 7 factor of f .
5 x = −4 5 x = 10
4 12. f ( x) = − 4 x3 + 5 x 2 + 8; c = − 3
x=− x=2
5 f (− 3) = − 4(− 3)3 + 5(− 3) 2 + 8
= 108 + 45 + 8 = 161 ≠ 0
4 Thus, –3 is not a zero of f and x + 3 is not a
So the solution set is − , 2
5 factor of f .

13. f ( x) = 3x 4 − 6 x3 − 5 x + 10; c = 2
Section 3.2 f (2) = 3(2) 4 − 6(2)3 − 5(2) + 10
2 = 48 − 48 − 10 + 10 = 0
1. f ( −1) = 2 ( −1) − ( −1) = 2 + 1 = 3
Thus, 2 is a zero of f and x − 2 is a factor of f .
2. 6 x 2 + x − 2 = ( 3x + 2 )( 2 x − 1)
14. f ( x ) = 4 x 4 − 15 x 2 − 4 ; c = 2
4 2
3. Using synthetic division: f ( 2 ) = 4 ( 2 ) − 15 ( 2 ) − 4 = 64 − 60 − 4 = 0
3 3 −5 0 7 −4 Thus, 2 is a zero of f and x − 2 is a factor of f .
9 12 36 129
3 4 12 43 125 15. f ( x) = 3 x 6 + 82 x3 + 27; c = − 3

Quotient: 3x3 + 4 x 2 + 12 x + 43 f (− 3) = 3(− 3)6 + 82(− 3)3 + 27


Remainder: 125 = 2187 − 2214 + 27 = 0
Thus, –3 is a zero of f and x + 3 is a factor
4. x 2 + x − 3 = 0
of f .
−1 ± 12 − 4 (1)( −3)
x= 16. f ( x ) = 2 x 6 − 18 x 4 + x 2 − 9 ; c = −3
2 (1)
6 4 2
−1 ± 1 + 12 −1 ± 13 f ( −3) = 2 ( −3) − 18 ( −3) + ( −3) − 9
= = = 1458 − 1458 + 9 − 9 = 0
2 2
Thus, –3 is a zero of f and x + 3 is a factor
 −1 − 13 −1 + 13 
The solution set is  , . of f .
 2 2 

5. Remainder, Dividend 17. f ( x) = 4 x6 − 64 x 4 + x 2 − 15; c = −4


6 4 2
6. f (c) f (−4) = 4 ( −4 ) − 64 ( −4 ) + ( −4 ) − 15
= 16,384 − 16,384 + 16 − 15 = 1 ≠ 0
7. −4 Thus, –4 is not a zero of f and x + 4 is not a
8. False; every polynomial function of degree 3 factor of f .
with real coefficients has at most three real zeros.
18. f ( x ) = x 6 − 16 x 4 + x 2 − 16 ; c = −4
9. 0.
6 4 2
f ( −4 ) = ( −4 ) − 16 ( −4 ) + ( −4 ) − 16
10. True
= 4096 − 4096 + 16 − 16 = 0

286
Copyright © 2015 Pearson Education, Inc.
Section 3.2: The Real Zeros of a Polynomial Function

Thus, –4 is a zero of f and x + 4 is a factor 23. f ( x ) = 2 x6 − 3x 2 − x + 1


of f . The maximum number of zeros is the degree of
the polynomial, which is 6.
1
19. f ( x) = 2 x 4 − x3 + 2 x − 1; c = Examining f ( x ) = 2 x6 − 3 x 2 − x + 1 , there are
2
4 3
two variations in sign; thus, there are two
1 1 1 1 positive real zeros or no positive real zeros.
f   = 2  −   + 2   −1
2 2 2 2 Examining
6 2
1 1 f (−x) = 2 ( −x) − 3( −x) − ( −x) +1 ,
= − +1−1 = 0
8 8 = 2 x6 − 3x 2 + x + 1
1 1 there are two variations in sign; thus, there are
Thus, is a zero of f and x − is a factor of f .
2 2 two negative real zeros or no negative real zeros.

20. f ( x) = 3x 4 + x3 − 3x + 1; c = −
1 24. f ( x ) = −3 x5 + 4 x 4 + 2
3 The maximum number of zeros is the degree of
4 3 the polynomial, which is 5.
 1  1  1  1
f  −  = 3 −  +  −  − 3 −  +1
 3   3   3   3 Examining f ( x ) = −3 x5 + 4 x 4 + 2 , there is one
1 1 variation in sign; thus, there is one positive real
= − +1+1 = 2 ≠ 0 zero.
27 27
Examining
1 1
Thus, − is not a zero of f and x + is not a 5 4
f ( − x ) = −3 ( − x ) + 4 ( − x ) + 2 ,
3 3
factor of f. = 3x5 + 4 x 4 + 2
there is no variation in sign; thus, there are no
21. f ( x ) = −4 x 7 + x3 − x 2 + 2 negative real zeros.
The maximum number of zeros is the degree of
the polynomial, which is 7. 25. f ( x ) = 3x3 − 2 x 2 + x + 2
Examining f ( x ) = −4 x 7 + x3 − x 2 + 2 , there are The maximum number of zeros is the degree of
the polynomial, which is 3.
three variations in sign; thus, there are three
positive real zeros or there is one positive real Examining f ( x ) = 3x3 − 2 x 2 + x + 2 , there are
zero. two variations in sign; thus, there are two
Examining positive real zeros or no positive real zeros.
7 3 2
f ( − x ) = −4 ( − x ) + ( − x ) − ( − x ) + 2 , Examining
3 2
f (−x) = 3(−x) − 2 (−x) + ( −x) + 2 ,
= 4 x7 − x3 − x 2 + 2
there are two variations in sign; thus, there are = −3 x3 − 2 x 2 − x + 2
two negative real zeros or no negative real zeros. there is one variation in sign; thus, there is one
negative real zero.
22. f ( x ) = 5x4 + 2 x2 − 6 x − 5
The maximum number of zeros is the degree of 26. f ( x ) = − x3 − x 2 + x + 1
the polynomial, which, is 4. The maximum number of zeros is the degree of
Examining f ( x ) = 5 x 4 + 2 x 2 − 6 x − 5 , there is the polynomial, which is 3.
one variation in sign; thus, there is one positive Examining f ( x ) = − x3 − x 2 + x + 1 , there is one
real zero. variation in sign; thus, there is one positive real
Examining zero.
4 2
f (−x) = 5(−x) + 2 (−x) − 6 (−x) − 5 , Examining
3 2
f (−x) = − (−x) − (−x) + (−x) +1 ,
= 5x4 + 2 x2 + 6 x − 5
there is one variation in sign; thus, there is one = x3 − x 2 − x + 1
negative real zero. there are two variations in sign; thus, there are
two negative real zeros or no negative real zeros.

287
Copyright © 2015 Pearson Education, Inc.
Chapter 3: Polynomial and Rational Functions

27. f ( x ) = − x4 + x2 − 1 31. f ( x ) = x6 − 1
The maximum number of zeros is the degree of The maximum number of zeros is the degree of
the polynomial, which is 4. the polynomial, which is 6.
Examining f ( x ) = − x 4 + x 2 − 1 , there are two Examining f ( x ) = x6 − 1 , there is one variation
variations in sign; thus, there are two positive in sign; thus, there is one positive real zero.
real zeros or no positive real zeros. 6
Examining f ( − x ) = ( − x ) − 1 = x 6 − 1 , there is
4 2
Examining f ( − x ) = − ( − x ) + ( − x ) − 1 one variation in sign; thus, there is one negative
= − x 4 + x 2 − 1 , there are two variations in sign; real zero.
thus, there are two negative real zeros or no
32. f ( x ) = x6 + 1
negative real zeros.
The maximum number of zeros is the degree of
28. f ( x ) = x 4 + 5 x3 − 2 the polynomial, which is 6.
The maximum number of zeros is the degree of Examining f ( x ) = x6 + 1 , there is no variation
the polynomial, which is 4. in sign; thus, there are no positive real zeros.
Examining f ( x ) = x 4 + 5 x3 − 2 , there is one 6
Examining f ( − x ) = ( − x ) + 1 = x6 + 1 , there is
variation in sign; thus, there is one positive real no variation in sign; thus, there are no negative
zero. real zeros.
Examining
4 3
f ( − x ) = ( − x ) + 5 ( − x ) − 2 = x 4 − 5 x3 − 2 , 33. f ( x ) = 3x 4 − 3x3 + x 2 − x + 1
there is one variation in sign; thus, there is one p must be a factor of 1: p = ±1
negative real zero. q must be a factor of 3: q = ±1, ±3
29. f ( x ) = x5 + x 4 + x 2 + x + 1 p 1
The possible rational zeros are: = ±1, ±
q 3
The maximum number of zeros is the degree of
the polynomial, which is 5.
34. f ( x ) = x5 − x 4 + 2 x 2 + 3
Examining f ( x ) = x5 + x 4 + x 2 + x + 1 , there are
p must be a factor of 3: p = ±1, ±3
no variations in sign; thus, there are no positive
real zeros. q must be a factor of 1: q = ±1
Examining p
5 4 2 The possible rational zeros are: = ±1, ±3
f (−x) = (−x) + (−x) + (−x) + (−x) +1 , q
= − x5 + x 4 + x 2 − x + 1
35. f ( x ) = x5 − 6 x 2 + 9 x − 3
there are three variations in sign; thus, there are
three negative real zeros or there is one negative p must be a factor of –3: p = ±1, ±3
real zero. q must be a factor of 1: q = ±1
p
30. f ( x ) = x5 − x 4 + x3 − x 2 + x − 1 The possible rational zeros are: = ±1, ±3
q
The maximum number of zeros is the degree of
the polynomial, which is 5.
36. f ( x ) = 2 x5 − x 4 − x 2 + 1
Examining f ( x ) = x5 − x 4 + x3 − x 2 + x − 1 ,
p must be a factor of 1: p = ±1
there are five variations in sign; thus, there are
q must be a factor of 2: q = ±1, ±2
five positive real zeros or three positive real
zeros or there is one positive real zero. p 1
The possible rational zeros are: = ±1, ±
Examining q 2
5 4 3 2
f (−x) = ( −x) − ( −x) + ( −x) − ( −x) + ( −x) −1
37. f ( x ) = −4 x3 − x 2 + x + 2
= − x5 − x 4 − x3 − x 2 − x − 1
there is no variation in sign; thus, there are no p must be a factor of 2: p = ±1, ±2
negative real zeros. q must be a factor of –4: q = ±1, ±2, ±4

288
Copyright © 2015 Pearson Education, Inc.
Section 3.2: The Real Zeros of a Polynomial Function

The possible rational zeros are: The possible rational zeros are:
p 1 1 p 1 1 2 1 4 5
= ±1, ±2, ± , ± = ±1, ± 2, ± , ± , ± , ± , ±4, ± , ±5, ± ,
q 2 4 q 2 3 3 6 3 2
5 5 10 20
38. f ( x ) = 6 x4 − x2 + 2 ± , ± , ±10, ± , ±20, ±
3 6 3 3
p must be a factor of 2: p = ±1, ±2
q must be a factor of 6: q = ±1, ±2, ±3, ±6 44. f ( x) = − 6 x3 − x 2 + x + 10
The possible rational zeros are: p must be a factor of 10: p = ±1, ± 2, ±5, ±10
p 1 1 2 1 q must be a factor of –6: q = ±1, ±2, ±3, ±6
= ±1, ±2, ± , ± , ± , ±
q 2 3 3 6 The possible rational zeros are:
p 1 1 1 2 5
= ±1, ± , ± , ± , ±2, ± , ±5, ± ,
39. f ( x) = 6 x 4 − x 2 + 9 q 2 3 6 3 2
p must be a factor of 9: p = ±1, ± 3, ±9 5 5 10
± , ± , ±10, ±
q must be a factor of 6: q = ±1, ± 2, ±3, ±6 3 6 3
The possible rational zeros are:
p 1 1 1 3 9 45. f ( x ) = x3 + 2 x 2 − 5 x − 6
= ±1, ± , ± , ± , ±3, ± , ± 9, ±
q 2 3 6 2 2 Step 1: f (x) has at most 3 real zeros.

40. f ( x) = − 4 x3 + x 2 + x + 6 Step 2: By Descartes’ Rule of Signs, there is


one positive real zero.
p must be a factor of 6: p = ±1, ± 2, ±3, ±6 3 2
f (−x) = (−x) + 2 (−x) − 5(−x) − 6
q must be a factor of –4: q = ±1, ± 2, ± 4
The possible rational zeros are: = − x3 + 2 x 2 + 5 x − 6
p 1 1 3 3 thus, there are two negative real zeros or no
= ±1, ± 2, ± , ± , ±3, ± , ± , ±6 negative real zeros.
q 2 4 2 4
Step 3: Possible rational zeros:
41. f ( x) = 2 x5 − x3 + 2 x 2 + 12 p = ±1, ± 2, ± 3, ± 6; q = ±1;
p must be a factor of 12: p
= ±1, ± 2, ± 3, ± 6
p = ±1, ± 2, ± 3, ±4, ±6, ±12 q
q must be a factor of 2: q = ±1, ±2 Step 4: Using synthetic division:
The possible rational zeros are: We try x + 3 :
p 1 3
= ±1, ± 2, ± 4, ± , ±3, ± , ±6, ±12 −3)1 2 −5 −6
q 2 2
−3 3 6
42. f ( x) = 3 x5 − x 2 + 2 x + 18 1 −1 − 2 0
p must be a factor of 18: Since the remainder is 0, x − (−3) = x + 3 is a
p = ±1, ± 2, ±3, ±6, ±9, ±18 factor. The other factor is the quotient:
q must be a factor of 3: q = ±1, ±3 x2 − x − 2 .
The possible rational zeros are: (
Thus, f ( x ) = ( x + 3) x 2 − x − 2 )
p 1 2
= ±1, ± , ±2, ± , ±3, ±6, ±9 ± 18 = ( x + 3)( x + 1)( x − 2 )
q 3 3
The real zeros are –3, –1, and 2, each of
43. f ( x) = 6 x 4 + 2 x3 − x 2 + 20 multiplicity 1.
p must be a factor of 20:
46. f ( x ) = x3 + 8 x 2 + 11x − 20
p = ±1, ± 2, ±4, ±5, ±10, ±20
Step 1: f (x) has at most 3 real zeros.
q must be a factor of 6: q = ±1, ±2, ±3, ±6
Step 2: By Descartes’ Rule of Signs, there is
one positive real zero.

289
Copyright © 2015 Pearson Education, Inc.
Chapter 3: Polynomial and Rational Functions

3 2
f ( − x ) = ( − x ) + 8 ( − x ) + 11( − x ) − 20 , 1
We try x − :
2
= − x3 + 8 x 2 − 11x − 20
1
thus, there are two negative real zeros or no 2 −1 2 −1
negative real zeros. 2
1 0 1
Step 3: Possible rational zeros:
2 0 2 0
p = ±1, ±2, ±4, ±5, ±10, ±20
q = ±1 1
x− is a factor and the quotient is 2 x 2 + 2 .
2
p
= ±1, ±2, ±4, ±5, ±10, ±20 Thus,
q  1
Step 4: Using synthetic division: 
(
2
)
f ( x ) = 2 x3 − x 2 + 2 x − 1 =  x −  2 x 2 + 2
We try x + 5 : .
 1 2
−5)1 8 11 − 20 = 2 x −  x +1
 2
( )
− 5 − 15 20 Since x 2 + 1 = 0 has no real solutions, the only
1 3 −4 0 1
real zero is x = , of multiplicity 1.
Since the remainder is 0, x − (−5) = x + 5 is a 2
factor. The other factor is the quotient:
x 2 + 3x − 4 . 48. f ( x) = 2 x3 + x 2 + 2 x + 1
Thus, Step 1: f (x) has at most 3 real zeros.
(
f ( x ) = ( x + 5 ) x 2 + 3x − 4 ) Step 2: By Descartes’ Rule of Signs, there are
= ( x + 5 )( x + 4 )( x − 1) no positive real zeros.
3 2
The real zeros are –5, –4, and 1, each of f (−x) = 2(−x) + (−x) + 2(−x) +1
multiplicity 1.
= −2 x3 + x 2 − 2 x + 1
47. f ( x ) = 2 x3 − x 2 + 2 x − 1 thus, there are three negative real zeros or there
is one negative real zero.
Step 1: f (x) has at most 3 real zeros.
Step 3: Possible rational zeros:
Step 2: By Descartes’ Rule of Signs, there are
p = ±1; q = ±1, ± 2;
three positive real zeros or there is one positive
real zero. p 1
= ±1, ±
f (− x) = 2(− x)3 − (− x) 2 + 2(− x) − 1 q 2
= − 2 x3 − x 2 − 2 x − 1 Step 4: Using synthetic division:
thus, there are no negative real zeros. We try x + 1 :
Step 3: Possible rational zeros: −1 2 1 2 1
p = ±1 q = ±1, ± 2 −2 1 −3
p 1 2 −1 3 − 2
= ±1, ±
q 2 x + 1 is not a factor
Step 4: Using synthetic division: 1
We try x − 1 : We try x + :
2
1 2 −1 2 −1 1
2 1 3 − 2 1 2 1
2
−1 0 −1
2 1 3 2
x − 1 is not a factor 2 0 2 0
1
x+ is a factor and the quotient is 2 x 2 + 2
2

290
Copyright © 2015 Pearson Education, Inc.
Section 3.2: The Real Zeros of a Polynomial Function

 1 f (− x) = 3(− x)3 + 6(− x) 2 − 15 ( − x ) − 30 ,


(
f ( x) = 2 x3 + x 2 + 2 x + 1 =  x +  2 x 2 + 2
 2
)
= −3 x3 + 6 x 2 + 15 x − 30
 1
(
= 2  x +  x2 + 1
 2
) thus, there are two negative real zeros or no
negative real zeros.
Since x 2 + 1 = 0 has no real solutions, the only Step 3: Possible rational zeros:
1 p = ±1, ±2, ±5, ±10; q = ±1;
real zero is x = − , of multiplicity 1.
2 p
= ±1, ±2, ±5, ±10
q
49. f ( x ) = 2 x 3 − 4 x 2 − 10 x + 20
Step 4: Using synthetic division:
(
= 2 x3 − 2 x 2 − 5 x + 10 ) We try x + 2 :
Step 1: f (x) has at most 3 real zeros. −2 1 2 − 5 − 10
Step 2: By Descartes’ Rule of Signs, there are −2 0 10
two positive real zeros or no positive real zeros. 1 0 −5 0
f (− x) = 2(− x)3 − 4(− x) 2 − 10 ( − x ) + 20 , Since the remainder is 0, x + 2 is a factor. The
= −2 x3 − 4 x 2 + 10 x + 20 other factor is the quotient: x 2 − 5 .
thus, there is one negative real zeros. We can find the remaining real zeros by solving
x2 − 5 = 0
Step 3: Possible rational zeros:
p = ±1, ±2, ±5, ±10; q = ±1; x2 = 5
p x=± 5
= ±1, ±2, ±5, ±10
q
( )(
Thus, f ( x) = 3 ( x + 2 ) x − 5 x + 5 . The )
Step 4: Using synthetic division: real zeros are −2 , 5 , and − 5 , each of
We try x − 2 : multiplicity 1.
2 1 −2 −5 10
2 0 − 10 51. f ( x) = 2 x 4 + x3 − 7 x 2 − 3 x + 3
Step 1: f (x) has at most 4 real zeros.
1 0 −5 0
Since the remainder is 0, x − 2 is a factor. The Step 2: By Descartes’ Rule of Signs, there are
other factor is the quotient: x 2 − 5 . two positive real zeros or no positive real zeros.
4 3 2
We can find the remaining real zeros by solving f ( −x ) = 2 ( −x ) + ( − x) − 7 ( − x ) − 3( − x ) + 3
x2 − 5 = 0
= 2 x 4 − x3 − 7 x 2 + 3x + 3
2
x =5 thus, there are two negative real zeros or no
x=± 5 negative real zeros.

( )(
Thus, f ( x) = 2 ( x − 2 ) x − 5 x + 5 . The ) Step 3: Possible rational zeros:
p = ±1, ± 3; q = ±1, ±2;
real zeros are 2, 5 , and − 5 , each of
p 1 3
multiplicity 1. = ± , ±1, ± , ±3
q 2 2
50. f ( x ) = 3 x3 + 6 x 2 − 15 x − 30
Step 4: Using synthetic division:
(
= 3 x3 + 2 x 2 − 5 x − 10 ) We try x + 1 :
Step 1: f (x) has at most 3 real zeros. −1 2 1 − 7 − 3 3
−2 1 6 −3
Step 2: By Descartes’ Rule of Signs, there is
one positive real zero. 2 −1 − 6 3 0
x + 1 is a factor and the quotient is

291
Copyright © 2015 Pearson Education, Inc.
Chapter 3: Polynomial and Rational Functions

2 x3 − x 2 − 6 x + 3 . 2x + 1 = 0 x2 − 2 = 0
Factoring by grouping gives 2 x = −1 x2 = 2
2 x3 − x 2 − 6 x + 3 = x 2 ( 2 x − 1) − 3 ( 2 x − 1) 1
x=− x=± 2
(
= ( 2 x − 1) x 2 − 3 ) Thus,
2

Set each of these factors equal to 0 and solve:


2x −1 = 0 x2 − 3 = 0
(
f ( x) = ( 2 x + 1)( x − 1) x − 2 )( x + 2 )
 1
2x = 1 x2 = 3 = 2  x +  ( x − 1) x −
 2
( 2 )( x + 2 )
1 x=± 3
x= 1
2 The real zeros are − , 1, 2 , and − 2 , each
Thus, 2
(
f ( x) = ( 2 x − 1)( x + 1) x − 3 x + 3 )( ) of multiplicity 1

 1 f ( x) = x 4 + x3 − 3 x 2 − x + 2
= 2  x −  ( x + 1) x −
 2
( 3 )( x + 3 )
53.
Step 1: f (x) has at most 4 real zeros.
1
The real zeros are , −1 , 3 , and − 3 , each Step 2: By Descartes’ Rule of Signs, there are
2
two positive real zeros or no positive real zeros.
of multiplicity 1. 4 3 2
f ( − x ) = ( − x ) + ( − x ) − 3 ( − x ) − ( − x ) + 2 thus,
52. f ( x) = 2 x 4 − x3 − 5 x 2 + 2 x + 2 = x 4 − x3 − 3x 2 + x + 2
Step 1: f (x) has at most 4 real zeros. there are two negative real zeros or no negative
real zeros.
Step 2: By Descartes’ Rule of Signs, there are
two positive real zeros or no positive real zeros. Step 3: Possible rational zeros:
4 3
f (−x) = 2 (−x) − (−x) − 5(−x) + 2 (−x) + 2
2 p = ±1, ± 2; q = ±1;
p
= 2 x 4 − x3 − 5 x 2 − 2 x + 2 = ±1, ± 2
q
thus, there are two negative real zeros or no
negative real zeros.
Step 4: Using synthetic division:
Step 3: Possible rational zeros: We try x + 2 :
p = ±1, ± 2; q = ±1, ±2; − 2)1 1 − 3 −1 2
p 1 −2 2 2 −2
= ± , ±1, ± 2, ±3
q 2 1 −1 −1 1 0
x + 2 is a factor and the quotient is
Step 4: Using synthetic division: x3 − x 2 − x + 1 .
We try x − 1 :
1 2 −1 − 5 2 2 We try x + 1 on x3 − x 2 − x + 1
2 1 −4 −2 −1)1 − 1 − 1 1
−1 2 −1
2 1 −4 −2 0
1 −2 1 0
x − 1 is a factor and the quotient is
2 x3 + x 2 − 4 x − 2 . x + 1 is a factor and the quotient is x 2 − 2 x + 1 .
Factoring by grouping gives Thus,
2 x3 + x 2 − 4 x − 2 = x 2 ( 2 x + 1) − 2 ( 2 x + 1) (
f ( x) = ( x + 2 )( x + 1) x 2 − 2 x + 1 )
( )
2
= ( 2 x + 1) x 2 − 2 = ( x + 2 )( x + 1)( x − 1)
Set each of these factors equal to 0 and solve: The real zeros are –2, –1, each of multiplicity 1,
and 1, of multiplicity 2.

292
Copyright © 2015 Pearson Education, Inc.
Section 3.2: The Real Zeros of a Polynomial Function

54. f (x) = x − x − 6x + 4x + 8
4 3 2
Step 4: Using synthetic division:
Step 1: f (x) has at most 4 real zeros. We try x + 1 :
−1 4 5 9 10 2
Step 2: By Descartes’ Rule of Signs, there are − 4 −1 − 8 − 2
two positive real zeros or no positive real zeros.
4 3 2 4 1 8 2 0
f (−x) = (−x) − (−x) − 6(−x) + 4(−x) + 8
x + 1 is a factor and the quotient is
= x 4 + x3 − 6 x 2 − 4 x + 8 4 x3 + x 2 + 8 x + 2 .
thus, there are two negative real zeros or no Factoring by grouping gives
negative real zeros.
4 x3 + x 2 + 8 x + 2 = x 2 ( 4 x + 1) + 2 ( 4 x + 1)
Step 3: Possible rational zeros:
p = ±1, ± 2, ± 4, ± 8; q = ±1; (
= ( 4 x + 1) x 2 + 2 )
p Set each of these factors equal to 0 and solve:
= ±1, ± 2, ± 4, ± 8
q 4x + 1 = 0 x2 + 2 = 0
4 x = −1 x 2 = −2
Step 4: Using synthetic division: 1
We try x + 2 : x=− x = ± −2
4
− 2)1 − 1 − 6 4 8 no real sol.
−2 6 0 −8 Thus,
1 −3 0 4 0 (
f ( x) = ( 4 x + 1)( x + 1) x 2 + 2 )
3 2
x + 2 is a factor and the quotient is x − 3x + 4 .  1
(
= 4  x +  ( x + 1) x 2 + 2
 4
)
3 2
We try x + 1 on x − 3x + 4 1
−1)1 − 3 0 4 The real zeros are − and −1 , each of
4
−1 4 − 4 multiplicity 1.
1 −4 4 0
x + 1 is a factor and the quotient is x 2 − 4 x + 4 . 56. f ( x) = 3x 4 + 4 x3 + 7 x 2 + 8 x + 2
Thus, Step 1: f (x) has at most 4 real zeros.
(
f ( x ) = ( x + 2 )( x + 1) x 2 − 4 x + 4 ) Step 2: By Descartes’ Rule of Signs, there are
2
= ( x + 2 )( x + 1)( x − 2 ) no positive real zeros.
4 3 2
The real zeros are –2, –1, each of multiplicity 1, f (−x) = 3(−x) + 4 ( −x) + 7 ( −x) + 8(−x) + 2
and 2, of multiplicity 2. = 3x 4 − 4 x3 + 7 x 2 − 8 x + 2
thus, there are four negative real zeros or two
55. f ( x) = 4 x 4 + 5 x3 + 9 x 2 + 10 x + 2 negative real zeros or no negative real zeros.
Step 1: f (x) has at most 4 real zeros.
Step 3: Possible rational zeros:
Step 2: By Descartes’ Rule of Signs, there are p = ±1, ± 2; q = ±1, ±3;
no positive real zeros. p 1 2
4 3 2 = ± , ± , ±1, ± 2
f ( − x ) = 4 ( − x ) + 5 ( − x ) + 9 ( − x ) + 10 ( − x ) + 2 q 3 3
= 4 x 4 − 5 x3 + 9 x 2 − 10 x + 2
Step 4: Using synthetic division:
thus, there are four negative real zeros or two
We try x + 1 :
negative real zeros or no negative real zeros.
−1 3 4 7 8 2
Step 3: Possible rational zeros: − 3 −1 − 6 − 2
p = ±1, ± 2; q = ±1, ±2, ±4;
3 1 6 2 0
p 1 1
= ± , ± , ±1, ± 2 x + 1 is a factor and the quotient is
q 4 2

293
Copyright © 2015 Pearson Education, Inc.
Chapter 3: Polynomial and Rational Functions

3x3 + x 2 + 6 x + 2 . We try x − 2 on x3 − 2 x 2 + 4 x − 8
Factoring by grouping gives 2)1 − 2 4 −8
3x3 + x 2 + 6 x + 2 = x 2 ( 3x + 1) + 2 ( 3 x + 1) 2 0 8

(
= ( 3 x + 1) x 2 + 2 ) 1 0 4 0
x − 2 is a factor and the quotient is x 2 + 4 .
Set each of these factors equal to 0 and solve:
3x + 1 = 0 x2 + 2 = 0 ( )
Thus, f ( x ) = ( x + 1)( x − 2 ) x 2 + 4 .
3x = −1 x 2 = −2 Since x 2 + 4 = 0 has no real solutions, the
1 x = ± −2 solution set is {−1, 2} .
x=−
3
no real sol.
Thus, 58. 2 x3 + 3 x 2 + 2 x + 3 = 0
Solve by factoring:
(
f ( x) = ( 3 x + 1)( x + 1) x 2 + 2 ) x 2 (2 x + 3) + (2 x + 3) = 0
 1
(
= 3  x +  ( x + 1) x 2 + 2
 3
) (
(2 x + 3) x 2 + 1 = 0)
1 3
The real zeros are − and −1 , each of x=−
3 2
2
multiplicity 1. Since x + 1 = 0 has no real solutions, the
 3
solution set is −  .
57. x 4 − x3 + 2 x 2 − 4 x − 8 = 0  2
The solutions of the equation are the zeros
of f ( x ) = x 4 − x3 + 2 x 2 − 4 x − 8 . 59. 3x3 + 4 x 2 − 7 x + 2 = 0
The solutions of the equation are the zeros of
Step 1: f (x) has at most 4 real zeros. f ( x ) = 3 x3 + 4 x 2 − 7 x + 2 .

Step 2: By Descartes’ Rule of Signs, there are Step 1: f (x) has at most 3 real zeros.
three positive real zeros or there is one positive Step 2: By Descartes’ Rule of Signs, there are
real zero. two positive real zeros or no positive real zeros.
4 3 2
f (−x) = (−x) − (−x) + 2(−x) − 4(−x) − 8 3 2
f (−x) = 3(−x) + 4 ( −x) − 7 ( −x) + 2
= x 4 + x3 + 2 x 2 + 4 x − 8 = −3 x3 + 4 x 2 + 7 x + 2
thus, there is one negative real zero. thus, there is one negative real zero.
Step 3: Possible rational zeros: Step 3: Possible rational zeros:
p = ±1, ± 2, ± 4, ± 8; q = ±1; p = ±1, ± 2; q = ±1, ± 3
p p 1 2
= ±1, ± 2, ± 4, ± 8 = ±1, ± 2, ± , ±
q q 3 3
Step 4: Using synthetic division:
Step 4: Using synthetic division:
We try x + 1 : 2
We try x − :
−1)1 − 1 2 − 4 − 8 3
−1 2 − 4 8 2
3 4 −7 2
1 − 2 4 −8 0 3
2 4 −2
x + 1 is a factor and the quotient is
3 6 −3 0
x3 − 2 x 2 + 4 x − 8 .
2
x− is a factor. The other factor is the quotient
3
3x 2 + 6 x − 3 . Thus,

294
Copyright © 2015 Pearson Education, Inc.
Section 3.2: The Real Zeros of a Polynomial Function

 2 Since x 2 + x + 1 = 0 has no real solutions, the


 3
(
f ( x) =  x −  3x 2 + 6 x − 3 ) 5
solution set is   .
 2 2
(
= 3  x −  x2 + 2 x − 1
 3
)
Using the quadratic formula to solve 61. 3x3 − x 2 − 15 x + 5 = 0
x2 + 2 x − 1 = 0 : Solving by factoring:
x 2 (3 x − 1) − 5(3 x − 1) = 0
− 2 ± 4 − 4(1)(−1)
x=
2(1) ( )
(3x − 1) x 2 − 5 = 0

=
−2± 8 (
(3 x − 1) x − 5 )( x + 5 ) = 0
2
 1
−2± 2 2 The solution set is − 5, 5, .
= = −1 ± 2  3
2
 2
62. 2 x 3 − 11x 2 + 10 x + 8 = 0
The solution set is −1 − 2, − 1 + 2,  .
 3 The solutions of the equation are the zeros
of f ( x ) = 2 x 3 − 11x 2 + 10 x + 8 .
60. 2 x 3 − 3x 2 − 3x − 5 = 0
The solutions of the equation are the zeros Step 1: f (x) has at most 3 real zeros.
of f ( x ) = 2 x3 − 3 x 2 − 3 x − 5 . Step 2: By Descartes’ Rule of Signs, there are
two positive real zeros or no positive real zeros.
Step 1: f (x) has at most 3 real zeros. 3 2
f ( − x ) = 2 ( − x ) − 11( − x ) + 10 ( − x ) + 8
Step 2: By Descartes’ Rule of Signs, there is
one positive real zero. = −2 x3 − 11x 2 − 10 x + 8
3 2
f ( −x ) = 2 ( −x ) − 3( − x ) − 3( − x ) − 5 thus, there is one negative real zero.
Step 3: Possible rational zeros:
= −2 x3 − 3 x 2 + 3x − 5 p = ±1, ± 2, ± 4, ± 8; q = ±1, ± 2
thus, there are two negative real zeros or no
negative real zeros. p 1
= ±1, ± 2, ± 4, ± 8, ±
q 2
Step 3: Possible rational zeros:
p = ±1, ± 5; q = ±1, ± 2 Step 4: Using synthetic division:
p 1 5 We try x − 4 :
= ±1, ± 5, ± , ±
q 2 2 4) 2 − 11 10 8
8 − 12 − 8
Step 4: Using synthetic division:
2 −3 −2 0
5
We try x − : x − 4 is a factor. The other factor is the
2
5 quotient: 2 x 2 − 3 x − 2 . Thus,
2
2 −3 −3 −5
5 5 5
(
f ( x) = ( x − 4 ) 2 x 2 − 3x − 2 )
= ( x − 4 )( 2 x + 1)( x − 2 )
2 2 2 0
 1 
5 The solution set is − , 2, 4  .
x− is a factor. The other factor is the  2 
2
quotient: 2 x 2 + 2 x + 2 . Thus,
63. x 4 + 4 x3 + 2 x 2 − x + 6 = 0
 5
(
f ( x) =  x −  2 x 2 + 2 x + 2
 2
) The solutions of the equation are the zeros
of f ( x ) = x 4 + 4 x3 + 2 x 2 − x + 6 .
 5
 2
(
= 2  x −  x2 + x + 1 ) Step 1: f (x) has at most 4 real zeros.

295
Copyright © 2015 Pearson Education, Inc.
Chapter 3: Polynomial and Rational Functions

Step 2: By Descartes’ Rule of Signs, there are 1)1 − 2 10 − 18 9


two positive real zeros or no positive real zeros. 1 −1 9 −9
4 3 2
f (−x) = (−x) + 4 (−x) + 2 (−x) − (−x) + 6 1 −1 9 −9 0
= x 4 − 4 x3 + 2 x 2 + x + 6 x − 1 is a factor and the quotient is
thus, there are two negative real zeros or no x3 − x 2 + 9 x − 9 .
negative real zeros.
Step 3: Possible rational zeros: We try x − 1 on x3 − x 2 + 9 x − 9
p = ±1, ± 2, ± 3, ± 6; q = ±1; 1)1 −1 9 − 9
p 1 0 9
= ±1, ± 2, ± 3, ± 6 1 0 9 0
q
x − 1 is a factor and the quotient is x 2 + 9 . Thus,
Step 4: Using synthetic division:
We try x + 3 : f ( x) = ( x − 1)
2
(x 2
+9 .)
− 3)1 4 2 −1 6 2
Since x + 9 = 0 has no real solutions, the
−3 −3 3 −6 solution set is {1} .
1 1 −1 2 0
x + 3 is a factor and the quotient is 2 2 8
65. x3 − x + x + 1 = 0  3 x3 − 2 x 2 + 8 x + 3 = 0
x3 + x 2 − x + 2 . 3 3
The solutions of the equation are the zeros of
We try x + 2 on x3 + x 2 − x + 2
f ( x) = 3x3 − 2 x 2 + 8 x + 3 .
− 2)1 1 −1 2
−2 2 −2 Step 1: f (x) has at most 3 real zeros.
1 −1 1 0 Step 2: By Descartes’ Rule of Signs, there are
x + 2 is a factor and the quotient is x 2 − x + 1 . two positive real zeros or no positive real zeros.
(
Thus, f ( x) = ( x + 3)( x + 2 ) x 2 − x + 1 . ) f (− x) = 3(− x)3 − 2(− x ) 2 + 8(− x ) + 3 ,
2
Since x − x + 1 = 0 has no real solutions , the = −3x 3 − 2 x 2 − 8 x + 3
thus, there is one negative real zero.
solution set is {−3, −2} .
Step 3: To find the possible rational zeros:
p = ±1, ± 3; q = ±1, ± 3
64. x 4 − 2 x 3 + 10 x 2 − 18 x + 9 = 0
The solutions of the equation are the zeros p 1
= ±1, ± 3, ±
of f ( x ) = x 4 − 2 x3 + 10 x 2 − 18 x + 9 q 3
Step 4: Using synthetic division:
Step 1: f (x) has at most 4 real zeros.
1
We try x + :
Step 2: By Descartes’ Rule of Signs, there are 3
four positive real zeros or two positive real zeros 1
or no positive real zeros. − 3 −2 8 3
4 3 2 3
f ( − x ) = ( − x ) − 2 ( − x ) + 10 ( − x ) − 18 ( − x ) + 9 −1 1 − 3
= x 4 + 2 x3 + 10 x 2 + 18 x + 9 3 −3 9 0
Thus, there are no negative real zeros. 1
x+ is a factor. The other factor is the
Step 3: Possible rational zeros: 3
p = ±1, ± 3, ± 9; q = ±1 quotient: 3x 2 − 3x + 9 .
p Thus,
= ±1, ± 3, ± 9
q
Step 4: Using synthetic division:
We try x − 1 :

296
Copyright © 2015 Pearson Education, Inc.
Section 3.2: The Real Zeros of a Polynomial Function

 1 Since x 2 + 2 x + 4 = 0 has no real solutions, the


 3
(
f ( x) =  x +  3 x 2 − 3 x + 9 ) 1 
solution set is   .
 1 2
(
=  x +  ( 3) x 2 − x + 3
 3
)
67. 2 x 4 − 19 x3 + 57 x 2 − 64 x + 20 = 0
(
= ( 3 x + 1) x 2 − x + 3 ) The solutions of the equation are the zeros of
Since x 2 − x + 3 = 0 has no real solutions, the f ( x) = 2 x 4 − 19 x3 + 57 x 2 − 64 x + 20 .
 1
solution set is −  . Step 1: f (x) has at most 4 real zeros.
 3
Step 2: By Descartes’ Rule of Signs, there are
3 four positive real zeros or two positive real zeros
66. x3 + x 2 + 3 x − 2 = 0  2 x3 + 3x 2 + 6 x − 4 = 0 or no positive real zeros.
2
4 3 2
The solutions of the equation are the zeros of f (− x) = 2 ( − x ) − 19 ( − x ) + 57 ( − x ) − 64 ( − x ) + 20
f ( x) = 2 x3 + 3x 2 + 6 x − 4 . = 2 x 4 + 19 x3 + 57 x 2 + 64 x + 20
Step 1: f (x) has at most 3 real zeros. Thus, there are no negative real zeros.
Step 3: To find the possible rational zeros:
Step 2: By Descartes’ Rule of Signs, there is
p = ±1, ± 2, ±4, ±5, ±10, ±20; q = ±1, ± 2;
one positive real zero.
p 1 5
f (− x) = 2(− x)3 + 3(− x) 2 + 6(− x) − 4 = ±1, ± , ±2, ±4, ±5, ± , ±10, ±20
q 2 2
= −2 x3 + 3 x 2 − 6 x − 4
thus, there are two negative real zeros or no Step 4: Using synthetic division:
negative real zeros. We try x − 1 :
Step 3: To find the possible rational zeros: 1 2 − 19 57 − 64 20
p = ±1, ± 2, ±4; q = ±1, ± 2 2 − 17 40 − 24
p 1 2 − 17 40 − 24 − 4
= ±1, ± , ± 2, ±4
q 2 x − 1 is not a factor
Step 4: Using synthetic division: 1
We try x − 1 : We try x − :
2
1 2 3 6 −4 1
2 5 11 2 − 19 57 − 64 20
2
1 −9 24 − 20
2 5 11 7
x − 1 is not a factor 2 − 18 48 − 40 0
1
We try x − 12 x− is a factor and the quotient is
2
1
2
2 3 6 −4 2 x3 − 18 x 2 + 48 x − 40 . Thus,
1 2 4  1
2 4 8 0 
(
f ( x) =  x −  2 x3 − 18 x 2 + 48 x − 40
2
)
 1
x − 12 is a factor Thus,
(
= 2  x −  x3 − 9 x 2 + 24 x − 20
 2
)
 1
 2
(
f ( x) =  x −  2 x 2 + 4 x + 8 ) Now try x – 2 as a factor of x3 − 9 x 2 + 24 x − 20 .
 1 2 1 − 9 24 − 20
 2
(
= 2  x −  x2 + 2 x + 4 ) 2 − 14 20
1 − 7 10 0
x − 2 is a factor, and the other factor is the

297
Copyright © 2015 Pearson Education, Inc.
Chapter 3: Polynomial and Rational Functions

quotient x 2 − 7 x + 10 . Thus, f ( x) = ( x − 2 )( x + 4 )( 2 x + 1)( x − 2 )


2
(
x − 9 x + 24 x − 20 = ( x − 2 ) x − 7 x + 10
3 2
) = ( x − 2)
2
( x + 4 )( 2 x + 1)
= ( x − 2 )( x − 2 )( x − 5 )  1 
The solution set is −4, − , 2  .
 1 2  2 
f ( x) = 2  x −  ( x − 2 ) ( x − 5 )
 2 
1 
The solution set is  , 2,5 .
2 

68. 2 x 4 + x3 − 24 x 2 + 20 x + 16 = 0
The solutions of the equation are the zeros of
f ( x) = 2 x 4 + x3 − 24 x 2 + 20 x + 16 .

Step 1: f (x) has at most 4 real zeros.


Step 2: By Descartes’ Rule of Signs, there are
two positive real zeros or no positive real zeros.
4 3 2
f (− x) = 2 ( − x ) + ( − x ) − 24 ( − x ) + 20 ( − x ) + 16
= 2 x 4 − x3 − 24 x 2 − 20 x + 16
thus, there are two negative real zeros or no
negative real zeros.
Step 3: To find the possible rational zeros:
p = ±1, ± 2, ±4, ±8, ±16; q = ±1, ± 2;
p 1
= ±1, ± , ±2, ±4, ±8, ±16
q 2
Step 4: Using synthetic division:
We try x − 2 :
2 2 1 − 24 20 16
4 10 − 28 − 16
2 5 − 14 − 8 0
x − 2 is a factor, and the other factor is the
quotient 2 x 3 + 5 x 2 − 14 x − 8 .
(
Thus, f ( x) = ( x − 2 ) 2 x3 + 5 x 2 − 14 x − 8 . )
3 2
Now try x + 4 as a factor of 2 x + 5 x − 14 x − 8 .
−4 2 5 − 14 − 8
− 8 12 8
2 −3 −2 0
x + 4 is a factor, and the other factor is the
quotient 2 x 2 − 3 x − 2 . Thus,
(
2 x3 + 5 x 2 − 14 x − 8 = ( x + 4 ) 2 x 2 − 3 x − 2 )
= ( x + 4 )( 2 x + 1)( x − 2 )

298
Copyright © 2015 Pearson Education, Inc.
Section 3.2: The Real Zeros of a Polynomial Function

69. f ( x ) = x 4 − 3x 2 − 4

r coeff of q(x) remainder

1 1 1 −2 −2 −6
2 1 2 1 2 0
−1 1 −1 −2 2 −6
−2 1 −2 1 −2 0
For r = 2, the last row of synthetic division contains only numbers that are positive or 0, so we know there are no
zeros greater than 2. For r = -2, the last row of synthetic division results in alternating positive (or 0) and negative
(or 0) values, so we know that there are no zeros less than -2. The upper bound is 2 and the lower bound is -2.
70. f ( x ) = x 4 − 5 x 2 − 36
r coeff of q(x) remainder

1 1 1 −4 −4 −40
2 1 2 −1 −2 −40
3 1 3 4 12 0
−1 1 −1 −4 4 −40
−2 1 −2 −1 2 −40
−3 1 −3 4 −12 0
For r = 3, the last row of synthetic division contains only numbers that are positive or 0, so we know there are no
zeros greater than 3. For r = -3, the last row of synthetic division results in alternating positive (or 0) and negative
(or 0) values, so we know that there are no zeros less than -3. The upper bound is 3 and the lower bound is -3.
71. f ( x ) = x 4 + x3 − x − 1
r coeff of q(x) remainder

1 1 2 2 1 0
−1 1 0 0 −1 0
For r = 1, the last row of synthetic division contains only numbers that are positive or 0, so we know there are no
zeros greater than 1. For r = -1, the last row of synthetic division results in alternating positive (or 0) and negative
(or 0) values, so we know that there are no zeros less than -1. The upper bound is 1 and the lower bound is -1.
72. f ( x ) = x 4 − x3 + x − 1
r coeff of q(x) remainder

1 1 0 0 1 0
−1 1 −2 2 −1 0
For r = 1, the last row of synthetic division contains only numbers that are positive or 0, so we know there are no
zeros greater than 1. For r = -1, the last row of synthetic division results in alternating positive (or 0) and negative
(or 0) values, so we know that there are no zeros less than -1. The upper bound is 1 and the lower bound is -1.

299
Copyright © 2015 Pearson Education, Inc.
Chapter 3: Polynomial and Rational Functions

73. f ( x ) = 3 x 4 + 3 x3 − x 2 − 12 x − 12
r coeff of q(x) remainder

1 3 6 5 −7 5
2 3 9 17 22 56
−1 3 0 −1 −11 23
−2 3 −3 5 −22 56
For r = 2, the last row of synthetic division contains only numbers that are positive or 0, so we know there are no
zeros greater than 2. For r = -2, the last row of synthetic division results in alternating positive (or 0) and negative
(or 0) values, so we know that there are no zeros less than -2. The upper bound is 2 and the lower bound is -2.
74. f ( x ) = 3x 4 − 3x3 − 5 x 2 + 27 x − 36
r coeff of q(x) remainder

1 3 0 −5 22 −14
2 3 3 1 29 22
−1 3 −6 1 26 −62
−2 3 −9 13 1 −38
−3 3 −12 31 −66 162

For r = 2, the last row of synthetic division contains only numbers that are positive or 0, so we know there are no
zeros greater than 2. For r = -3, the last row of synthetic division results in alternating positive (or 0) and negative
(or 0) values, so we know that there are no zeros less than -3. The upper bound is 2 and the lower bound is -3.
75. f ( x ) = 4 x5 − x 4 + 2 x3 − 2 x 2 + x − 1
r coeff of q(x) remainder

1 4 3 5 3 4 3
−1 4 −5 7 −9 10 −11
For r = 1, the last row of synthetic division contains only numbers that are positive or 0, so we know there are no
zeros greater than 1. For r = -1, the last row of synthetic division results in alternating positive (or 0) and negative
(or 0) values, so we know that there are no zeros less than -1. The upper bound is 1 and the lower bound is -1.
 1 1 1 1 1
76. f ( x ) = 4 x 5 + x 4 + x3 + x 2 − 2 x − 2 = 4  x5 + x 4 + x3 + x 2 − x − 
 4 4 4 2 2
r coeff of q(x) remainder

1 4 5 6 7 5 3
−1 4 −3 4 −3 1 −3
For r = 1, the last row of synthetic division contains only numbers that are positive or 0, so we know there are no
zeros greater than 1. For r = -1, the last row of synthetic division results in alternating positive (or 0) and negative
(or 0) values, so we know that there are no zeros less than -1. The upper bound is 1 and the lower bound is -1.

300
Copyright © 2015 Pearson Education, Inc.
Section 3.2: The Real Zeros of a Polynomial Function

77. (
f ( x ) = − x 4 + 3 x3 − 4 x 2 − 2 x + 9 = − x 4 − 3x3 + 4 x 2 + 2 x − 9 )
r coeff of q(x) remainder

1 1 −2 2 4 5
2 1 −1 2 6 3
3 1 0 4 14 33
−1 1 −4 8 −6 −3
−2 1 −5 14 −26 43

For r = 3, the last row of synthetic division contains only numbers that are positive or 0, so we know there are no
zeros greater than 3. For r = -2, the last row of synthetic division results in alternating positive (or 0) and negative
(or 0) values, so we know that there are no zeros less than -2. The upper bound is 3 and the lower bound is -2.
78. (
f ( x ) = −4 x5 + 5 x3 + 9 x 2 + 3 x − 12 = − 4 x5 − 5 x3 − 9 x 2 − 3 x + 12 )
r coeff of q(x) remainder

1 4 4 −1 −10 −13 −1
2 4 8 11 13 23 58
−1 4 −4 −1 −8 5 7
−2 4 −8 11 −31 59 −106
For r = 2, the last row of synthetic division contains only numbers that are positive or 0, so we know there are no
zeros greater than 2. For r = -2, the last row of synthetic division results in alternating positive (or 0) and negative
(or 0) values, so we know that there are no zeros less than -2. The upper bound is 2 and the lower bound is -2.

79. f ( x ) = 8 x 4 − 2 x 2 + 5 x − 1; [0,1] 82. f ( x) = 3x3 − 10 x + 9; [ −3, − 2]


f (0) = −1 < 0 and f (1) = 10 > 0 f (−3) = − 42 < 0 and f (− 2) = 5 > 0
The value of the function is positive at one The value of the function is positive at one
endpoint and negative at the other. Since the endpoint and negative at the other. Since the
function is continuous, the Intermediate Value function is continuous, the Intermediate Value
Theorem guarantees at least one zero in the Theorem guarantees at least one zero in the
given interval. given interval.

80. f ( x) = x 4 + 8 x3 − x 2 + 2; [ −1, 0] 83. f ( x) = x5 − x 4 + 7 x3 − 7 x 2 − 18 x + 18; [1.4, 1.5]


f (−1) = − 6 < 0 and f (0) = 2 > 0 f (1.4) = − 0.1754 < 0 and f (1.5) = 1.4063 > 0
The value of the function is positive at one The value of the function is positive at one
endpoint and negative at the other. Since the endpoint and negative at the other. Since the
function is continuous, the Intermediate Value function is continuous, the Intermediate Value
Theorem guarantees at least one zero in the Theorem guarantees at least one zero in the
given interval. given interval.

81. f ( x) = 2 x3 + 6 x 2 − 8 x + 2; [ −5, − 4] 84. f ( x) = x5 − 3 x 4 − 2 x3 + 6 x 2 + x + 2; [1.7, 1.8]


f (−5) = −58 < 0 and f (− 4) = 2 > 0 f (1.7) = 0.35627 > 0 and f (1.8) = –1.021 < 0
The value of the function is positive at one The value of the function is positive at one
endpoint and negative at the other. Since the endpoint and negative at the other. Since the
function is continuous, the Intermediate Value function is continuous, the Intermediate Value
Theorem guarantees at least one zero in the Theorem guarantees at least one zero in the
given interval. given interval.

301
Copyright © 2015 Pearson Education, Inc.
Chapter 3: Polynomial and Rational Functions

85. 8 x 4 − 2 x 2 + 5 x − 1 = 0; 0 ≤ r ≤1 f ( −0.65 ) ≈ −0.4410; f ( −0.64 ) ≈ −0.3390


Consider the function f ( x ) = 8 x − 2 x + 5 x − 1
4 2
f ( −0.64 ) ≈ −0.3390; f ( −0.63) ≈ −0.2397
Subdivide the interval [0,1] into 10 equal f ( −0.63) ≈ −0.2397; f ( −0.62 ) ≈ −0.1433
subintervals:
[0,0.1]; [0.1,0.2]; [0.2,0.3]; [0.3,0.4]; [0.4,0.5]; f ( −0.62 ) ≈ −0.1433; f ( −0.61) ≈ −0.0495
[0.5,0.6]; [0.6,0.7]; [0.7,0.8]; [0.8,0.9]; [0.9,1] f ( −0.61) ≈ −0.0495; f ( −0.60 ) ≈ 0.0416
f ( 0 ) = −1; f ( 0.1) = −0.5192 So f has a real zero on the interval
f ( 0.1) = −0.5192; f ( 0.2 ) = −0.0672 [–0.61, –0.6], therefore r = −0.60 , correct to two
f ( 0.2 ) = −0.0672; f ( 0.3) = 0.3848 decimal places.
So f has a real zero on the interval [0.2,0.3]. 87. 2 x3 + 6 x 2 − 8 x + 2 = 0; − 5 ≤ r ≤ −4
Subdivide the interval [0.2,0.3] into 10 equal Consider the function f ( x ) = 2 x3 + 6 x 2 − 8 x + 2
subintervals: Subdivide the interval [–5, –4] into 10 equal
[0.2,0.21]; [0.21,0.22]; [0.22,0.23]; [0.23,0.24]; subintervals:
[0.24,0.25]; [0.25,0.26];[0.26,0.27]; [0.27,0.28]; [–5, –4.9]; [–4.9, –4.8]; [–4.8, –4.7]; [–4.7, –4.6];
[0.28,0.29]; [0.29,0.3] [–4.6, –4.5]; [–4.5, –4.4]; [–4.4, –4.3];
f ( 0.2 ) = −0.0672; f ( 0.21) ≈ −0.02264 [–4.3, –4.2]; [–4.2, –4.1]; [–4.1, –4]
f ( −5 ) = −58; f ( −4.9 ) = −50.038
f ( 0.21) ≈ −0.02264; f ( 0.22 ) ≈ 0.0219
So f has a real zero on the interval [0.21,0.22], f ( −4.9 ) = −50.038; f ( −4.8 ) = −42.544
therefore r = 0.21 , correct to two decimal f ( −4.8 ) = −42.544; f ( −4.7 ) = −35.506
places. f ( −4.7 ) = −35.506; f ( −4.6 ) = −28.912
86. x 4 + 8 x3 − x 2 + 2 = 0; −1 ≤ r ≤ 0 f ( −4.6 ) = −28.912; f ( −4.5 ) = −22.75
Consider the function f ( x ) = x + 8 x − x + 2
4 3 2 f ( −4.5 ) = −22.75; f ( −4.4 ) = −17.008
Subdivide the interval [–1, 0] into 10 equal f ( −4.4 ) = −17.008; f ( −4.3) = −11.674
subintervals:
f ( −4.3) = −11.674; f ( −4.2 ) = −6.736
[–1, –0.9]; [–0.9, –0.8]; [–0.8, –0.7]; [–0.7, –0.6];
[–0.6, –0.5]; [–0.5, –0.4]; [–0.4, –0.3]; f ( −4.2 ) = −6.736; f ( −4.1) = −2.182
[–0.3, –0.2]; [–0.2, –0.1]; [–0.1,0]
f ( −4.1) = −2.182; f ( −4 ) = 2
f ( −1) = −6; f ( −0.9 ) = −3.9859
So f has a real zero on the interval [–4.1, –4].
f ( −0.9 ) = −3.9859; f ( −0.8 ) = −2.3264
f ( −0.8 ) = −2.3264; f ( −0.7 ) = −0.9939 Subdivide the interval [–4.1, –4] into 10 equal
subintervals:
f ( −0.7 ) = −0.9939; f ( −0.6 ) = 0.0416 [–4.1, –4.09]; [–4.09, –4.08]; [–4.08, –4.07];
So f has a real zero on the interval [–0.7, –0.6]. [–4.07, –4.06]; [–4.06, –4.05]; [–4.05, –4.04];
[–4.04, –4.03]; [–4.03, –4.02]; [–4.02, –4.01];
Subdivide the interval [–0.7, –0.6] into 10 equal [–4.01, –4]
subintervals: f ( −4.1) = −2.182; f ( −4.09 ) ≈ −1.7473
[–0.7, –0.69]; [–0.69, –0.68]; [–0.68, –0.67];
f ( −4.09 ) ≈ −1.7473; f ( −4.08 ) ≈ −1.3162
[–0.67, –0.66]; [–0.66, –0.65]; [–0.65, –0.64];
[–0.64, –0.63]; [–0.63, –0.62]; [–0.62, –0.61]; f ( −4.08 ) ≈ −1.3162; f ( −4.07 ) ≈ −0.8889
[–0.61, –0.6]
f ( −4.07 ) ≈ −0.8889; f ( −4.06 ) ≈ −0.4652
f ( −0.7 ) = −0.9939; f ( −0.69 ) ≈ −0.8775
f ( −4.06 ) ≈ −0.4652; f ( −4.05 ) ≈ −0.0453
f ( −0.69 ) ≈ −0.8775; f ( −0.68 ) ≈ −0.7640
f ( −4.05 ) ≈ −0.4653; f ( −4.04 ) ≈ 0.3711
f ( −0.68 ) ≈ −0.7640; f ( −0.67 ) ≈ −0.6535
So f has a real zero on the interval
f ( −0.67 ) ≈ −0.6535; f ( −0.66 ) ≈ −0.5458
[–4.05, –4.04], therefore r = −4.04 , correct to
f ( −0.66 ) ≈ −0.5458; f ( −0.65 ) ≈ −0.4410 two decimal places.

302
Copyright © 2015 Pearson Education, Inc.
Section 3.2: The Real Zeros of a Polynomial Function

88. 3x3 − 10 x + 9 = 0; − 3 ≤ r ≤ −2 [1.14,1.15]; [1.15,1.16];[1.16,1.17]; [1.17,1.18];


[1.18,1.19]; [1.19,1.2]
Consider the function f ( x ) = 3 x3 − 10 x + 9
f (1.1) = −0.359; f (1.11) ≈ −0.2903
Subdivide the interval [–3, –2] into 10 equal
subintervals: f (1.11) ≈ −0.2903; f (1.12 ) ≈ −0.2207
[–3, –2.9]; [–2.9, –2.8]; [–2.8, –2.7]; [–2.7, –2.6]; f (1.12 ) ≈ −0.2207; f (1.13) ≈ −0.1502
[–2.6, –2.5]; [–2.5, –2.4]; [–2.4, –2.3];
[–2.3, –2.2]; [–2.2, –2.1]; [–2.1, –2] f (1.13) ≈ −0.1502; f (1.14 ) ≈ −0.0789
f ( −3) = −42; f ( −2.9 ) = −35.167 f (1.14 ) ≈ −0.0789; f (1.15 ) ≈ −0.0066
f ( −2.9 ) = −35.167; f ( −2.8 ) = −28.856 f (1.15 ) ≈ −0.0066; f (1.16 ) ≈ 0.0665
f ( −2.8 ) = −28.856; f ( −2.7 ) = −23.049 So f has a real zero on the interval [1.15,1.16],
f ( −2.7 ) = −23.049; f ( −2.6 ) = −17.728 therefore r = 1.15 , correct to two decimal
places.
f ( −2.6 ) = −17.728; f ( −2.5 ) = −12.875
f ( −2.5 ) = −12.875; f ( −2.4 ) = −8.472 90. f ( x ) = 2 x4 + x2 − 1

f ( −2.4 ) = −8.472; f ( −2.3) = −4.501 f ( 0 ) = −1; f (1) = 2

f ( −2.3) = −4.501; f ( −2.2 ) = −0.944 So f has a real zero on the interval [0,1].
Subdivide the interval [0,1] into 10 equal
f ( −2.2 ) = −0.944; f ( −2.1) = 2.217 subintervals:
So f has a real zero on the interval [–2.2, –2.1]. [0,0.1]; [0.1,0.2]; [0.2,0.3]; [0.3,0.4]; [0.4,0.5];
[0.5,0.6]; [0.6,0.7]; [0.7,0.8]; [0.8,0.9]; [0.9,1]
Subdivide the interval [–2.2, –2.1] into 10 equal f ( 0 ) = −1; f ( 0.1) = −0.9898
subintervals:
[–2.2, –2.19]; [–2.19, –2.18]; [–2.18, –2.17]; f ( 0.1) = −0.9898; f ( 0.2 ) = −0.9568
[–2.17, –2.16]; [–2.16, –2.15]; [–2.15, –2.14]; f ( 0.2 ) = −0.9568; f ( 0.3) = −0.8938
[–2.14, –2.13]; [–2.13, –2.12]; [–2.12, –2.11];
[–2.11, –2.1] f ( 0.3) = −0.8938; f ( 0.4 ) = −0.7888
f ( −2.2 ) = −0.944; f ( −2.19 ) ≈ −0.6104 f ( 0.4 ) = −0.7888; f ( 0.5 ) = −0.625
f ( −2.19 ) ≈ −0.6104; f ( −2.18 ) ≈ −0.2807 f ( 0.5 ) = −0.625; f ( 0.6 ) = −0.3808
f ( −2.18 ) ≈ −0.2807; f ( −2.17 ) ≈ 0.0451 f ( 0.6 ) = −0.3808; f ( 0.7 ) = −0.0298
So f has a real zero on the interval f ( 0.7 ) = −0.0298; f ( 0.8 ) = 0.4592
[–2.18, –2.17], therefore r = −2.17 , correct to So f has a real zero on the interval [0.7,0.8].
two decimal places.
Subdivide the interval [0.7,0.8] into 10 equal
89. f ( x ) = x3 + x 2 + x − 4 subintervals:
f (1) = −1; f ( 2 ) = 10 [0.7,0.71]; [0.71,0.72]; [0.72,0.73]; [0.73,0.74];
[0.74,0.75]; [0.75,0.76];[0.76,0.77]; [0.77,0.78];
So f has a real zero on the interval [1,2]. [0.78,0.79]; [0.79,0.8]
Subdivide the interval [1,2] into 10 equal f ( 0.7 ) = −0.298; f ( 0.71) ≈ 0.0123
subintervals:
So f has a real zero on the interval [0.7,0.71],
[1,1.1]; [1.1,1.2]; [1.2,1.3]; [1.3,1.4]; [1.4,1.5];
[1.5,1.6]; [1.6,1.7]; [1.7,1.8]; [1.8,1.9]; [1.9,2] therefore r = 0.70 , correct to two decimal
f (1) = −1; f (1.1) = −0.359 places.

f (1.1) = −0.359; f (1.2 ) = 0.368 91. f ( x ) = 2 x 4 − 3 x3 − 4 x 2 − 8


So f has a real zero on the interval [1.1,1.2]. f ( 2 ) = −16; f ( 3) = 37
Subdivide the interval [1.1,1.2] into 10 equal So f has a real zero on the interval [2,3].
subintervals: Subdivide the interval [2,3] into 10 equal
[1.1,1.11]; [1.11,1.12]; [1.12,1.13]; [1.13,1.14]; subintervals:

303
Copyright © 2015 Pearson Education, Inc.
Chapter 3: Polynomial and Rational Functions

[2,2.1]; [2.1,2.2]; [2.2,2.3]; [2.3,2.4]; [2.4,2.5]; subintervals:


[2.5,2.6]; [2.6,2.7]; [2.7,2.8]; [2.8,2.9]; [2.9,3] [2,2.1]; [2.1,2.2]; [2.2,2.3]; [2.3,2.4]; [2.4,2.5];
f ( 2 ) = −16; f ( 2.1) = −14.5268 [2.5,2.6]; [2.6,2.7]; [2.7,2.8]; [2.8,2.9]; [2.9,3]
f ( 2 ) = −4; f ( 2.1) = −1.037
f ( 2.1) = −14.5268; f ( 2.2 ) = −12.4528
f ( 2.1) = −1.037; f ( 2.2 ) = 2.264
f ( 2.2 ) = −12.4528; f ( 2.3) = −9.6928
So f has a real zero on the interval [2.1,2.2].
f ( 2.3) = −9.6928; f ( 2.4 ) = −6.1568
Subdivide the interval [2.1,2.2] into 10 equal
f ( 2.4 ) = −6.1568; f ( 2.5 ) = −1.75
subintervals:
f ( 2.5 ) = −1.75; f ( 2.6 ) = 3.6272 [2.1,2.11]; [2.11,2.12]; [2.12,2.13]; [2.13,2.14];
So f has a real zero on the interval [2.5,2.6]. [2.14,2.15]; [2.15,2.16];[2.16,2.17]; [2.17,2.18];
[2.18,2.19]; [2.19,2.2]
Subdivide the interval [2.5,2.6] into 10 equal f ( 2.1) = −1.037; f ( 2.11) ≈ −0.7224
subintervals:
[2.5,2.51]; [2.51,2.52]; [2.52,2.53]; [2.53,2.54]; f ( 2.11) ≈ −0.7224; f ( 2.12 ) ≈ −0.4044
[2.54,2.55]; [2.55,2.56];[2.56,2.57]; [2.57,2.58]; f ( 2.12 ) ≈ −0.4044; f ( 2.13) ≈ −0.0830
[2.58,2.59]; [2.59,2.6]
f ( 2.13) ≈ −0.0830; f ( 2.14 ) ≈ 0.2418
f ( 2.5 ) = −1.75; f ( 2.51) ≈ −1.2576
So f has a real zero on the interval [2.13,2.14],
f ( 2.51) ≈ −1.2576; f ( 2.52 ) ≈ −0.7555
therefore r = 2.13 , correct to two decimal
f ( 2.52 ) ≈ −0.7555; f ( 2.53) ≈ −0.2434 places.
f ( 2.53) ≈ −0.2434; f ( 2.54 ) ≈ 0.2787
So f has a real zero on the interval [2.53,2.54],
therefore r = 2.53 , correct to two decimal
places.

92. f ( x ) = 3x3 − 2 x 2 − 20
f ( 2 ) = −4; f ( 3) = 43
So f has a real zero on the interval [2,3].
Subdivide the interval [2,3] into 10 equal

93. f ( x) = x3 + 2 x 2 − 5 x − 6 = ( x + 3)( x + 1)( x − 2)


x-intercepts: –3, –1, 2;
Near −3 : f ( x ) ≈ ( x + 3)( −3 + 1)( −3 − 2 ) = 10 ( x + 3)
Near −1 : f ( x ) ≈ ( −1 + 3)( x + 1)( −1 − 2 ) = −6 ( x + 1)
Near 2: f ( x ) ≈ ( 2 + 3)( 2 + 1)( x − 2 ) = 15 ( x − 2 )
Plot the point ( −3, 0 ) and show a line with positive slope there.
Plot the point ( −1, 0 ) and show a line with negative slope there.
Plot the point ( 2, 0 ) and show a line with positive slope there.

2
y-intercept: f ( 0 ) = 03 + 2 ( 0 ) − 5 ( 0 ) − 6 = −6 ;
The graph of f crosses the x-axis at x = –3, –1 and 2 since each zero has multiplicity 1.

304
Copyright © 2015 Pearson Education, Inc.
Section 3.2: The Real Zeros of a Polynomial Function

Interval ( −∞, −3) ( −3, −1) ( −1, 2 ) ( 2, ∞ )


Number Chosen –4 –2 0 3
Value of f –18 4 –6 24
Location of Graph Below x-axis Above x-axis Below x-axis Above x-axis
Point on Graph ( −4, −18 ) ( −2, 4 ) ( 0, −6 ) ( 3, 24 )

94. f ( x) = x3 + 8 x 2 + 11x − 20 = ( x + 5)( x + 4)( x − 1)


x-intercepts: –5, –4, 1;
Near −5 : f ( x ) ≈ ( x + 5 )( −5 + 4 )( −5 − 1) = 6 ( x + 5 )
Near −4 : f ( x ) ≈ ( −4 + 5 )( x + 4 )( −4 − 1) = −5 ( x + 4 )
Near 1: f ( x ) ≈ (1 + 5 )(1 + 4 )( x − 1) = 30 ( x − 1)
Plot the point ( −5, 0 ) and show a line with positive slope there.
Plot the point ( −4, 0 ) and show a line with negative slope there.
Plot the point (1, 0 ) and show a line with positive slope there.

2
y-intercept: f ( 0 ) = 03 + 8 ( 0 ) + 11( 0 ) − 20 = −20
The graph of f crosses the x-axis at x = –5, –4 and 1 since each zero has multiplicity 1.

Interval ( −∞, −5) ( −5, −4 ) ( −4,1) (1, ∞ )


Number Chosen –6 –4.5 0 2
Value of f –14 1.375 –20 42
Location of Graph Below x-axis Above x-axis Below x-axis Above x-axis
Point on Graph ( −6, −14 ) ( −4.5,1.375) ( 0, −20 ) ( 2, 42 )

305
Copyright © 2015 Pearson Education, Inc.
Chapter 3: Polynomial and Rational Functions

 1
95.
 2
(
f ( x ) = 2 x3 − x 2 + 2 x − 1 =  x −  2 x 2 + 2 )
1   1   5
2
1 1  1
x-intercept: ; Near : f ( x) ≈  x −   2  + 2  =  x − 
2 2  2    2   2
 2
1 
Plot the point  , 0  and show a line with positive slope there.
2 
3
y-intercept: f ( 0 ) = 2 ( 0 ) − 02 + 2 ( 0 ) − 1 = −1
1
The graph of f crosses the x-axis at x = since the zero has multiplicity 1.
2
 1 1 
Interval  −∞,   ,∞
 2 2 
Number Chosen 0 1
Value of f –1 2
Location of Graph Below x-axis Above x-axis
Point on Graph ( 0, −1) (1, 2 )

 1
96.
 2
(
f ( x) = 2 x3 + x 2 + 2 x + 1 =  x +  2 x 2 + 2)
1
x-intercept: −
2
1   1   5
2
1  1
Near − : f ( x ) ≈  x +   2  −  + 2  =  x + 
2  2 
  2   2  2

 1 
Plot the point  − , 0  and show a line with positive slope there.
 2 

306
Copyright © 2015 Pearson Education, Inc.
Section 3.2: The Real Zeros of a Polynomial Function

3
y-intercept: f ( 0 ) = 2 ( 0 ) + 02 + 2 ( 0 ) + 1 = 1

1
The graph of f crosses the x-axis at x = − since the zero has multiplicity 1.
2

 1  1 
Interval  −∞, −  − ,∞
 2  2 
Number Chosen –1 0
Value of f –2 1
Location of Graph Below x-axis Above x-axis
Point on Graph ( −1, −2 ) ( 0,1)

97. (
f ( x) = x 4 + x 2 − 2 = ( x + 1)( x − 1) x 2 + 2 )
x-intercepts: –1, 1
( 2
Near −1 : f ( x ) ≈ ( x + 1)( −1 − 1) ( −1) + 2 = −6 ( x + 1) )
( )
Near 1: f ( x ) ≈ (1 + 1)( x − 1) 12 + 2 = 6 ( x − 1)
Plot the point ( −1, 0 ) and show a line with negative slope there.
Plot the point (1, 0 ) and show a line with positive slope there.

y-intercept: f ( 0 ) = 04 + 02 − 2 = −2
The graph of f crosses the x-axis at x = –1 and 1 since each zero has multiplicity 1.

Interval ( −∞, −1) ( −1,1) (1, ∞ )


Number Chosen –2 0 2
Value of f 18 –2 18
Location of Graph Above x-axis Below x-axis Above x-axis
Point on Graph ( −2,18 ) ( 0, −2 ) ( 2,18 )

307
Copyright © 2015 Pearson Education, Inc.
Chapter 3: Polynomial and Rational Functions

98. (
f ( x ) = x 4 − 3 x 2 − 4 = ( x + 2 )( x − 2 ) x 2 + 1 )
x-intercepts: –2, 2
( 2
Near −2 : f ( x ) ≈ ( x + 2 )( −2 − 2 ) ( −2 ) + 1 = −20 ( x + 2 ) )
(
Near 2: f ( x ) ≈ ( 2 + 2 )( x − 2 ) 2 + 1 = 20 ( x − 2 )
2
)
Plot the point ( −2, 0 ) and show a line with negative slope there.
Plot the point ( 2, 0 ) and show a line with positive slope there.

2
y-intercept: f ( 0 ) = 04 − 3 ( 0 ) − 4 = −4
The graph of f crosses the x-axis at x = –2 and 2 since each zero has multiplicity 1.

Interval ( −∞, −2 ) ( −2, 2 ) ( 2, ∞ )


Number Chosen –3 0 3
Value of f 50 –4 50
Location of Graph Above x-axis Below x-axis Above x-axis
Point on Graph ( −3, 50 ) ( 0, −4 ) ( 3,50 )

99. f ( x ) = 4 x 4 + 7 x 2 − 2 = ( 2 x + 1)( 2 x − 1) x 2 + 2 ( )
1 1
x-intercepts: − ,
2 2
  1   1  
2
1 9
Near − : f ( x ) ≈ ( 2 x + 1)  2  −  − 1   −  + 2  = − ( 2 x + 1)
2   2   2   2

1  1    1 2  9
Near : f ( x ) ≈  2   + 1 ( 2 x − 1)    + 2  = ( 2 x − 1)
2  2   2   2
 

308
Copyright © 2015 Pearson Education, Inc.
Section 3.2: The Real Zeros of a Polynomial Function

 1 
Plot the point  − , 0  and show a line with negative slope there.
 2 
1 
Plot the point  , 0  and show a line with positive slope there.
2 

4 2
y-intercept: f ( 0 ) = 4 ( 0 ) + 7 ( 0 ) − 2 = −2
1 1
The graph of f crosses the x-axis at x = − and since each zero has multiplicity 1.
2 2

 1  1 1 1 
Interval  −∞, −  − ,   ,∞
 2  2 2 2 
Number Chosen –1 0 1
Value of f 9 –2 9
Location of Graph Above x-axis Below x-axis Above x-axis
Point on Graph ( −1,9 ) ( 0, −2 ) (1,9 )

100. (
f ( x ) = 4 x 4 + 15 x 2 − 4 = ( 2 x + 1)( 2 x − 1) x 2 + 4 )
1 1
x-intercepts: − ,
2 2
  1   1  
2
1 17
Near − : f ( x ) ≈ ( 2 x + 1)  2  −  − 1   −  + 4  = − ( 2 x + 1)
2   2    2   2

1  1    1 2  17
Near : f ( x ) ≈  2   + 1 ( 2 x − 1)    + 4  = ( 2 x − 1)
2  2   2   2
 
 1 
Plot the point  − , 0  and show a line with negative slope there.
 2 
1 
Plot the point  , 0  and show a line with positive slope there.
2 

4 2
y-intercept: f ( 0 ) = 4 ( 0 ) + 15 ( 0 ) − 4 = −4
1 1
The graph of f crosses the x-axis at x = − and since each zero has multiplicity 1.
2 2

309
Copyright © 2015 Pearson Education, Inc.
Chapter 3: Polynomial and Rational Functions

 1  1 1 1 
Interval  −∞, −  − ,   ,∞
 2  2 2 2 
Number Chosen –1 0 1
Value of f 15 –4 15
Location of Graph Above x-axis Below x-axis Above x-axis
Point on Graph ( −1,15) ( 0, −4 ) (1,15 )

101. f ( x) = x 4 + x 3 − 3x 2 − x + 2 = ( x + 2)( x + 1)( x − 1) 2


x-intercepts: –2, –1, 1
2
Near −2 : f ( x ) ≈ ( x + 2 )( −2 + 1)( −2 − 1) = −9 ( x + 2 )
2
Near −1 : f ( x ) ≈ ( −1 + 2 )( x + 1)( −1 − 1) = 4 ( x + 1)
2 2
Near 1: f ( x ) ≈ (1 + 2 )(1 + 1)( x − 1) = 6 ( x − 1)
Plot the point ( −2, 0 ) and show a line with negative slope there.
Plot the point ( −1, 0 ) and show a line with positive slope there.
Plot the point (1, 0 ) and show a parabola opening up there.

2
y-intercept: f ( 0 ) = 04 + 03 − 3 ( 0 ) − 0 + 2 = 2
The graph of f crosses the x-axis at x = –2 and –1 since each zero has multiplicity 1.
The graph of f touches the x-axis at x = 1 since the zero has multiplicity 2.

Interval ( −∞, −2 ) ( −2, −1) ( −1,1) (1, ∞ )


Number Chosen –3 –1.5 0 2
Value of f 32 –1.5625 2 12
Location of Graph Above x-axis Below x-axis Above x-axis Above x-axis
Point on Graph ( −3,32 ) ( −1.5, −1.5625 ) ( 0, 2 ) ( 2,12 )

310
Copyright © 2015 Pearson Education, Inc.
Section 3.2: The Real Zeros of a Polynomial Function

102. f ( x) = x 4 − x 3 − 6 x 2 + 4 x + 8 = ( x + 2)( x + 1)( x − 2) 2


x-intercepts: –2, –1, 2
2
Near −2 : f ( x ) ≈ ( x + 2 )( −2 + 1)( −2 − 2 ) = −16 ( x + 2 )
2
Near −1 : f ( x ) ≈ ( −1 + 2 )( x + 1)( −1 − 2 ) = 9 ( x + 1)
2 2
Near 2: f ( x ) ≈ ( 2 + 2 )( 2 + 1)( x − 2 ) = 12 ( x − 2 )
Plot the point ( −2, 0 ) and show a line with negative slope there.
Plot the point ( −1, 0 ) and show a line with positive slope there.
Plot the point ( 2, 0 ) and show a parabola opening up there.

2
y-intercept: f ( 0 ) = 04 − 03 − 6 ( 0 ) + 4 ( 0 ) + 8 = 8
The graph of f crosses the x-axis at x = –2 and –1 since each zero has multiplicity 1.
The graph of f touches the x-axis at x = 2 since the zero has multiplicity 2.

Interval ( −∞, −2 ) ( −2, −1) ( −1, 2 ) ( 2, ∞ )


Number Chosen –3 –1.5 0 3
Value of f 50 –3.0625 8 20
Location of Graph Above x-axis Below x-axis Above x-axis Above x-axis
Point on Graph ( −3, 50 ) ( −1.5, −3.0625 ) ( 0,8 ) ( 3, 20 )

103. f ( x) = 4 x5 − 8 x 4 − x + 2 = ( x − 2) ( 2x −1 )( )(
2 x + 1 2x2 + 1 )
2 2
x-intercepts: − , ,2
2 2
2  2   2    2 2 
Near − : f ( x ) ≈  − − 2   2  −
  − 1 ( )
2 x + 1  2   + 1 = 2
  2  
( 2+4 )( 2x +1)
2  2   2    
  2    2  
2
2  2 
Near : f ( x ) ≈  − 2  ( 2 x − 1  2 
 )
 + 1
     2 
 + 1 =2 ( 2 −4 )( 2x −1 )
2  2    2     2  

Near 2: f ( x ) ≈ ( x − 2 ) ( 2 ( 2) − 1 )( )( 2
)
2 ( 2 ) + 1 2 ( 2 ) + 1 = 63 ( x − 2 )

 2 
Plot the point  − , 0  and show a line with positive slope there.
 2 

311
Copyright © 2015 Pearson Education, Inc.
Chapter 3: Polynomial and Rational Functions

 2 
Plot the point  , 0  and show a line with negative slope there.
 2 
Plot the point ( 2, 0 ) and show a line with positive slope there.

5 4
y-intercept: f ( 0 ) = 4 ( 0 ) − 8 ( 0 ) − 0 + 2 = 2
2 2
The graph of f crosses the x-axis at x = − , x= and x = 2 since each zero has multiplicity 1.
2 2

 2  2 2  2 
Interval  −∞, −   − ,   , 2  ( 2, ∞ )
 2   2 2   2 
Number Chosen –1 0 1 3
Value of f –9 2 –3 323
Location of Graph Below x-axis Above x-axis Below x-axis Above x-axis
Point on Graph ( −1, −9 ) ( 0, 2 ) (1, −3) ( 3,323)

104. f ( x) = 4 x5 + 12 x 4 − x − 3 = ( x + 3) ( 2x −1 )( )(
2 x + 1 2 x2 + 1 )
2 2
x-intercepts: −3, − ,
2 2
Near −3 : f ( x ) ≈ ( x + 3) ( 2 ( −3) − 1 )( )( 2
)
2 ( −3) + 1 2 ( −3) + 1 = 323 ( x + 3)

2  2   2    2
2 
Near − : f ( x ) ≈  − + 3   2  −
  − 1 ( ) 
2 x + 1 2  −
  2 
 + 1 = 2

( 2 −6 )( )
2x +1
2  2   2    
  2    2  
2
2  2 
Near : f ( x ) =  + 3  ( )
2 x − 1  2   + 1
     2 
 + 1 =2 ( 2 +6 )( 2x −1 )
2  2    2     2  

Plot the point ( −3, 0 ) and show a line with positive slope there.
 2 
Plot the point  − , 0  and show a line with negative slope there.
 2 
 2 
Plot the point  , 0  and show a line with positive slope there.
 2 

312
Copyright © 2015 Pearson Education, Inc.
Section 3.2: The Real Zeros of a Polynomial Function

5 4
y-intercept: f ( 0 ) = 4 ( 0 ) + 12 ( 0 ) − 0 − 3 = −3
2 2
The graph of f crosses the x-axis at x = − , x= and x = −3 since each zero has multiplicity 1.
2 2

 2  2 2  2 
Interval ( −∞, −3)  −3, −   − ,   , ∞ 
 2   2 2   2 
Number Chosen –4 –2 0 1
Value of f –1023 63 –3 12
Location of Graph Below x-axis Above x-axis Below x-axis Above x-axis
Point on Graph ( −4, −1023) ( −2, 63) ( 0, −3) (1,12 )

105. f ( x) = 3x 3 + 16 x 2 + 3x − 10 will factor into 107. From the Remainder and Factor Theorems,
f ( x) = ( x + 1)(3x − 2)( x + 5) . Solving x − 2 is a factor of f if f ( 2 ) = 0 .
( x + 1)(3x − 2)( x + 5) = 0 we get ( 2 )3 − k ( 2 ) 2 + k ( 2 ) + 2 = 0
2 8 − 4k + 2k + 2 = 0
x = −1, x = , x = −5 . f ( x + 3) would shift the
3 −2k + 10 = 0
graph left by three units and thus would shift the −2k = −10
zeros left by three units. So the zeros would be
x = −1 − 3 = −4 k =5
2 7 108. From the Remainder and Factor Theorems,
x= −3 = −
3 3 x + 2 is a factor of f if f ( −2 ) = 0 .
x = −5 − 3 = −8
( −2 )4 − k ( −2 )3 + k ( −2 )2 + 1 = 0
106. f ( x) = 4 x3 − 11x 2 − 26 x + 24 will factor into 16 + 8k + 4k + 1 = 0
f ( x) = ( x + 2)(4 x − 3)( x − 4) . Solving 12k + 17 = 0
( x + 2)(4 x − 3)( x − 4) = 0 we get 12k = −17
3 17
x = −2, x = , x = 4 . f ( x − 2) would shift the k =−
4 12
graph right by two units and thus would shift the
zeros right by two units. So the zeros would be 109. From the Remainder Theorem, we know that the
x = −2 + 2 = 0 remainder is
20 10
3 11 f (1) = 2 (1) − 8 (1) + 1 − 2 = 2 − 8 + 1 − 2 = −7
x= +2=
4 4 The remainder is –7.
x = 4+2 = 6

313
Copyright © 2015 Pearson Education, Inc.
Chapter 3: Polynomial and Rational Functions

( )
f ( x ) = x3 + 5 x 2 + 5 x − 2 = ( x + 2 ) x 2 + 3x − 1 .
110. From the Remainder Theorem, we know that the 2
Solving x + 3x − 1 = 0 ,
remainder is
17 9 5 −3 ± 9 + 4 −3 ± 13
f ( −1) = −3 ( −1) + ( −1) − ( −1) + 2 ( −1) = 1 x= = .
2 2
The remainder is 1. The sum of these two roots is
111. We want to prove that x − c is a factor of −3 + 13 −3 − 13 −6
+ = = −3 .
x n − c n , for any positive integer n. By the Factor 2 2 2
Theorem, x − c will be a factor of f ( x ) 115. Let x be the length of a side of the original
provided f ( c ) = 0 . Here, f ( x ) = x n − c n , so cube. After removing the 1-inch slice, one
dimension will be x − 1 . The volume of the new
that f ( c ) = c n − c n = 0 . Therefore, x − c is a solid will be: ( x − 1) ⋅ x ⋅ x .
factor of x n − c n . Solve the volume equation:
( x − 1) ⋅ x ⋅ x = 294
112. We want to prove that x + c is a factor of
x3 − x 2 = 294
x n + c n , if n ≥ 1 is an odd integer. By the Factor
Theorem, x + c will be a factor of f ( x ) x3 − x 2 − 294 = 0
The solutions to this equation are the same as the
provided f ( −c ) = 0 . Here, f ( x ) = x n + c n , so real zeros of f ( x ) = x3 − x 2 − 294 .
n
that f ( −c ) = ( −c ) + c n = −c n + c n = 0 if n ≥ 1 By Descartes’ Rule of Signs, we know that there
is an odd integer. Therefore, x + c is a factor of is one positive real zero.
p = ±1, ±2, ±3, ±6, ±7, ±14, ±21, ±42, ±49, ±98,
x n + c n if n ≥ 1 is an odd integer.
±147, ±294
113. x3 − 8 x 2 + 16 x − 3 = 0 has solution x = 3 , so q = ±1
x − 3 is a factor of f ( x ) = x3 − 8 x 2 + 16 x − 3 . The possible rational zeros are the same as the
values for p.
Using synthetic division
p
3 1 − 8 16 − 3 = ±1, ±2, ±3, ±6, ±7, ±14, ±21, ±42, ±49, ±98,
q
3 − 15 3
±147, ±294
1 −5 1 0
Using synthetic division:
Thus, 7)1 −1 0 − 294
( )
f ( x ) = x3 − 8 x 2 + 16 x − 3 = ( x − 3) x 2 − 5 x + 1 . 7 42 294
2 1 6 42 0
Solving x − 5 x + 1 = 0
7 is a zero, so the length of the edge of the
5 ± 25 − 4 5 ± 21 original cube was 7 inches.
x= =
2 2
The sum of these two roots is 116. Let x be the length of a side of the original
3
5 + 21 5 − 21 10 cube. The volume is x . The dimensions are
+ = = 5. changed to x + 6, x + 12, and x − 4 . The volume
2 2 2
of the new solid will be (x + 6)(x + 12)(x − 4) .
114. x3 + 5 x 2 + 5 x − 2 = 0 has solution x = −2 , so Solve the volume equation:
x + 2 is a factor of f ( x ) = x3 + 5 x 2 + 5 x − 2 . ( x + 6)( x + 12)( x − 4) = 2 x3
Using synthetic division (x 2
)
+ 18 x + 72 ( x − 4) = 2 x3
−2 1 5 5 −2
x3 + 14 x 2 − 288 = 2 x3
−2 −6 2
x3 − 14 x 2 + 288 = 0
1 3 −1 0
The solutions to this equation are the same as the
Thus,
real zeros of f ( x ) = x3 − 14 x 2 + 288 .

314
Copyright © 2015 Pearson Education, Inc.
Section 3.2: The Real Zeros of a Polynomial Function

x − 8x − 48 = (x − 12 )( x + 4) . The other
2
By Descartes’ Rule of Signs, we know that there
are two positive real zeros or no positive real zeros are 12 and –4. The length of the edge of
zeros. the original cube was 6 inches or 12 inches.
p = ±1, ±2, ±3, ±4, ±6, ±8, ±9, ±12, ±16, ±18, ±24,
±32, ±36, ±48, ±72, ±96, ±144, ±288 117. f ( x ) = x n + an −1 x n −1 + an − 2 x n − 2 + ... + a1 x + a0 ;
q = ±1 where an −1 , an − 2 ,...a1 , a0 are integers. If r is a
The possible rational zeros are the same as the real zero of f , then r is either rational or
values for p:
irrational. We know that the rational roots of f
p
= ±1, ±2, ±3, ±4, ±6, ±8, ±9, ±12, ±16, ±18, ±24, p
q must be of the form where p is a divisor of
q
±32, ±36, ±48, ±72, ±96, ±144, ±288
Using synthetic division: a0 and q is a divisor of 1. This means that
6)1 −14 0 288 q = ±1 . So if r is rational, then r =
p
= ±p .
6 − 48 − 288 q
1 − 8 − 48 0 Therefore, r is an integer or r is irrational.
Therefore, 6 is a zero; the other factor is
p
118. Let be a root of the polynomial f ( x ) = an x n + an −1 x n −1 + an − 2 x n − 2 + ... + a1 x + a0 where
q
an , an −1 , an − 2 ,...a1 , a0 are integers . Suppose also that p and q have no common factors other than 1 and – 1. Then
n n −1 n−2
 p  p  p  p  p
f   = an   + an −1   + an − 2   + ... + a1   + a0 = 0
q
  q
  q q q

1

q n (anp
n
)
+ an −1 p n −1q + an − 2 p n − 2 q 2 + ... + a1 pq n −1 + a0 q n = 0

 an p n + an −1 p n −1q + an − 2 p n − 2 q 2 + ... + a1 pq n −1 + a0 q n = 0
 an p n + an −1 p n −1q + an − 2 p n − 2 q 2 + ... + a1 pq n −1 = −a0 q n
Because p is a factor of the left side of this equation, p must also be a factor of a0 q n . Since p is not a factor of q, p
must be a factor of a0 . Similarly, q must be a factor of an

119. f ( x ) = 8x4 − 2 x2 + 5x − 1 0 ≤ r ≤1

We begin with the interval [0,1].


f ( 0 ) = −1; f (1) = 10
Let mi = the midpoint of the interval being considered.
So m1 = 0.5
n mn f ( mn ) New interval
1 0.5 f ( 0.5 ) = 1.5 > 0 [0,0.5]
2 0.25 f ( 0.25 ) = 0.15625 > 0 [0,0.25]
3 0.125 f ( 0.125 ) ≈ −0.4043 < 0 [0.125,0.25]
4 0.1875 f ( 0.1875 ) ≈ −0.1229 < 0 [0.1875,0.25]
5 0.21875 f ( 0.21875 ) ≈ 0.0164 > 0 [0.1875,0.21875]
6 0.203125 f ( 0.203125 ) ≈ −0.0533 < 0 [0.203125,0.21875]

315
Copyright © 2015 Pearson Education, Inc.
Chapter 3: Polynomial and Rational Functions

7 0.2109375 f ( 0.2109375 ) ≈ −0.0185 < 0 [0.2109375,0.21875]


8 0.21484375 f ( 0.21484375 ) ≈ −0.00105 < 0 [0.21484375,0.21875]
9 0.216796875 f ( 0.216796875 ) ≈ 0.007655271 > 0 [0.21484375,0.2167968]
10 0.2158203125 f ( 0.216796875 ) ≈ 0.007655271 > 0 [0.21484375,0.21582031]
11 0.21533203125 f ( 0.216796875 ) ≈ 0.007655271 > 0 [0.21484375,0.21533203]
Since the endpoints of the new interval at Step 11 agree to three decimal places,
r = 0.215, correct to three decimal places.

120. f ( x ) = 2 x3 + 3 x 2 − 6 x + 7 124. 2 x − 5 y = 3
By the Rational Zero Theorem, the only possible −5 y = −2 x + 3
p 1 7 2 3
rational zeros are: = ±1, ±7, ± , ± . y = x−
q 2 2 5 5
1
Since is not in the list of possible rational 125. [3,8)
3
zeros, it is not a zero of f . 126. y-intercepts: 3(0) + y 2 = 12

121. f ( x ) = 4 x3 − 5 x 2 − 3x + 1 y 2 = 12
By the Rational Zero Theorem, the only possible y = ± 12 = −2 3, 2 3

rational zeros are:


p 1 1
= ±1, ± , ± . x-intercepts: 3 x + (0) 2 = 12
q 2 4 3 x = 12
1 x=4
Since is not in the list of possible rational
3
zeros, it is not a zero of f . ( )( )
So the intercepts are: 0, −2 3 , 0, 2 3 , ( 4, 0)

122. f ( x ) = 2 x 6 − 5 x 4 + x3 − x + 1 127. Increasing: ( −3, 2) and (5, ∞ )


By the Rational Zero Theorem, the only possible
p 1
rational zeros are: = ±1, ± .
q 2
3
Since is not in the list of possible rational
5
zeros, it is not a zero of f (x) .

123. f ( x ) = x7 + 6 x5 − x 4 + x + 2
By the Rational Zero Theorem, the only possible
p
rational zeros are: = ±1, ±2 .
q
2
Since is not in the list of possible rational
3
zeros, it is not a zero of f .

316
Copyright © 2015 Pearson Education, Inc.
Section 3.3: Complex Zeros; Fundamental Theorem of Algebra

Section 3.3 14. Since complex zeros appear in conjugate pairs,


2 + i , the conjugate of 2 − i , and i , the
1. ( 3 − 2i ) + ( −3 + 5i ) = 3 − 3 − 2i + 5i conjugate of −i , are the remaining zeros of f .
= 3i 15. Since complex zeros appear in conjugate pairs,
( 3 − 2i )( −3 + 5i ) −9 + 15i + 6i − 10i 2
= 2 − i , the conjugate of 2 + i , and −3 + i , the
= −9 + 21i − 10 ( −1) conjugate of −3 − i , are the remaining zeros.
= 1 + 21i 16. Since complex zeros appear in conjugate pairs,
−i , the conjugate of i , 3 + 2i , the conjugate of
2. The zeros of f ( x ) are the solutions to the 3 − 2i , and − 2 − i , the conjugate of − 2 + i , are
equation x 2 + 2 x + 2 = 0 . the remaining zeros of f .
x2 + 2 x + 2 = 0
For 17–22, we will use an = 1 as the lead coefficient of
a = 1, b = 2, c = 2
the polynomial. Also note that
−2 ± 22 − 4 (1)( 2 ) −2 ± −4 −2 ± 2i
x= = = = −1 ± i ( x − ( a + b i )) ( x − ( a − b i )) = (( x − a ) − b i ) (( x − a ) + b i )
2 (1) 2 2
= ( x − a ) − (b i )
2 2

The solution set is {−1 − i, − 1 + i} .


17. Since 3 + 2i is a zero, its conjugate 3 − 2i is also
3. one
a zero of f .
4. 3 − 4i
f ( x) = ( x − 4)( x − 4) ( x − (3 + 2i ) ) ( x − (3 − 2i ) )
5. True
6. False; would also need −3 − 5i
(
= x 2 − 8 x + 16 ) ( ( x − 3) − 2i ) ( ( x − 3) + 2i )
= (x − 8 x + 16 )( x − 6 x + 9 − 4i )
2 2 2
7. Since complex zeros appear in conjugate pairs,
4 + i , the conjugate of 4 − i , is the remaining = (x − 8 x + 16 )( x − 6 x + 13 )
2 2

zero of f .
= x 4 − 6 x 3 + 13 x 2 − 8 x3 + 48 x 2
8. Since complex zeros appear in conjugate pairs,
3 − i , the conjugate of 3 + i , is the remaining − 104 x + 16 x 2 − 96 x + 208
zero of f . = x 4 − 14 x 3 + 77 x 2 − 200 x + 208

9. Since complex zeros appear in conjugate pairs,


−i , the conjugate of i , and 1 − i , the conjugate
18. Since 1 + 2i and i are zeros, their conjugates
of 1 + i , are the remaining zeros of f .
1 − 2i and −i are also zeros of f .
10. Since complex zeros appear in conjugate pairs,
2 − i , the conjugate of 2 + i , is the remaining f ( x) = ( x − i )( x − (−i )) ( x − (1 + 2i ) )( x − (1 − 2i ) )
zero of f . = ( x − i )( x + i ) ( ( x − 1) − 2i )( ( x − 1) + 2i )
11. Since complex zeros appear in conjugate pairs, (
= x2 − i 2)( x − 2 x + 1 − 4i )
2 2

−i , the conjugate of i , and − 2i , the conjugate


= ( x + 1)( x − 2 x + 5 )
2 2
of 2i , are the remaining zeros of f .
= x 4 − 2 x3 + 5 x 2 + 1x 2 − 2 x + 5
12. Since complex zeros appear in conjugate pairs,
−i , the conjugate of i , is the remaining zero. = x 4 − 2 x3 + 6 x 2 − 2 x + 5

13. Since complex zeros appear in conjugate pairs, 19. Since −i is a zero, its conjugate i is also a zero,
−i , the conjugate of i , is the remaining zero. and since 1 + i is a zero, its conjugate 1 − i is
also a zero of f .

317
Copyright © 2015 Pearson Education, Inc.
Chapter 3: Polynomial and Rational Functions

f ( x) = ( x − 2)( x + i )( x − i ) ( x − (1 + i ) ) ( x − (1 − i ) )

( ) ( ( x − 1) − i ) ( ( x − 1) + i )
= ( x − 2) x 2 − i 2

= ( x − 2) ( x + 1)( x − 2 x + 1 − i )
2 2 2

= ( x − 2 x + x − 2 )( x − 2 x + 2 )
3 2 2

= x5 − 2 x 4 + 2 x3 − 2 x 4 + 4 x3 − 4 x 2
+ x3 − 2 x 2 + 2 x − 2 x 2 + 4 x − 4
= x5 − 4 x 4 + 7 x3 − 8 x 2 + 6 x − 4

20. Since i is a zero, its conjugate −i is also a zero; since 4 − i is a zero, its conjugate 4 + i is also a zero; and since
2 + i is a zero, its conjugate 2 − i is also a zero of f .
f ( x) = ( x + i )( x − i ) ( x − (4 + i ) ) ( x − (4 − i ) ) ( x − (2 + i ) ) ( x − (2 − i ) )

(
= x2 − i2 ) ( ( x − 4) − i ) ( ( x − 4) + i ) ( ( x − 2) − i ) ( ( x − 2) + i )
= (x 2
+ 1)( x − 8 x + 16 − i )( x − 4 x + 4 − i )
2 2 2 2

= (x 2
+ 1)( x − 8 x + 17 )( x − 4 x + 5 )
2 2

= (x 4
− 8 x + 17 x + x − 8 x + 17 )( x − 4 x + 5 )
3 2 2 2

= (x 4
− 8 x + 18 x − 8 x + 17 )( x − 4 x + 5 )
3 2 2

= x 6 − 4 x 5 + 5 x 4 − 8 x 5 + 32 x 4 − 40 x3 + 18 x 4 − 72 x3 + 90 x 2 − 8 x3 + 32 x 2 − 40 x + 17 x 2 − 68 x + 85
= x 6 − 12 x5 + 55 x 4 − 120 x3 + 139 x 2 − 108 x + 85
21. Since −i is a zero, its conjugate i is also a zero. 23. Since 2i is a zero, its conjugate − 2i is also a
f ( x) = ( x − 3)( x − 3)( x + i ) ( x − i ) zero of f . x − 2i and x + 2i are factors of f .
( 2
= x − 6x + 9 x − i )( 2 2
) Thus, ( x − 2i )( x + 2i ) = x 2 + 4 is a factor of f .
Using division to find the other factor:
= (x 2
− 6 x + 9 )( x 2
+1 )
x−4
= x 4 + x 2 − 6 x3 − 6 x + 9 x 2 + 9
x + 4 x3 − 4 x 2 + 4 x − 16
2
= x 4 − 6 x3 + 10 x 2 − 6 x + 9
x3 + 4x
22. Since 1 + i is a zero, its conjugate 1 − i is also a
− 4 x2 − 16
zero of f .
2
− 4x − 16
f ( x) = ( x − 1)3 ( x − (1 + i ) ) ( x − (1 − i ) )

( )
= x3 − 3 x 2 + 3x − 1 ( ( x − 1) − i ) ( ( x − 1) + i ) x − 4 is a factor, so the remaining zero is 4.
The zeros of f are 4, 2i, − 2i .
= (x 3
− 3x 2 + 3x − 1)( x 2
− 2x + 1 − i2 )
24. Since −5i is a zero, its conjugate 5i is also a
= (x 3
− 3x 2 + 3x − 1)( x 2
− 2x + 2 ) zero of g . x + 5 i and x − 5 i are factors of g .
= x5 − 2 x 4 + 2 x3 − 3 x 4 + 6 x3 − 6 x 2 Thus, ( x + 5 i )( x − 5 i ) = x 2 + 25 is a factor of g .
+ 3 x3 − 6 x 2 + 6 x − x 2 + 2 x − 2 Using division to find the other factor:
= x5 − 5 x 4 + 11x3 − 13x 2 + 8 x − 2

318
Copyright © 2015 Pearson Education, Inc.
Section 3.3: Complex Zeros; Fundamental Theorem of Algebra

x+3 3x 2 + 5 x − 2 = (3 x − 1)( x + 2)
2 3 2
x + 25 x + 3x + 25 x + 75
1
x3 + 25 x The remaining zeros are and − 2 .
3
3x 2 + 75 1
2
The zeros of h are 3i, − 3i, − 2, .
3x + 75 3

x + 3 is a factor, so the remaining zero is –3. 27. Since 3 − 2i is a zero, its conjugate 3 + 2i is also
The zeros of g are –3, 5 i, − 5 i . a zero of h . x − (3 − 2i ) and x − (3 + 2i ) are
factors of h .
25. Since − 2i is a zero, its conjugate 2i is also a Thus,
zero of f . x − 2i and x + 2i are factors of f . ( x − (3 − 2i ))( x − (3 + 2i )) = (( x − 3) + 2i )(( x − 3) − 2i )

Thus, ( x − 2i )( x + 2i ) = x 2 + 4 is a factor of f . = x 2 − 6 x + 9 − 4i 2
Using division to find the other factor: = x 2 − 6 x + 13
is a factor of h .
2 x2 + 5x − 3 Using division to find the other factor:
x 2 + 4 2 x 4 + 5 x3 + 5 x 2 + 20 x − 12
x 2 − 3x − 10
2 x4 + 8x2
x 2 − 6 x + 13 x 4 − 9 x3 + 21x 2 + 21x − 130
3 2
5 x − 3x + 20 x
x 4 − 6 x3 + 13 x 2
3
5x + 20 x
− 3 x3 + 8 x 2 + 21x
2
− 3x − 12
− 3x3 + 18 x 2 − 39 x
2
− 3x − 12
− 10 x 2 + 60 x − 130
2 x 2 + 5 x − 3 = (2 x − 1)( x + 3) − 10 x 2 + 60 x − 130
x 2 − 3 x − 10 = ( x + 2)( x − 5)
1
The remaining zeros are and − 3 . The remaining zeros are –2 and 5.
2
The zeros of h are 3 − 2i, 3 + 2i, − 2, 5 .
1
The zeros of f are 2i, − 2i, − 3, .
2 28. Since 1 + 3i is a zero, its conjugate 1 − 3i is also
a zero of f . x − (1 + 3 i ) and x − (1 − 3 i ) are
26. Since 3i is a zero, its conjugate − 3i is also a
factors of f . Thus,
zero of h . x − 3 i and x + 3 i are factors of h .
( x − (1 + 3i ))( x − (1 − 3i )) = (( x − 1) − 3i )(( x − 1) + 3i ) is
Thus, ( x − 3i )( x + 3i ) = x 2 + 9 is a factor of h .
a factor of f .
Using division to find the other factor:
(( x − 1) − 3i )(( x − 1) + 3i ) = x 2 − 2 x + 1 − 9i 2
2
3x + 5 x − 2 = x 2 − 2 x + 10
2 4 3 2
x + 9 3x + 5 x + 25 x + 45 x − 18 Using division to find the other factor:
3x 4 + 27 x 2
5 x3 − 2 x 2 + 45 x
5 x3 + 45 x
− 2 x2 − 18
2
− 2x − 18

319
Copyright © 2015 Pearson Education, Inc.
Chapter 3: Polynomial and Rational Functions

x2 − 5x − 6 2 x3 − 3x 2 − 23 x + 12
x 2 − 2 x + 10 x 4 − 7 x3 + 14 x 2 − 38 x − 60 x 2 + 9 2 x5 − 3 x 4 − 5 x3 − 15 x 2 − 207 x + 108
x 4 − 2 x3 + 10 x 2 2 x5 + 18 x3
− 5 x3 + 4 x 2 − 38 x − 3x 4 − 23x3 − 15 x 2
− 5 x3 + 10 x 2 − 50 x −3 x 4 − 27 x 2
− 6 x 2 + 12 x − 60 − 23x3 + 12 x 2 − 207 x
− 6 x 2 + 12 x − 60 − 23x3 − 207 x
2 2
x − 5 x − 6 = ( x + 1)( x − 6) 12 x + 108
The remaining zeros are –1 and 6. 12 x 2 + 108
The zeros of f are 1 + 3i, 1 − 3i, − 1, 6 .
0
29. Since − 4i is a zero, its conjugate 4i is also a
Using the Rational Root theorem, we see that
zero of h . x − 4i and x + 4i are factors of h . −3 is a potential rational zero.
Thus, ( x − 4i )( x + 4i ) = x 2 + 16 is a factor of h .
−3 2 − 3 − 23 12
Using division to find the other factor:
−6 27 − 12
3 2
3 x + 2 x − 33 x − 22 2 −9 4 0
2 5 4 3 2
x + 16 3 x + 2 x + 15 x + 10 x − 528 x − 352 x + 3 is a factor. The remaining factor is
3x
5
+ 48 x 3
2 x 2 − 9 x + 4 = (2 x − 1)( x − 4) .
4 3
2 x − 33 x + 10 x
2 1
The zeros of g are 3i, − 3i, − 3, , 4.
4
2
2x + 32 x 2

− 33 x3 − 22 x 2 − 528 x
31. (
f ( x) = x3 − 1 = ( x − 1) x 2 + x + 1 The solutions )
of x 2 + x + 1 = 0 are:
− 33 x3 − 528 x

− 22 x 2 − 352 −1 ± 12 − 4 (1) (1) −1 ± −3


x= =
− 22 x 2
− 352 2 (1) 2
1 3 1 3
3x3 + 2 x 2 − 33 x − 22 = x 2 (3 x + 2) − 11(3x + 2) =− + i and − − i
2 2 2 2
= (3x + 2)( x 2 − 11) 1 3 1 3
The zeros are: 1, − + i, − − i.
2 2 2 2
(
= (3 x + 2) x − 11 x + 11 )( )  1 3  1 3 
f ( x ) = ( x − 1)  x + − i x + + i
2  2 2   2 2 
The remaining zeros are − , 11, and − 11 . 
3
2
The zeros of h are 4i, − 4i, − 11, 11, − .
32. (
f ( x) = x 4 − 1 = x 2 − 1 x 2 + 1 )( )
3
(
= ( x − 1)( x + 1) x 2 + 1 )
30. Since 3 i is a zero, its conjugate − 3i is also a
The solutions of x 2 + 1 = 0 are x = ± i .
zero of g . x − 3 i and x + 3 i are factors of g .
The zeros are: −1, 1, − i, i .
Thus, ( x − 3 i )( x + 3 i ) = x 2 + 9 is a factor of g .
f ( x ) = ( x + 1)( x − 1)( x + i )( x − i )
Using division to find the other factor:

320
Copyright © 2015 Pearson Education, Inc.
Section 3.3: Complex Zeros; Fundamental Theorem of Algebra

33. f ( x) = x3 − 8 x 2 + 25 x − 26 Step 3: Possible rational zeros:


p = ±1, ± 5, ± 17, ± 85; q = ±1;
Step 1: f ( x) has 3 complex zeros.
p
= ±1, ± 5, ± 17, ± 85
Step 2: By Descartes Rule of Signs, there are q
three positive real zeros or there is one
positive real zero. Step 4: Using synthetic division:

f (− x) = (− x)3 − 8(− x) 2 + 25(− x) − 26 , thus, We try x + 5 :

= − x3 − 8 x 2 − 25 x − 26 −5 1 13 57 85
there are no negative real zeros. − 5 − 40 − 85
Step 3: Possible rational zeros: 1 8 17 0
p = ±1, ± 2, ± 13, ± 26; q = ±1; x + 5 is a factor. The other factor is the
p quotient: x 2 + 8 x + 17 .
= ±1, ± 2, ± 13, ± 26
q
The solutions of x 2 + 8 x + 17 = 0 are:
Step 4: Using synthetic division:
− 8 ± 82 − 4 (1)(17 ) −8 ± − 4
We try x − 2 : x= =
2 (1) 2
2 1 −8 25 − 26 − 8 ± 2i
= = −4±i
2 − 12 26 2
1 −6 13 0 The zeros are −5, − 4 − i, − 4 + i .
f ( x ) = ( x + 5 )( x + 4 + i )( x + 4 − i )
x − 2 is a factor. The other factor is the
quotient: x 2 − 6 x + 13 . 35. (
f ( x) = x 4 + 5 x 2 + 4 = x 2 + 4 x 2 + 1 )( )
The solutions of x 2 − 6 x + 13 = 0 are: = ( x + 2i )( x − 2i )( x + i )( x − i )

The zeros are: − 2i, − i, i, 2i .


−(− 6) ± (− 6) 2 − 4(1)(13)
x= .
2(1)
36. (
f ( x) = x 4 + 13 x 2 + 36 = x 2 + 4 x 2 + 9 )( )
6 ± −16 6 ± 4i
= = = 3 ± 2i = ( x + 2i )( x − 2i )( x + 3i )( x − 3i )
2 2
The zeros are 2, 3 − 2i, 3 + 2i . The zeros are: − 3 i, − 2 i, 2 i, 3 i .
f ( x ) = ( x − 2 )( x − 3 + 2i )( x − 3 − 2i ) f ( x ) = ( x + 3i )( x + 2i )( x − 2i )( x − 3i )

34. f ( x) = x3 + 13 x 2 + 57 x + 85 37. f ( x) = x 4 + 2 x3 + 22 x 2 + 50 x − 75

Step 1: f ( x) has 3 complex zeros. Step 1: f ( x) has 4 complex zeros.


Step 2: By Descartes Rule of Signs, there are Step 2: By Descartes Rule of Signs, there is 1
no positive real zeros. positive real zero.
f (− x) = (− x)3 + 13(− x) 2 + 57(− x) + 85 , thus, f (− x) = (− x) 4 + 2(− x)3 + 22(− x)2 + 50(− x) − 75
= − x3 + 13x 2 − 57 x + 85 = x 4 − 2 x3 + 22 x 2 − 50 x − 75
there are three negative real zeros or there is one
negative real zero. Thus, there are three negative real zeros or there
is one negative real zero.

321
Copyright © 2015 Pearson Education, Inc.
Chapter 3: Polynomial and Rational Functions

Step 3: Possible rational zeros: −7 1 3 − 19 27 − 252


p = ±1, ± 3, ± 5, ± 15, ± 25, ± 75; q = ±1; −7 28 − 63 252
p 1 −4 9 − 36 0
= ±1, ± 3, ± 5, ± 15, ± 25, ± 75
q
x + 7 is a factor. The other factor is the
Step 4: Using synthetic division: quotient:

We try x + 3 : x3 − 4 x 2 + 9 x − 36 = x 2 ( x − 4) + 9( x − 4) .

−3 1 2 22 50 − 75 (
= ( x − 4) x 2 + 9 )
−3 3 − 75 75 = ( x − 4)( x + 3 i )( x − 3 i )
1 −1 25 − 25 0 The zeros are −7, 4, − 3 i, 3 i .
x + 3 is a factor. The other factor is the f ( x ) = ( x + 7 )( x − 4 )( x + 3i )( x − 3i )
3 2
quotient: x − x + 25 x − 25 .
39. f ( x) = 3x 4 − x3 − 9 x 2 + 159 x − 52
3 2 2
x − x + 25 x − 25 = x ( x − 1) + 25( x − 1)
Step 1: f ( x ) has 4 complex zeros.
(
= ( x − 1) x 2 + 25 )
= ( x − 1)( x + 5i )( x − 5i ) Step 2: By Descartes Rule of Signs, there are
three positive real zeros or there is one positive
The zeros are −3, 1, − 5 i, 5 i . real zero.
f ( x ) = ( x + 3)( x − 1)( x + 5i )( x − 5i ) f ( − x ) = 3( − x ) 4 − ( − x )3 − 9( − x) 2 + 159( − x ) − 52
4 3 2
= 3 x + x − 9 x − 159 x − 52
38. f ( x) = x 4 + 3 x3 − 19 x 2 + 27 x − 252
Thus, there is 1 negative real zero.
Step 1: f ( x ) has 4 complex zeros.
Step 3: Possible rational zeros:
Step 2: By Descartes Rule of Signs, there are
three positive real zeros or there is one positive p = ±1, ± 2, ± 4, ± 13, ± 26, ± 52;
real zero. q = ±1, ± 3;
f ( − x ) = ( − x ) 4 + 3( − x)3 − 19( − x ) 2 + 27( − x) − 252 p
= ±1, ± 2, ± 4, ± 13, ± 26, ± 52,
4 3 2 q
= x − 3 x − 19 x − 27 x − 252
1 2 4 13 26 52
± ,± ,± ,± ,± ,±
Thus, there is 1 negative real zero. 3 3 3 3 3 3
Step 3: Possible rational zeros: Step 4: Using synthetic division:
p = ±1, ± 2, ± 3, ± 4, ± 6, ± 7, ± 9, We try x + 4 :
± 12, ± 14, ± 18, ± 21, ± 28, ± 36,
−4 3 −1 − 9 159 − 52
± 42, ± 63, ± 84, ± 126, ± 252;
− 12 52 − 172 52
q = ±1;
3 − 13 43 − 13 0
The possible rational zeros are the same as the
values of p . x + 4 is a factor and the quotient is
3x3 − 13 x 2 + 43x − 13 .
Step 4: Using synthetic division:
1
We try x + 7 : We try x − on 3x3 − 13 x 2 + 43x − 13 :
3

322
Copyright © 2015 Pearson Education, Inc.
Section 3.3: Complex Zeros; Fundamental Theorem of Algebra

1 x − 5 is a factor and the quotient is


3 − 13 43 − 13
3 2 x3 + 11x 2 + 20 x − 13
1 −4 13
3 − 12 39 0 1
We try x − on 2 x3 + 11x 2 + 20 x − 13 :
2
1
x− is a factor and the quotient is 1
3 2 11 20 − 13
2 2
3x − 12 x + 39 . 1 6 13
2 12 26 0
(
3x 2 − 12 x + 39 = 3 x 2 − 4 x + 13 )
1
The solutions of x 2 − 4 x + 13 = 0 are: x− is a factor and the quotient is
2
−(− 4) ± (− 4) 2 − 4(1)(13) 2 x 2 + 12 x + 26 .
x= .
2(1)
4 ± −36 4 ± 6i
(
2 x 2 + 12 x + 26 = 2 x 2 + 6 x + 13 )
= = = 2 ± 3i
2 2
The solutions of x 2 + 6 x + 13 = 0 are:
1
The zeros are − 4, , 2 − 3 i, 2 + 3 i . − 6 ± 62 − 4(1)(13)
3 x= .
2(1)
 1
f ( x ) = 3 ( x + 4 )  x −  ( x − 2 + 3i )( x − 2 − 3i )
 3 − 6 ± −16 − 6 ± 4i
= = = −3 ± 2 i
2 2
40. f ( x) = 2 x 4 + x3 − 35 x 2 − 113 x + 65
1
The zeros are 5, , − 3 − 2 i, − 3 + 2 i .
Step 1: f ( x) has 4 complex zeros. 2
 1
Step 2: By Descartes Rule of Signs, there are f ( x ) = 2 ( x − 5 )  x −  ( x + 3 + 2i )( x + 3 − 2i )
 2
two positive real zeros or no positive real zeros.

f ( − x) = 2( − x) 4 + ( − x )3 − 35( − x) 2 − 113( − x) + 65 41. f ( x) = 2 x3 − 14 x 2 + bx − 3

= 2 x 4 − x3 − 35 x 2 + 113 x + 65 0 = 2(2)3 − 14(2) 2 + b(2) − 3


0 = 16 − 26 + 2b − 3
Thus, there are two negative real zeros or no 43
negative real zeros. b=
2
Step 3: Possible rational zeros: so
p = ±1, ± 5, ± 13, ± 65; q = ±1, ± 2; 43
f ( x) = 2 x3 − 14 x 2 + x−3
p 2
1 5 13 65
= ±1, ± 5, ± 13, ± 65, ± , ± , ± , ±
q 2 2 2 2 g ( x) = (3 − i )3 + (3 − i )2 c − 8(3 − i ) + 30
Step 4: Using synthetic division: 0 = (18 − 26i ) + (8 − 6i )c − (24 − 8i ) + 30
0 = 18 − 26i + 8 − 6ic − 24 + 8i + 30
We try x − 5 :
0 = 18 − 24 + 30 + 8c − 26i − 6ic + 8i
5 2 1 − 35 − 113 65 c = −3
10 55 100 − 65 g ( x) = x3 − 3x 2 − 8 x + 30
2 11 20 − 13 0

323
Copyright © 2015 Pearson Education, Inc.
Chapter 3: Polynomial and Rational Functions

43 13
( 2)
2
f (1) = 2 − 14 + −3= 2± − 4(1)(1)
2 2 x=
g (1) = 1 − 3 − 8 + 30 = 20 2(1)
13 2 ± −2
( f ⋅ g )(1) = 20 = 130 =
2 2
2 −2 2 −2
42. f ( x) = ( x − (3 + i ))( x − (3 − i ))( x − 2)( x + 2) x= − , +
2 2 2 2
= ( x − 3 − i )( x − 3 + i )( x − 2)( x + 2)
= x 4 − 6 x3 + 6 x 2 + 24 x − 40 44. If the coefficients are real numbers and 2 + i is a
zero, then 2 − i would also be a zero. This
g ( x) = a( x 2 + 1)( x 2 + 4) would then require a polynomial of degree 4.

−4 = a(02 + 1)(02 + 4) 45. Three zeros are given. If the coefficients are real
−4 = a(1)(4) numbers, then the complex zeros would also
have their conjugates as zeros. This would mean
a = −1 that there are 5 zeros, which would require a
g ( x) = − ( x 2 + 1)( x 2 + 4) polynomial of degree 5.
= − ( x 4 + 5 x 2 + 4) 46. If the coefficients are real numbers, then
complex zeros must appear in conjugate pairs.
( f + g )( x) = x 4 − 6 x3 + 6 x 2 + 24 x − 40 We have a conjugate pair and one real zero.
Thus, there is only one remaining zero, and it
− x4 − 5x2 − 4 must be real because a complex zero would
= −6 x3 + x 2 + 24 x − 44 require a pair of complex conjugates.
( f + g )( x) = −6 + 1 + 24 − 44 47. One of the remaining zeros must be 4 + i , the
= −25 conjugate of 4 − i . The third zero is a real
number. Thus, the fourth zero must also be a
43. a. f ( x) = x 4 + 1 real number in order to have a degree 4
= x4 + 2 x2 + 1 − 2 x2 polynomial.

( )
2
= x2 + 1 − 2 x2 48. a. f ( x) = (3 − i )2 + 2(3 − i )i − 10
= (8 − 6i ) + 6i + 2 − 10
= ( x + 1 − 2 x )( x
2 2
+ 1+ 2x ) =0
= ( x − 2 x + 1)( x
2 2
+ 2 x + 1)
b. f ( x) = (3 + i ) 2 + 2(3 + i )i − 10
= (8 + 6i ) + 6i − 2 − 10
( 2)
2
− 2± − 4(1)(1)
= 12i − 4 ≠ 0
b. x=
2(1)
c. The coeffieients are not real numbers.
− 2 ± −2
= 49.
2
2 −2 2 −2
x=− − ,− +
2 2 2 2

and

324
Copyright © 2015 Pearson Education, Inc.
Section 3.4: Properties of Rational Functions

50. 3− x = 5 3 units.
y
3 − x = 25
x = −22 5

51. (2 x − 5)(3 x 2 + x − 4) = (2 x)3 x 2 + (2 x) x − (2 x)4


− 5(3x 2 ) − 5( x ) − 5( −4)
−5 x
= 6 x 3 + 2 x 2 − 8 x − 15 x 2 − 5 x + 20 (0, −1) 5
(−2, −1)
= 6 x 3 − 13 x 2 − 13 x + 20
(−1, −3)
52. A = π r 2 −5

= π (3) 2 5. False (See number 3 for an example)


= 9π ft 2 ≈ 28.274 ft 2 6. horizontal asymptote
C = 2π r
7. vertical asymptote
= 2π (3)
= 6π ft ≈ 18.850 ft 8. proper
9. True
10. False, a graph cannot intersect a vertical
Section 3.4 asymptote.
1. True 11. y = 0
2
2. Quotient: 3x + 3 ; Remainder: 2 x − 3 x − 3 12. True
3x + 3
3 2 4 3 2
x − x + 1 3x + 0 x − x + 0 x + 0 4x
13. In R ( x) = , the denominator, q( x) = x − 3 ,
4
− (3x − 3x 3
+ 3 x) x −3
3 2 has a zero at 3. Thus, the domain of R( x) is all
3x − x − 3 x
− (3x3 − 3x 2 + 3) real numbers except 3. { x | x ≠ 3}
2
2 x − 3x − 3
5x2
14. In R ( x) = , the denominator, q( x) = 3 + x ,
1 3+ x
3. y =
x has a zero at –3. Thus, the domain of R ( x) is all
real numbers except –3. { x | x ≠ −3}
− 4 x2
15. In H ( x) = , the denominator,
( x − 2)( x + 4)
q( x) = ( x − 2)( x + 4) , has zeros at 2 and –4.
Thus, the domain of H ( x) is all real numbers
except –4 and 2. { x | x ≠ −4, x ≠ 2}
4. Using the graph of y = x 2 , stretch vertically by a 6
16. In G ( x) = , the denominator,
factor of 2, then shift left 1 unit, then shift down ( x + 3)(4 − x)
q( x) = ( x + 3)(4 − x) , has zeros at –3 and 4.
Thus, the domain of G ( x) is all real numbers
except –3 and 4. { x | x ≠ −3, x ≠ 4}
325
Copyright © 2015 Pearson Education, Inc.
Chapter 3: Polynomial and Rational Functions

3 x( x − 1) − 2( x 2 − 4)
17. In F ( x) = , the denominator, 24. In F ( x) = , the denominator,
2x2 − 5x − 3 3( x 2 + 4 x + 4)
q( x) = 2 x 2 − 5 x − 3 = (2 x + 1)( x − 3) , has zeros at q( x) = 3( x 2 + 4 x + 4) = 3( x + 2) 2 , has a zero at
1 –2. Thus, the domain of F ( x) is all real
− and 3 . Thus, the domain of F ( x) is all real
2
numbers except –2. { x | x ≠ −2}
1  1 
numbers except − and 3 .  x | x ≠ − , x ≠ 3
2 2
  25. a. Domain: {x x ≠ 2} ; Range: {y y ≠ 1}
− x(1 − x) b. Intercept: (0, 0)
18. In Q( x) = , the denominator,
3x 2 + 5 x − 2 c. Horizontal Asymptote: y = 1
q( x) = 3 x 2 + 5 x − 2 = (3x − 1)( x + 2) , has zeros at
d. Vertical Asymptote: x = 2
1
and – 2 . Thus, the domain of Q( x) is all real
3 e. Oblique Asymptote: none
1  1
numbers except –2 and .  x | x ≠ −2, x ≠ 
3  3
26. a. Domain: {x x ≠ −1} ; Range: {y y > 0}

b. Intercept: (0, 2)
x
19. In R ( x) = 3 , the denominator, c. Horizontal Asymptote: y = 0
x −8
q( x) = x3 − 8 = ( x − 2)( x 2 + 2 x + 4) , has a zero d. Vertical Asymptote: x = −1
2
at 2 ( x + 2 x + 4 has no real zeros). Thus, the e. Oblique Asymptote: none
domain of R( x) is all real numbers except 2.
{ x | x ≠ 2} 27. a. Domain: {x x ≠ 0} ;
Range: all real numbers
x b. Intercepts: (–1, 0) and (1, 0)
20. In R ( x) = 4 , the denominator,
x −1
c. Horizontal Asymptote: none
q( x) = x 4 − 1 = ( x − 1)( x + 1)( x 2 + 1) , has zeros at
d. Vertical Asymptote: x = 0
–1 and 1 ( x 2 + 1 has no real zeros). Thus, the
domain of R( x) is all real numbers except –1 e. Oblique Asymptote: y = 2 x
and 1. { x | x ≠ −1, x ≠ 1}
28. a. Domain: {x x ≠ 0} ;

21. In H ( x) =
3x 2 + x
, the denominator,
Range: { y y ≤ − 2 or y ≥ 2 }
x2 + 4
b. Intercepts: none
q( x) = x 2 + 4 , has no real zeros. Thus, the
domain of H ( x) is all real numbers. c. Horizontal Asymptote: none
d. Vertical Asymptote: x = 0
x−3
22. In G ( x) = 4 , the denominator, e. Oblique Asymptote: y = − x
x +1
q( x) = x 4 + 1 , has no real zeros. Thus, the
29. a. Domain: {x x ≠ − 2, x ≠ 2} ;
domain of G ( x) is all real numbers.
Range: {y y ≤ 0 or y > 1}
2
3( x − x − 6)
23. In R( x) = , the denominator, b. Intercept: (0, 0)
4( x 2 − 9)
c. Horizontal Asymptote: y = 1
q( x) = 4( x 2 − 9) = 4( x − 3)( x + 3) , has zeros at 3
and –3. Thus, the domain of R( x) is all real d. Vertical Asymptotes: x = − 2, x = 2
numbers except –3 and 3. { x | x ≠ −3, x ≠ 3} e. Oblique Asymptote: none

326
Copyright © 2015 Pearson Education, Inc.
Section 3.4: Properties of Rational Functions

30. a. Domain: {x x ≠ − 1, x ≠ 1} ;
34. Q( x) =
3 1
= 3   ; Using the function y = ,
1
Range: all real numbers x x
  x
stretch the graph vertically by a factor of 3.
b. Intercept: (0, 0)
c. Horizontal Asymptote: y = 0
d. Vertical Asymptotes: x = − 1, x = 1
e. Oblique Asymptote: none

1 1
31. F ( x ) = 2 + ; Using the function, y = , shift
x x
the graph vertically 2 units up.

−2  1 
35. H ( x) = = −2   ; Using the function
x +1  x +1
1
y= , shift the graph horizontally 1 unit to the
x
left, reflect about the x-axis, and stretch
vertically by a factor of 2.

1 1
32. Q( x) = 3 + 2
; Using the function y = 2 , shift
x x
the graph vertically 3 units up.

2  1 
36. G ( x) = 2
= 2 2 
; Using the
( x + 2)  ( x + 2) 
1
function y = 2 , shift the graph horizontally 2
x
units to the left, and stretch vertically by a factor
1 1 of 2.
33. R ( x ) = ; Using the function, y = ,
( x − 1)
2
x2
shift the graph horizontally 1 unit to the right.

−1 1
37. R ( x ) = =− ; Using the
( x + 2)
2 2
x + 4x + 4
1
function y = , shift the graph horizontally 2
x2

327
Copyright © 2015 Pearson Education, Inc.
Chapter 3: Polynomial and Rational Functions

units to the left, and reflect about the x-axis. about the x-axis, and shift vertically up 2 units.

1 1 x2 − 4 4  1 
38. R( x) = + 1 ; Using the function y = , shift 41. R ( x) = = 1 − 2 = −4  2  + 1 ; Using the
x −1 x x 2
x x 
the graph horizontally 1 unit to the right, and 1
shift vertically 1 unit up. function y = 2 , reflect about the x-axis, stretch
x
vertically by a factor of 4, and shift vertically 1
unit up.

2 2  1 
39. G ( x) = 1 + 2
= 2
+1 = 2 2 
+1 ;
( x − 3) ( x − 3)  ( x − 3) 
1
Using the function y = 2 , shift the graph right 3
x x−4 4 1
42. R ( x ) = = 1 − = −4   + 1 ; Using the
units, stretch vertically by a factor of 2, and shift x x  x
vertically 1 unit up. 1
function y = , reflect about the x-axis, stretch
x
vertically by a factor of 4, and shift vertically 1
unit up.

1  1 
40. F ( x ) = 2 − = −  + 2 ; Using the
x +1  x +1
1
function y = , shift the graph left 1 unit, reflect
x 3x
43. R ( x ) = ; The degree of the numerator,
x+4
p( x) = 3x, is n = 1 . The degree of the
denominator, q( x) = x + 4, is m = 1 . Since

328
Copyright © 2015 Pearson Education, Inc.
Section 3.4: Properties of Rational Functions

3 x+5
n = m , the line y = = 3 is a horizontal
1 x − 5 x − 14 x3
2
+1
asymptote. The denominator is zero at x = − 4 ,
so x = − 4 is a vertical asymptote.
( 3 2
− x − 5 x − 14 x )
2
5 x + 14 x + 1

44. R ( x) =
3x + 5
; The degree of the numerator, (
− 5 x 2 − 25 x − 70 )
x−6 39 x + 71
p( x) = 3 x + 5, is n = 1 . The degree of the 39 x + 71
G ( x) = x + 5 + 2 , x ≠ −2, 7
denominator, q( x) = x − 6, is m = 1 . Since x − 5 x − 14
3 Thus, the oblique asymptote is y = x + 5 .
n = m , the line y = = 3 is a horizontal
1 The denominator is zero at x = −2 and x = 7 , so
asymptote. The denominator is zero at x = 6 , so x = −2 and x = 7 are vertical asymptotes.
x = 6 is a vertical asymptote.
x3
47. T ( x) = ; The degree of the numerator,
x3 − 8 ( x − 2) ( x2 + 2 x + 4) x4 − 1
45. H ( x) = = p( x) = x3 , is n = 3 . The degree of the
x2 − 5x + 6 ( x − 2 )( x − 3)
denominator, q( x) = x 4 − 1 is m = 4 . Since
x2 + 2x + 4
= , where x ≠ 2,3 n < m , the line y = 0 is a horizontal asymptote.
x −3
The degree of the numerator in lowest terms is The denominator is zero at x = −1 and x = 1 , so
n = 2 . The degree of the denominator in lowest x = −1 and x = 1 are vertical asymptotes.
terms is m = 1 . Since n = m + 1 , there is an
oblique asymptote. 4x2
48. P ( x) = ; The degree of the numerator,
Dividing: x3 − 1
x+5 p( x) = 4 x 2 , is n = 2 . The degree of the
x − 3 x2 + 2 x + 4 denominator, q( x) = x3 − 1 is m = 3 . Since
− ( x 2 − 3x ) n < m , the line y = 0 is a horizontal asymptote.
The denominator is zero at x = 1 , so x = 1 is a
5x + 4
vertical asymptote.
− ( 5 x − 15 )
2 x 2 − 5 x − 12 (2 x + 3)( x − 4)
19 49. Q( x) = =
3 x 2 − 11x − 4 (3 x + 1)( x − 4)
19
H ( x) = x + 5 + , x ≠ 2,3 2x + 3 1
x −3 = , where x ≠ − , 4
Thus, the oblique asymptote is y = x + 5 . 3x + 1 3
The degree of the numerator in lowest terms is
The denominator in lowest terms is zero at x = 3
n = 1 . The degree of the denominator in lowest
so x = 3 is a vertical asymptote.
2
terms is m = 1 . Since n = m , the line y = is a

46. G ( x) =
x3 + 1
=
( x + 1)
3

horizontal asymptote. The denominator in


3

x 2 − 5 x − 14 ( x + 2 )( x − 7 ) 1 1
lowest terms is zero at x = − , so x = − is a
The degree of the numerator, p( x) = x3 + 1, is 3 3
n = 3 . The degree of the denominator, vertical asymptote.
q( x) = x 2 − 5 x − 14, is m = 2 . Since n = m + 1 ,
x2 + 6 x + 5 ( x + 5)( x + 1)
there is an oblique asymptote. 50. F ( x ) = 2
=
2 x + 7 x + 5 (2 x + 5)( x + 1)
Dividing:
x+5 5
= , where x ≠ − , −1
2x + 5 2
The degree of the numerator in lowest terms is

329
Copyright © 2015 Pearson Education, Inc.
Chapter 3: Polynomial and Rational Functions

n = 1 . The degree of the denominator in lowest x 4 − 16 ( x 2 + 4)( x 2 − 4)


1 54. F ( x) = =
terms is m = 1 . Since n = m , the line y = is a x2 − 2x x( x − 2)
2 ( x 2 + 4)( x + 2)( x − 2)
horizontal asymptote. The denominator in =
x( x − 2)
5 5
lowest terms is zero at x = − , so x = − is a ( x 2 + 4)( x + 2)
2 2 = , where x ≠ 0, 2
vertical asymptote. x
The degree of the numerator in lowest terms is
6 x 2 + 7 x − 5 (3x + 5)(2 x − 1) n = 3 . The degree of the denominator in lowest
51. R ( x ) = =
3x + 5 3x + 5 terms is m = 1 . Since n ≥ m + 2 , there is no
5 horizontal asymptote or oblique asymptote. The
= 2 x − 1, where x ≠ − denominator in lowest terms is zero at x = 0 , so
3
The degree of the numerator in lowest terms is x = 0 is a vertical asymptote.
n = 1 . The degree of the denominator in lowest
1 1
terms is m = 0 . Since n = m + 1 , there is an 55. a. R ( x) = − 2 ; Using the function y = ,
oblique asymptote. From the simplification x+3 x
shown above, the oblique asymptote is shift the graph horizontally 3 units to the left,
y = 2 x − 1 . The denominator of R(x) in lowest and shift vertically 2 units down.
terms is 1, so there is no vertical asymptote.

8 x 2 − 26 x − 7 (4 x − 1)(2 x + 7)
52. R ( x) = =
4x −1 4x −1
1
= 2 x + 7, where x ≠ −
4
The degree of the numerator in lowest terms is
n = 1 . The degree of the denominator in lowest
terms is m = 0 . Since n = m + 1 , there is an
oblique asymptote. From the simplification b. Domain: { x | x ≠ −3}
shown above, the oblique asymptote is Range: { y | y ≠ −2}
y = 2 x + 7 . The denominator of R(x) in lowest
c. HA: y = −2 . VA: x = −3
terms is 1, so there is no vertical asymptote.
1 1
x 4 − 1 ( x 2 + 1)( x 2 − 1) 56. R ( x) = + 5 ; Using the function y = ,
53. G ( x) = = x −1 x
x2 − x x( x − 1)
shift the graph horizontally 1 unit to the right,
( x 2 + 1)( x + 1)( x − 1) and shift vertically 5 units up.
=
x( x − 1)
( x 2 + 1)( x + 1)
= , where x ≠ 0, 1
x
The degree of the numerator in lowest terms is
n = 3 . The degree of the denominator in lowest
terms is m = 1 . Since n ≥ m + 2 , there is no
horizontal asymptote or oblique asymptote. The
denominator in lowest terms is zero at x = 0 , so
x = 0 is a vertical asymptote.
b. Domain: { x | x ≠ 1}
Range: { y | y ≠ 5}
c. HA: y = 5 . VA: x = 1

330
Copyright © 2015 Pearson Education, Inc.
Section 3.4: Properties of Rational Functions

−2 1 unit to the right, stretch by a factor of 4 and shift


57. R ( x) = 2
+ 3 = −2 +3; vertically 1 unit up.
( x − 1) ( x 1) 2
1
Using the function y = , shift the graph right 1
x2
unit, stretch vertically by a factor of 2, reflect it
about the x-axis and shift vertically 3 units up.

b. Domain: { x | x ≠ 1}
Range: { y | y > 1}
c. HA: y = 1 . VA: x = 1

2 2
b. Domain: { x | x ≠ 1} 60. R ( x ) = 2 − =− + 2 ; Using
2
x + 6x + 9 ( x + 3)2
Range: { y | y < 3}
1
c. HA: y = 3 . VA: x = 1 = −2 +2
( x + 3)2
3 1 1
58. R ( x ) = −1 = 3 −1 ; the function y = , shift the graph horizontally
( x + 2) 2 ( x 2) 2 x2
1 3 units to the left, stretch by a factor of 2, reflect
Using the function y = , shift the graph left 2 about the x-axis and shift vertically 2 units up.
x2
units, stretch vertically by a factor of 3, and shift
vertically 1 unit down.

b. Domain: { x | x ≠ −3}
Range: { y | y < 2}
b. Domain: { x | x ≠ −2} c. HA: y = 2 . VA: x = −3
Range: { y | y > −1}
3.99 × 1014
c. HA: y = −1 . VA: x = −2 61. g ( h ) =
( 6.374 ×10 + h)
6 2

4 4
59. R ( x) = 1 + = + 1 ; Using the 3.99 × 1014
x − 2 x + 1 ( x − 1)2
2
a. g (0) = ≈ 9.8208 m/s 2
( 6.374 ×10 + 0)
6 2

1
=4 +1
( x − 1)2
1
function y = , shift the graph horizontally 1
x2

331
Copyright © 2015 Pearson Education, Inc.
Chapter 3: Polynomial and Rational Functions

3.99 × 1014 10 R2
b. g ( 443) = 63. a. Rtot =
( 6.374 ×10 + 443) 10 + R2
6 2

≈ 9.8195 m/s 2
3.99 × 1014
c. g ( 8848 ) =
( 6.374 ×10 + 8848 )
6 2

≈ 9.7936 m/s 2
3.99 × 1014
d. g (h) =
( 6.374 ×10 + h)
2
6 b. Horizontal asymptote: y = Rtot = 10
As the value of R2 increases without bound,
3.99 × 1014
≈ → 0 as h → ∞ the total resistance approaches 10 ohms, the
h2
resistance of R1 .
Thus, the h-axis is the horizontal asymptote.
R1 R2
3.99 × 1014 c. Rtot =
e. g ( h) = = 0 , to solve this R1 + R2
( 6.374 ×10 + h)
6 2

R1 ⋅ 2 R1
equation would require that 3.99 × 1014 = 0 , 17 =
which is impossible. Therefore, there is no R1 + 2 R1
height above sea level at which g = 0. In Solving graphically, let Y1 = 17 and
other words, there is no point in the entire
universe that is unaffected by the Earth’s
Y2 = 2 x x (x + 2 x) .
gravity! 25

50 (1 + 0.5t )
62. P ( t ) =
2 + 0.01t
50 (1 + 0 ) 50
a. P (0) = = = 25 insects −10 150
2+0 2
−5
b. 5 years = 60 months;
We would need R1 ≈ 103.5 ohms.
50 (1 + 0.5 ( 60 ) ) 1550
P ( 60 ) = =
2 + 0.01 ( 60 ) 2.6 64. a. p(−3) = (−3)3 − 7(−3) − 40 = −46
≈ 596 insects
p (5) = (5)3 − 7(5) − 40 = 50
50 (1 + 0.5t ) 50 ( 0.5t )
c. P (t ) = ≈
= 2500 b. Since p is continuous and p(−3) < 0 < p(5) ,
2 + 0.01t 0.01t
there must be at least one zero in the interval
as t → ∞
Thus, y = 2500 is the horizontal asymptote. ( −3,5) [Intermediate Value Theorem].
The area can sustain a maximum population c. From the problem statement, we find the
of 2500 insects. derivative to be p '( x) = 3 x 2 − 7 .
From part (b), we know there is at least one
real zero in the interval ( −3,5 ) .
p(−3) = (−3)3 − 7(−3) − 40 = −34
Since p(3) < 0 < p (5) , we start with x0 = 4 .

332
Copyright © 2015 Pearson Education, Inc.
Section 3.4: Properties of Rational Functions

p (4)
x1 = 4 − ≈ 4.097560976
p '(4) c. HA: y = 2 . VA: x = 1
p(4.097560976)
x2 = 4.097560976 − 66. a. Dividing:
p '(4.097560976)
−3
≈ 4.094906
2 x − 7 −6 x + 16
p(4.094906)
x3 = 4.094906 − ≈ 4.094904 − ( −6 x + 21)
p '(4.094906)
The zero is approximately x = 4.0949 . -5

d. p ( x) = x3 − 7 x − 40 −5
R ( x) = −3 +
2x − 7

−5
b. R ( x) = −3
2x − 7
−5
= −3
7
2 x−
2
From the graph we can see that there is
exactly one real zero in the interval ( −3,5 ) . −5
c. R ( x) = −3
7
2 x−
p ( 4.0949 ) = ( 4.0949 ) − 7 ( 4.0949 ) − 40
3
e. 2
≈ −0.00017 5 1
=− −3
This result is close to 0. Since 4.0949 is 2 x − 72
rounded, we expect some error when
evaluating the function. d.
65. a. Dividing:
2
x −1 2x + 3
− ( 2 x − 2)
5

5 1
R( x) = 2 + =5 +2
x −1 x −1

b. 7
e. HA: y = −3 . VA: x =
2
67. Answers will vary. If x = 4 is a vertical
asymptote, then x = 4 is a zero of the
denominator. If x = 4 is a zero of a polynomial,
then ( x − 4) is a factor of the polynomial.
Therefore, if x = 4 is a vertical asymptote of a
rational function, then ( x − 4) must be a factor
of the denominator.

333
Copyright © 2015 Pearson Education, Inc.
Chapter 3: Polynomial and Rational Functions

68. Answers will vary. With rational functions, the 73. 2 x3 − xy 2 = 4


only way to get a non-zero horizontal asymptote
Test x-axis symmetry: Let y = − y
is if the degree of the numerator equals the
2 x3 − x ( − y ) = 4
2
degree of the denominator. In such cases, the
horizontal asymptote is the ratio of the leading
coefficients. 2 x3 − xy 2 = 4 same
Test y-axis symmetry: Let x = − x
69. A rational function cannot have both a horizontal
2 ( − x ) − ( − x) y 2 = 4
3
and oblique asymptote. To have an oblique
asymptote, the degree of the numerator must be
−2 x3 + xy 2 = 4 not the same
exactly one larger than the degree of the
denominator. However, if the numerator has a Test origin symmetry: Let x = − x and y = − y
higher degree, there is no horizontal asymptote.
2 ( − x ) − ( − x) ( − y ) = 4
3 2

70. Answers will vary. We want a rational function


−2 x3 + xy 2 = 4 not the same
n ( x)
such that r ( x ) = 2 x + 1 + where n and d Therefore, the graph will have x-axis symmetry.
d ( x)
are polynomial functions and the degree of n ( x ) 74. f ( x) = −3x + 2; g ( x) = x 2 − 2 x − 4

is less than the degree of d ( x ) . We could let −3 x + 2 = x 2 − 2 x − 4


n ( x ) = 1 and d ( x ) = x + 1 . Then our function is 0 = x2 + x − 6
1 0 = ( x − 2)( x + 3)
r ( x) = 2x +1+ . Getting a common
x +1 x = 2 or x = −3
denominator yields f (2) = −3(2) + 2 = −4
r ( x) =
( 2 x + 1)( x + 1) + 1 f ( −3) = −3( −3) + 2 = 11
x +1 x +1
2
2x + x + 2x +1+ 1 So the intersection points are: (2, −4), ( −3,11)
=
x +1
2
2 x + 3x + 2
=
x +1
2
Therefore, one possibility is r ( x ) = 2 x + 3x + 2 .
x +1
71. The equation of a vertical line through the point
(5, −3) is x = 5 .

72.
2 x
(3x − 7) + 1 = − 2
5 4
6 14 x
x − +1 = − 2
5 5 4
6 x 14
x − = −2 − 1 +
5 4 5
19 1
x=−
20 5
1 20 4
x=− =−
5 19 19

334
Copyright © 2015 Pearson Education, Inc.
Section 3.5: The Graph of a Rational Function

Section 3.5 2.
x−3
= −2
1. y-intercept: x2 + 1
x − 3 = −2( x 2 + 1)
02 − 1 −1 1
f (0) = = = x − 3 = −2 x 2 − 2
02 − 4 −4 4
x-intercepts: 2 x2 + x − 1 = 0
Set the numerator equal to 0 and solve for x. (2 x − 1)( x + 1) = 0
x2 − 1 = 0 1
x= or x = −1
2
( x + 1)( x − 1) = 0
 1
x = −1 or x = 1 The solution set is −1,  .
 2
 1
The intercepts are  0,  , (−1, 0) , and (1, 0) . 3. in lowest terms
 4
4. vertical
5. True
6. True

p ( x)
In problems 7–44, we will use the terminology: R ( x) = , where the degree of p( x) = n and the degree of
q ( x)
q( x) = m .
x +1
7. R ( x) = p ( x) = x + 1; q( x) = x( x + 4) = x 2 + 4 x; n = 1; m = 2
x( x + 4)

Step 1: Domain: {x x ≠ − 4, x ≠ 0}
Since 0 is not in the domain, there is no y-intercept.

Step 2 & 3:The function is in lowest terms. The x-intercept is the zero of p ( x) : x = −1 with odd multiplicity.
Plot the point ( −1, 0 ) . The graph will cross the x-axis at this point.

x +1
Step 4: R( x) = is in lowest terms.
x( x + 4)
The vertical asymptotes are the zeros of q( x) : x = − 4 and x = 0 . Plot these lines using dashes. The
multiplicity of 0 and -4 are odd so the graph will approach plus or minus infinity on either side of the
asymptotes.

Step 5: Since n < m , the line y = 0 is the horizontal asymptote. Solve R ( x ) = 0 to find intersection points:
x +1
=0
x( x + 4)
x +1 = 0
x = −1
R ( x) intersects y = 0 at (–1, 0). Plot the point ( −1, 0 ) and the line y = 0 using dashes.

335
Copyright © 2015 Pearson Education, Inc.
Chapter 3: Polynomial and Rational Functions

Step 6:

Steps 7: Graphing

x
8. R ( x) = p ( x) = x; q ( x ) = ( x − 1)( x + 2) = x 2 + x − 2; n = 1; m = 2
( x − 1)( x + 2)

Step 1: Domain: {x x ≠ − 2, x ≠ 1}
The y-intercept; R (0) = 0

Step 2 & 3:The function is in lowest terms. The x-intercept is the zero of p ( x) . 0 with odd multiplicity.
Plot the point ( 0, 0 ) . The graph will cross the x-axis at this point.

x
Step 4: R( x) = is in lowest terms.
( x − 1)( x + 2)
The vertical asymptotes are the zeros of q( x) : x = − 2 and x = 1 . Graph these asymptotes using
dashed lines. The multiplicity of -2 and 1 are both odd so the graph will approach plus or minus
infinity on either side of the asymptotes.
Step 5: Since n < m , the line y = 0 is the horizontal asymptote. Solve to find intersection points:
x
=0
( x − 1)( x + 2)
x=0
R ( x) intersects y = 0 at (0, 0). Plot the point ( 0, 0 ) and the line y = 0 using dashes.

336
Copyright © 2015 Pearson Education, Inc.
Section 3.5: The Graph of a Rational Function

Step 6:

Steps 7: Graphing:

3 x + 3 3( x + 1)
9. R ( x) = = p ( x) = 3 x + 3; q ( x) = 2 x + 4; n = 1; m = 1
2 x + 4 2( x + 2)

Step 1: Domain: {x x ≠ − 2}
3( 0) + 3 3  3
The y-intercept is R (0) = = . Plot the point  0,  .
2 ( 0) + 4 4  4
3 ( x + 1)
Step 2 & 3: R( x) = is in lowest terms. The x-intercept is the zero of p ( x) , x = −1 with odd multiplicity.
2 ( x + 2)
Plot the point ( −1, 0 ) . The graph will cross the x-axis at this point.

3x + 3 3 ( x + 1)
Step 4: R( x) = = is in lowest terms.
2x + 4 2 ( x + 2)
The vertical asymptote is the zero of q( x) : x = − 2 . Graph this asymptote using a dashed line. The
multiplicity of -2 is odd so the graph will approach plus or minus infinity on either side of the
asymptote.
3
Step 5: Since n = m , the line y = is the horizontal asymptote.
2
Solve to find intersection points:

337
Copyright © 2015 Pearson Education, Inc.
Chapter 3: Polynomial and Rational Functions

3x + 3 3
=
2x + 4 2
2 ( 3 x + 3) = 3 ( 2 x + 4 )
6x + 6 = 6x + 4
0≠2
3 3
R ( x) does not intersect y = . Plot the line y = with dashes.
2 2
Step 6:

Steps 7: Graphing:

2x + 4 2 ( x + 2)
10. R ( x) = = p( x) = 2 x + 4; q ( x) = x − 1; n = 1; m = 1
x −1 x −1

Step 1: Domain: {x x ≠ 1}
2(0) + 4 4
The y-intercept is R (0) = = = − 4 . Plot the point ( 0, −4 ) .
0 −1 −1

Step 2 & 3:R is in lowest terms. The x-intercept is the zero of p( x) : x = −2 with odd multiplicity.
Plot the point ( −2, 0 ) . The graph will cross the x-axis at this point.

2x + 4 2 ( x + 2)
Step 4: R( x) = = is in lowest terms.
x −1 x −1
The vertical asymptote is the zero of q( x) : x = 1 . Graph this asymptote using a dashed line. The
multiplicity of 1 is odd so the graph will approach plus or minus infinity on either side of the
asymptote.

338
Copyright © 2015 Pearson Education, Inc.
Section 3.5: The Graph of a Rational Function

Step 5: Since n = m , the line y = 2 is the horizontal asymptote. Solve to find intersection points:
2x + 4
=2
x −1
2 x + 4 = 2 ( x − 1)
2x + 4 = 2x −1
0 ≠ −5
R ( x) does not intersect y = 2 . Plot the line y = 2 with dashes.

Step 6:

Steps 7: Graphing:

3 3
11. R ( x) = = p ( x) = 3; q ( x) = x 2 − 4; n = 0; m = 2
2
x −4 ( x − 2 )( x + 2 )

Step 1: Domain: {x x ≠ − 2, x ≠ 2}
3 3 3  3
The y-intercept is R (0) = 2
= = − . Plot the point  0, −  .
0 −4 −4 4  4

Step 2 & 3:R is in lowest terms. The x-intercepts are the zeros of p( x) . Since p ( x ) is a constant, there are no x-
intercepts.

3
Step 4: R( x) = is in lowest terms. The vertical asymptotes are the zeros of q( x) : x = − 2 and x = 2 .
x2 − 4
Graph each of these asymptotes using dashed lines. The multiplicity of -2 and 2 is odd so the graph
will approach plus or minus infinity on either side of the asymptotes.

339
Copyright © 2015 Pearson Education, Inc.
Chapter 3: Polynomial and Rational Functions

Step 5: Since n < m , the line y = 0 is the horizontal asymptote. Solve to find intersection points:
3
2
=0
x −4
(
3 = 0 x2 − 4 )
3≠0
R ( x) does not intersect y = 0 . Plot the line y = 0 with dashes.

Step 6:

Steps 7: Graphing:

6 6
12. R ( x) = 2
= p ( x) = 6; q ( x) = x 2 − x − 6; n = 0; m = 2
x − x−6 ( x − 3)( x + 2)

Step 1: Domain: {x x ≠ − 2, x ≠ 3}
6 6
The y-intercept is R (0) = 2
= = −1 . Plot the point ( 0, −1) .
0 −0−6 −6

Step 2 & 3:R is in lowest terms. The x-intercepts are the zeros of p( x) . Since p ( x ) is a constant, there are no x-
intercepts.

6
Step 4: R( x) = is in lowest terms. The vertical asymptotes are the zeros of q( x) : x = − 2 and x = 3
x2 − x − 6
Graph each of these asymptotes using dashed lines. The multiplicity of -2 and 3 is odd so the graph
will approach plus or minus infinity on either side of the asymptotes.

340
Copyright © 2015 Pearson Education, Inc.
Section 3.5: The Graph of a Rational Function

Step 5: Since n < m , the line y = 0 is the horizontal asymptote. Solve to find intersection points:
6
2
=0
x − x−6
(
6 = 0 x2 − x − 6 )
6≠0
R ( x) does not intersect y = 0 .

Step 6:

Steps 7: Graphing:

x4 + x2 + 1
13. P ( x) = 2
p ( x) = x 4 + x 2 + 1; q ( x) = x 2 − 1; n = 4; m = 2
x −1

Step 1: Domain: {x x ≠ −1, x ≠ 1}


04 + 02 + 1 1
The y-intercept is P (0) = 2
= = −1 . Plot the point ( 0, −1) .
0 −1 −1

x4 + x2 + 1
Step 2 & 3: P ( x ) = is in lowest terms. The x-intercept is the zero of p ( x) . Since p ( x ) is never 0,
x2 − 1
there are no x-intercepts.

x4 + x2 + 1
Step 4: P( x) = is in lowest terms. The vertical asymptotes are the zeros of q( x) : x = −1 and x = 1 .
x2 − 1
Graph each of these asymptotes using dashed lines. The multiplicity of -1 and 1 is odd so the graph
will approach plus or minus infinity on either side of the asymptotes.

341
Copyright © 2015 Pearson Education, Inc.
Chapter 3: Polynomial and Rational Functions

Step 5: Since n > m + 1 , there is no horizontal or oblique asymptote.

Step 6:

Steps 7: Graphing:

x4 − 1 ( x 2 + 1)( x + 1)( x − 1)
14. Q( x) = = p ( x ) = x 4 − 1; q( x) = x 2 − 4; n = 4; m = 2
x2 − 4 ( x + 2)( x − 2)

Step 1: Domain: {x x ≠ − 2, x ≠ 2}
04 − 1 −1 1  1
The y-intercept is Q(0) = 2
= = . Plot the point  0,  .
0 −4 −4 4  4

( x 2 + 1)( x + 1)( x − 1)
Step 2 & 3: Q( x) = is in lowest terms. The x-intercepts are the zeros of p( x) : –1 and 1 both
( x + 2)( x − 2)
with odd multiplicity.
Plot the points ( −1, 0 ) and (1, 0 ) . The graph crosses the x-axis at both points.

x4 − 1 ( x 2 + 1)( x + 1)( x − 1)
Step 4: Q( x) = 2
= is in lowest terms.
x −4 ( x + 2)( x − 2)
The vertical asymptotes are the zeros of q( x) : x = − 2 and x = 2 . Graph each of these asymptotes
using dashed lines. The multiplicity of -2 and 2 is odd so the graph will approach plus or minus infinity
on either side of the asymptotes.

Step 5: Since n > m + 1 , there is no horizontal asymptote and no oblique asymptote.

342
Copyright © 2015 Pearson Education, Inc.
Section 3.5: The Graph of a Rational Function

Step 6:

Steps 7: Graphing:

x3 − 1 ( x − 1) ( x 2 + x + 1)
15. H ( x) = = p( x) = x3 − 1; q( x) = x 2 − 9; n = 3; m = 2
x2 − 9 ( x + 3)( x − 3)
Step 1: Domain: {x x ≠ − 3, x ≠ 3}
03 − 1
−1 1  1
The y-intercept is H (0) = = = . Plot the point  0,  .
02 − 9 − 9 9  9
Step 2 & 3: H ( x) is in lowest terms. The x-intercept is the zero of p ( x) : 1 with odd multiplicity.
Plot the point (1, 0 ) . The graph will cross the x-axis at this point.
Step 4: H ( x) is in lowest terms. The vertical asymptotes are the zeros of q( x) : x = − 3 and x = 3 . Graph
each of these asymptotes using dashed lines. The multiplicity of -3 and 3 is odd so the graph will
approach plus or minus infinity on either side of the asymptotes.
Step 5: Since n = m + 1 , there is an oblique asymptote. Dividing:
x
9x −1
x − 9 x3 + 0 x 2 + 0 x − 1
2
H ( x) = x + 2
x −9
x3 − 9x
9x −1
The oblique asymptote is y = x . Graph the asymptote with a dashed line. Solve to find intersection
points:

343
Copyright © 2015 Pearson Education, Inc.
Chapter 3: Polynomial and Rational Functions

x3 − 1
=x
x2 − 9
x3 − 1 = x3 − 9 x
− 1 = −9 x
1
x=
9
1 1
The oblique asymptote intersects H ( x) at  ,  .
9 9
Step 6:

Steps 7: Graphing:

x3 + 1 ( x + 1)( x 2 − x + 1)
16. G ( x) = 2
= p ( x) = x3 + 1; q ( x) = x 2 + 2 x; n = 3; m = 2
x + 2x x( x + 2)

Step 1: Domain: {x x ≠ − 2, x ≠ 0}
03 + 1 1
There is no y-intercept since G (0) = 2
= .
0 + 2(0) 0

x3 + 1
Step 2 & 3: G ( x) = is in lowest terms. The x-intercept is the zero of p( x) : –1 with odd multiplicity.
x2 + 2x
Plot the point ( −1, 0 ) . The graph will cross the x-axis at this point.

x3 + 1
Step 4: G ( x) = is in lowest terms. The vertical asymptotes are the zeros of q( x) : x = − 2 and x = 0 .
x2 + 2x
Graph each of these asymptotes using dashed lines. The multiplicity of -2 and 0 is odd so the graph
will approach plus or minus infinity on either side of the asymptotes.

344
Copyright © 2015 Pearson Education, Inc.
Section 3.5: The Graph of a Rational Function

Step 5: Since n = m + 1 , there is an oblique asymptote. Dividing:


x−2
4x + 1
x + 2x x + 0x2 + 0 x + 1
2 3
G ( x) = x − 2 + 2
x + 2x
x3 + 2 x 2
− 2x2 +1
2
− 2x − 4x
4x +1
The oblique asymptote is y = x − 2 . Graph this asymptote with a dashed line. Solve to find
intersection points:
x3 + 1
= x−2
x2 + 2 x
x3 + 1 = x3 − 4 x
1 = − 4x
1
x=−
4
 1 9
The oblique asymptote intersects G ( x) at  − , −  .
 4 4

Step 6:

Steps 7: Graphing:

345
Copyright © 2015 Pearson Education, Inc.
Chapter 3: Polynomial and Rational Functions

x2 x2
17. R ( x) = = p( x) = x 2 ; q ( x) = x 2 + x − 6; n = 2; m = 2
x2 + x − 6 ( x + 3)( x − 2)

Step 1: Domain: {x x ≠ − 3, x ≠ 2}
02 0
The y-intercept is R (0) = 2
= = 0 . Plot the point ( 0, 0 ) .
0 +0−6 −6

x2
Step 2 & 3: R ( x) = is in lowest terms. The x-intercept is the zero of p ( x) : 0 with even multiplicity.
x2 + x − 6
Plot the point ( 0, 0 ) . The graph will touch the x-axis at this point.

x2
Step 4: R( x) = is in lowest terms. The vertical asymptotes are the zeros of q( x) :
x2 + x − 6
x = − 3 and x = 2 . Graph each of these asymptotes using dashed lines. The multiplicity of -3 and 2 is
odd so the graph will approach plus or minus infinity on either side of the asymptotes.

Step 5: Since n = m , the line y = 1 is the horizontal asymptote. Graph this asymptote with a dashed line.
Solve to find intersection points:
x2
=1
x2 + x − 6
x2 = x2 + x − 6
0 = x−6
x=6
R ( x) intersects y = 1 at (6, 1).

Step 6:

346
Copyright © 2015 Pearson Education, Inc.
Section 3.5: The Graph of a Rational Function

Steps 7: Graphing:

x 2 + x − 12 ( x + 4)( x − 3)
18. R ( x) = 2
= p ( x) = x 2 + x − 12; q( x) = x 2 − 4; n = 2; m = 2
x −4 ( x + 2)( x − 2)

Step 1: Domain: {x x ≠ − 2, x ≠ 2}
02 + 0 − 12 −12
The y-intercept is R (0) = = = 3 . Plot the point ( 0,3) .
02 − 4 −4

( x + 4)( x − 3)
Step 2 & 3: R ( x ) = is in lowest terms. The x-intercepts are the zeros of p ( x) : –4 and 3 each with
( x + 2)( x − 2)
odd multiplicity.
Plot the points ( −4, 0 ) and ( 3, 0 ) . The graph will cross the x-axis at these point.

x 2 + x − 12
Step 4: R( x) = is in lowest terms.
x2 − 4
The vertical asymptotes are the zeros of q( x) : x = − 2 and x = 2 . Graph each of these asymptotes
using a dashed line. The multiplicity of -2 and 2 is odd so the graph will approach plus or minus
infinity on either side of the asymptotes.

Step 5: Since n = m , the line y = 1 is the horizontal asymptote. Graph this asymptote using a dashed line.
Solve to find intersection points:
x 2 + x − 12
=1
x2 − 4
x 2 + x − 12 = x 2 − 4
x =8
R ( x) intersects y = 1 at (8, 1).

347
Copyright © 2015 Pearson Education, Inc.
Chapter 3: Polynomial and Rational Functions

Step 6:

Steps 7: Graphing:

x x
19. G ( x) = = p( x) = x; q ( x) = x 2 − 4; n = 1; m = 2
x2 − 4 ( x + 2)( x − 2)

Step 1: Domain: {x x ≠ − 2, x ≠ 2}
0 0
The y-intercept is G (0) = 2
= = 0 . Plot the point ( 0, 0 ) .
0 −4 −4

x
Step 2 & 3: G ( x) = 2
is in lowest terms. The x-intercept is the zero of p( x) : 0 with odd multiplicity.
x −4
Plot the point ( 0, 0 ) . The graph will cross the x-axis at this point.

x
Step 4: G ( x) = is in lowest terms. The vertical asymptotes are the zeros of q( x) : x = − 2 and x = 2 .
2
x −4
Graph each of these asymptotes using a dashed line. The multiplicity of -2 and 2 is odd so the graph
will approach plus or minus infinity on either side of the asymptote.

Step 5: Since n < m , the line y = 0 is the horizontal asymptote. Graph this asymptote using a dashed line.
Solve to find intersection points:
x
=0
x2 − 4
x=0
G ( x) intersects y = 0 at (0, 0).

348
Copyright © 2015 Pearson Education, Inc.
Section 3.5: The Graph of a Rational Function

Step 6:

Steps 7: Graphing:

3x 3x
20. G ( x) = = p( x) = 3 x; q( x) = x 2 − 1; n = 1; m = 2
x2 −1 ( x + 1)( x − 1)

Step 1: Domain: {x x ≠ −1, x ≠ 1}


3(0) 0
The y-intercept is G (0) = 2
= = 0 . Plot the point ( 0, 0 ) .
0 −1 −1

3x
Step 2 & 3: G ( x) = 2
is in lowest terms. The x-intercept is the zero of p ( x) : 0 with odd multiplicity.
x −1
Plot the point ( 0, 0 ) . The graph will cross the x-axis at this point.

3x
Step 4: G ( x) = is in lowest terms. The vertical asymptotes are the zeros of q( x) : x = −1 and x = 1
2
x −1
Graph each of these asymptotes using a dashed line. The multiplicity of -1 and 1 is odd so the graph
will approach plus or minus infinity on either side of the asymptotes.

Step 5: Since n < m , the line y = 0 is the horizontal asymptote. Graph this asymptote using a dashed line.
Solve to find intersection points:
3x
=0
x2 − 1
3x = 0
x=0
G ( x) intersects y = 0 at (0, 0).

349
Copyright © 2015 Pearson Education, Inc.
Chapter 3: Polynomial and Rational Functions

Step 6:

Steps 7: Graphing:

3 3
21. R ( x) = 2
= p( x) = 3; q( x) = ( x − 1)( x 2 − 4); n = 0; m = 3
( x − 1)( x − 4) ( x − 1)( x + 2)( x − 2)

Step 1: Domain: {x x ≠ − 2, x ≠ 1, x ≠ 2}
3 3  3
The y-intercept is R (0) = 2
= . Plot the point  0,  .
(0 − 1)(0 − 4) 4  4

3
Step 2 & 3: R ( x ) = is in lowest terms. There is no x-intercept.
( x − 1)( x 2 − 4)

3
Step 4: R( x) = is in lowest terms.
( x − 1)( x 2 − 4)
The vertical asymptotes are the zeros of q( x) : x = − 2, x = 1, and x = 2 .
Graph each of these asymptotes using a dashed line. The multiplicity of -2, 1 and 2 is odd so the graph
will approach plus or minus infinity on either side of the asymptotes.

Step 5: Since n < m , the line y = 0 is the horizontal asymptote. Graph this asymptote with a dashed line.
Solve to find intersection points:
3
=0
( x − 1)( x 2 − 4)
3≠0
R ( x) does not intersect y = 0 .

350
Copyright © 2015 Pearson Education, Inc.
Section 3.5: The Graph of a Rational Function

Step 6:

Steps 7: Graphing:

−4 −4
22. R ( x) = 2
= p ( x) = − 4; q ( x) = ( x + 1)( x 2 − 9); n = 0; m = 3
( x + 1)( x − 9) ( x + 1)( x + 3)( x − 3)

Step 1: Domain: {x x ≠ − 3, x ≠ −1, x ≠ 3}


−4 −4 4  4
The y-intercept is R (0) = 2
= = . Plot the point  0,  .
(0 + 1)(0 − 9) −9 9  9

−4
Step 2 & 3: R ( x ) = is in lowest terms. There is no x-intercept.
( x + 1)( x 2 − 9)

−4
Step 4: R( x) = is in lowest terms.
( x + 1)( x 2 − 9)
The vertical asymptotes are the zeros of q( x) : x = − 3, x = −1, and x = 3
Graph each of these asymptotes using a dashed line. The multiplicity of -3, -1 and 3 is odd so the graph
will approach plus or minus infinity on either side of the asymptotes.

Step 5: Since n < m , the line y = 0 is the horizontal asymptote. Graph this asymptote with a dashed line.
Solve to find intersection points:
−4
=0
( x + 1)( x 2 − 9)
−4 ≠ 0
R ( x) does not intersect y = 0 .

351
Copyright © 2015 Pearson Education, Inc.
Chapter 3: Polynomial and Rational Functions

Step 6:

Steps 7: Graphing:

x2 − 1 ( x − 1)( x + 1)
23. H ( x) = 4
= 2 p ( x) = x 2 − 1; q ( x) = x 4 − 16; n = 2; m = 4
x − 16 ( x + 4)( x + 2)( x − 2)

Step 1: Domain: {x x ≠ − 2, x ≠ 2}
02 − 1 −1 1  1
The y-intercept is H (0) = 4
= = . Plot the point  0,  .
0 − 16 −16 16  16 
2
x −1
Step 2 & 3: H ( x) = is in lowest terms. The x-intercepts are the zeros of p( x) : –1 and 1 each with odd
x 4 − 16
multiplicity.
Plot ( −1, 0 ) and (1, 0 ) . The graph will cross the x-axis at these points.

x2 − 1
Step 4: H ( x) = is in lowest terms. The vertical asymptotes are the zeros of q( x) : x = − 2 and x = 2
x 4 − 16
Graph each of these asymptotes using a dashed line. The multiplicity of -2 and 2 is odd so the graph
will approach plus or minus infinity on either side of the asymptotes.

Step 5: Since n < m , the line y = 0 is the horizontal asymptote. Graph this asymptote using a dashed line.
Solve to find intersection points:
x2 − 1
=0
x 4 − 16
x2 − 1 = 0
x = ±1
H ( x) intersects y = 0 at (–1, 0) and (1, 0).

352
Copyright © 2015 Pearson Education, Inc.
Section 3.5: The Graph of a Rational Function

Step 6:

Steps 7: Graphing:

x2 + 4 x2 + 4
24. H ( x) = = p ( x) = x 2 + 4; q( x) = x 4 − 1; n = 2; m = 4
x4 − 1 ( x 2 + 1)( x + 1)( x − 1)

Step 1: Domain: {x x ≠ −1, x ≠ 1}


02 + 4 4
The y-intercept is H (0) = 4
= = − 4 . Plot the point ( 0, −4 ) .
0 −1 −1

x2 + 4
Step 2 & 3: H ( x) = is in lowest terms. There are no x-intercepts.
x4 − 1

x2 + 4
Step 4: H ( x) = is in lowest terms. The vertical asymptotes are the zeros of q( x) : x = −1 and x = 1
x4 − 1
Graph each of these asymptotes using a dashed line. The multiplicity of -1 and 1 is odd so the graph
will approach plus or minus infinity on either side of the asymptotes.

Step 5: Since n < m , the line y = 0 is the horizontal asymptote. Graph this asymptote using a dashed line.
Solve to find intersection points:
x2 + 4
=0
x4 − 1
x2 + 4 = 0
no real solution
H ( x) does not intersect y = 0 .

353
Copyright © 2015 Pearson Education, Inc.
Chapter 3: Polynomial and Rational Functions

Step 6:

Steps 7: Graphing:

x 2 − 3x − 4 ( x + 1)( x − 4)
25. F ( x) = = p( x) = x 2 − 3 x − 4; q( x) = x + 2; n = 2; m = 1
x+2 x+2

Step 1: Domain: {x x ≠ − 2}
02 − 3(0) − 4 − 4
The y-intercept is F (0) = = = − 2 . Plot the point ( 0, −2 ) .
0+2 2

x 2 − 3x − 4
Step 2 & 3: F ( x) = is in lowest terms. The x-intercepts are the zeros of p ( x) : –1 and 4 each with odd
x+2
multiplicity.
Plot ( −1, 0 ) and ( 4, 0 ) . The graph will cross the x-axis at these points.

x 2 − 3x − 4
Step 4: F ( x) = is in lowest terms. The vertical asymptote is the zero of q( x) : x = − 2
x+2
Graph this asymptote using a dashed line. The multiplicity of -2 is odd so the graph will approach plus
or minus infinity on either side of the asymptote.

354
Copyright © 2015 Pearson Education, Inc.
Section 3.5: The Graph of a Rational Function

Step 5: Since n = m + 1 , there is an oblique asymptote. Dividing:


x−5
2 6
x + 2 x − 3x − 4 F ( x) = x − 5 +
x+2
x2 + 2 x
− 5x − 4
− 5 x − 10
6
The oblique asymptote is y = x − 5 . Graph this asymptote using a dashed line. Solve to find
intersection points:
x 2 − 3x − 4
= x −5
x+2
x 2 − 3x − 4 = x 2 − 3 x − 10
− 4 ≠ −10
The oblique asymptote does not intersect F ( x) .

Step 6:

Steps 7: Graphing:

x 2 + 3 x + 2 ( x + 2)( x + 1)
26. F ( x) = = p ( x) = x 2 + 3 x + 2; q ( x) = x − 1; n = 2; m = 1
x −1 x −1

Step 1: Domain: {x x ≠ 1}
02 + 3(0) + 2 2
The y-intercept is F (0) = = = − 2 . Plot the point ( 0, −2 ) .
0 −1 −1

355
Copyright © 2015 Pearson Education, Inc.
Chapter 3: Polynomial and Rational Functions

x2 + 3x + 2
Step 2 & 3: F ( x) = is in lowest terms. The x-intercepts are the zeros of p( x) : –2 and –1 each with
x −1
odd multiplicity.
Plot ( −2, 0 ) and ( −1, 0 ) . The graph will cross the x-axis at these points.

x2 + 3x + 2
Step 4: F ( x) = is in lowest terms. The vertical asymptote is the zero of q( x) : x = 1
x −1
Graph this asymptote using a dashed line. The multiplicity of 1 is odd so the graph will approach plus
or minus infinity on either side of the asymptote.

Step 5: Since n = m + 1 , there is an oblique asymptote. Dividing:


x+4
6
x − 1 x 2 + 3x + 2 F ( x) = x + 4 +
x −1
x2 − x
4x + 2
4x − 4
6
The oblique asymptote is y = x + 4 . Graph this asymptote using a dashed line. Solve to find
intersection points:
x2 + 3x + 2
= x+4
x −1
x2 + 3x + 2 = x 2 + 3x − 4
2 ≠ −4
The oblique asymptote does not intersect F ( x ) .

Step 6:

356
Copyright © 2015 Pearson Education, Inc.
Section 3.5: The Graph of a Rational Function

Steps 7: Graphing:

x 2 + x − 12 ( x + 4)( x − 3)
27. R ( x) = = p( x) = x 2 + x − 12; q( x) = x − 4; n = 2; m = 1
x−4 x−4

Step 1: Domain: {x x ≠ 4}
02 + 0 − 12 − 12
The y-intercept is R (0) = = = 3 . Plot the point ( 0,3) .
0−4 −4

x 2 + x − 12
Step 2 & 3: R ( x) = is in lowest terms. The x-intercepts are the zeros of p( x) : –4 and 3 each with odd
x−4
multiplicity .
Plot ( −4, 0 ) and ( 3, 0 ) . The graph will cross the x-axis at these points.

x 2 + x − 12
Step 4: R( x) = is in lowest terms. The vertical asymptote is the zero of q( x) : x = 4
x−4
Graph this asymptote using a dashed line. The multiplicity of 4 is odd so the graph will approach plus
or minus infinity on either side of the asymptote.

Step 5: Since n = m + 1 , there is an oblique asymptote. Dividing:


x+5
8
x − 4 x 2 + x − 12 R( x) = x + 5 +
x−4
x2 − 4 x
5 x − 12
5 x − 20
8
The oblique asymptote is y = x + 5 . Graph this asymptote using a dashed line. Solve to find
intersection points:
x 2 + x − 12
= x+5
x−4
x 2 + x − 12 = x 2 + x − 20
−12 ≠ − 20
The oblique asymptote does not intersect R ( x) .

357
Copyright © 2015 Pearson Education, Inc.
Chapter 3: Polynomial and Rational Functions

Step 6:

Steps 7: Graphing:

x 2 − x − 12 ( x − 4)( x + 3)
28. R ( x) = = p( x) = x 2 − x − 12; q ( x) = x + 5; n = 2; m = 1
x+5 x+5

Step 1: Domain: {x x ≠ −5}


02 − 0 − 12 12  12 
The y-intercept is R (0) = =− . Plot the point  0, −  .
0+5 5  5

x 2 − x − 12
Step 2 & 3: R ( x ) = is in lowest terms. The x-intercepts are the zeros of p( x) : –3 and 4 each with odd
x+5
multiplicity.
Plot ( −3, 0 ) and ( 4, 0 ) . The graph will cross the x-axis at these points.

x 2 − x − 12
Step 4: R( x) = is in lowest terms. The vertical asymptote is the zero of q( x) : x = −5
x+5
Graph this asymptote using a dashed line. The multiplicity of -5 is odd so the graph will approach plus
or minus infinity on either side of the asymptote.

358
Copyright © 2015 Pearson Education, Inc.
Section 3.5: The Graph of a Rational Function

Step 5: Since n = m + 1 , there is an oblique asymptote. Dividing:


x−6
2 18
x + 5 x − x − 12 R( x) = x − 6 +
x+5
x2 + 5x
− 6 x − 12
− 6 x − 30
18
The oblique asymptote is y = x − 6 . Graph this asymptote using a dashed line. Solve to find
intersection points:
x 2 − x − 12
= x−6
x+5
x 2 − x − 12 = x 2 − x − 30
−12 ≠ − 30
The oblique asymptote does not intersect R ( x) .

Step 6:

Steps 7: Graphing:

x 2 + x − 12 ( x + 4)( x − 3)
29. F ( x ) = = p( x) = x 2 + x − 12; q( x) = x + 2; n = 2; m = 1
x+2 x+2

Step 1: Domain: {x x ≠ − 2}
02 + 0 − 12 − 12
The y-intercept is F (0) = = = − 6 . Plot the point ( 0, −6 ) .
0+2 2

359
Copyright © 2015 Pearson Education, Inc.
Chapter 3: Polynomial and Rational Functions

x 2 + x − 12
Step 2 & 3: F ( x ) = is in lowest terms. The x-intercepts are the zeros of p( x) : –4 and 3 each with odd
x+2
multiplicity.
Plot ( −4, 0 ) and ( 3, 0 ) . The graph will cross the x-axis at these points.

x 2 + x − 12
Step 4: F ( x) = is in lowest terms. The vertical asymptote is the zero of q( x) : x = − 2
x+2
Graph this asymptote using a dashed line. The multiplicity of -2 is odd so the graph will approach plus
or minus infinity on either side of the asymptote.

Step 5: Since n = m + 1 , there is an oblique asymptote. Dividing:


x −1
−10
x + 2 x 2 + x − 12 F ( x) = x − 1 +
x+2
x2 + 2 x
− x − 12
−x− 2
− 10
The oblique asymptote is y = x − 1 . Graph this asymptote using a dashed line. Solve to find
intersection points:
x 2 + x − 12
= x −1
x+2
x 2 + x − 12 = x 2 + x − 2
−12 ≠ − 2
The oblique asymptote does not intersect F ( x ) .

Step 6:

360
Copyright © 2015 Pearson Education, Inc.
Section 3.5: The Graph of a Rational Function

Steps 7: Graphing:

x 2 − x − 12 ( x + 3)( x − 4)
30. G ( x) = = p ( x ) = x 2 − x − 12; q( x) = x + 1; n = 2; m = 1
x +1 x +1

Step 1: Domain: {x x ≠ −1}


02 − 0 − 12 − 12
The y-intercept is F (0) = = = −12 . Plot the point ( 0, −12 ) .
0 +1 1

x 2 − x − 12
Step 2 & 3: G ( x) = is in lowest terms. The x-intercepts are the zeros of p( x) : –3 and 4 each with odd
x +1
multiplicity.
Plot ( −3, 0 ) and ( 4, 0 ) . The graph will cross the x-axis at these points.

x 2 − x − 12
Step 4: G ( x) = is in lowest terms. The vertical asymptote is the zero of q( x) : x = − 1
x +1
Graph this asymptote using a dashed line. The multiplicity of -1 is odd so the graph will approach plus
or minus infinity on either side of the asymptote.

Step 5: Since n = m + 1 , there is an oblique asymptote. Dividing:


x−2
2 −10
x + 1 x − x − 12 G ( x) = x − 2 +
x +1
x2 + x
− 2 x − 12
− 2x − 2
− 10
The oblique asymptote is y =x − 2. Graph this asymptote using a dashed line. Solve to find
intersection points:
x 2 − x − 12
= x−2
x +1
x 2 − x − 12 = x 2 − x − 2
−12 ≠ − 2
The oblique asymptote does not intersect G ( x) .

361
Copyright © 2015 Pearson Education, Inc.
Chapter 3: Polynomial and Rational Functions

Step 6:

Steps 7: Graphing:

x( x − 1) 2
31. R ( x) = p ( x) = x( x − 1) 2 ; q( x) = ( x + 3)3 ; n = 3; m = 3
( x + 3)3

Step 1: Domain: {x x ≠ − 3}
0(0 − 1) 2 0
The y-intercept is R (0) = 3
= = 0 . Plot the point ( 0, 0 ) .
(0 + 3) 27

x( x − 1) 2
Step 2 & 3: R ( x) = is in lowest terms. The x-intercepts are the zeros of p( x) : 0 with odd multiplicity
( x + 3)3
and 1 with even multiplicity.
Plot ( 0, 0 ) . The graph will cross the x-axis at this point.
Plot (1, 0 ) . The graph will touch the x-axis at this point.

x( x − 1) 2
Step 4: R( x) = is in lowest terms. The vertical asymptote is the zero of q( x) : x = − 3
( x + 3)3
Graph this asymptote with a dashed line. The multiplicity of -2 is odd so the graph will approach plus
or minus infinity on either side of the asymptote.

Step 5: Since n = m , the line y = 1 is the horizontal asymptote. Graph this asymptote with a dashed line.
Solve to find intersection points:

362
Copyright © 2015 Pearson Education, Inc.
Section 3.5: The Graph of a Rational Function

x( x − 1)2
=1
( x + 3)3
x3 − 2 x 2 + x = x3 + 9 x 2 + 27 x + 27
0 = 11x 2 + 26 x + 27
b 2 − 4ac = 262 − 4 (11)( 27 ) = −512
no real solution
R ( x) does not intersect y = 1 .

Step 6:

Steps 7: Graphing:

( x − 1)( x + 2)( x − 3)
32. R ( x) = p( x) = ( x − 1)( x + 2)( x − 3); q ( x) = x( x − 4) 2 ; n = 3; m = 3
x ( x − 4) 2

Step 1: Domain: {x x ≠ 0, x ≠ 4}
(0 − 1)(0 + 2)(0 − 3) 6
There is no y-intercept since R (0) = 2
= .
0(0 − 4) 0

( x − 1)( x + 2)( x − 3)
Step 2 & 3: R ( x) = is in lowest terms. The x-intercepts are the zeros of p( x) : –2, 1, and 3
x( x − 4) 2
each with odd multiplicity.
Plot ( −2, 0 ) , (1, 0 ) and ( 3, 0 ) . The graph will cross the x-axis at these points.

363
Copyright © 2015 Pearson Education, Inc.
Chapter 3: Polynomial and Rational Functions

( x − 1)( x + 2)( x − 3)
Step 4: R( x) = is in lowest terms.
x( x − 4) 2
The vertical asymptotes are the zeros of q( x) : x = 0 and x = 4
Graph each of these asymptotes with a dashed line. The multiplicity of 0 is odd so the graph will
approach plus or minus infinity on either side of the asymptote. The multiplicity of 4 is even so the
graph will approach the same infinity on either side of the asymptote.

Step 5: Since n = m , the line y = 1 is the horizontal asymptote. Graph this asymptote with a dashed line.
Solve to find intersection points:
( x − 1)( x + 2)( x − 3)
=1
x( x − 4) 2
( x 2 + x − 2)( x − 3) = x( x 2 − 8 x + 16)
x3 − 2 x 2 − 5 x + 6 = x3 − 8 x 2 + 16 x
6 x 2 − 21x + 6 = 0
2 x2 − 7 x + 2 = 0
7 ± 49 − 4(2)(2) 7 ± 33
x= =
2(2) 4
 7 − 33   7 + 33 
R( x) intersects y = 1 at  , 1 and  , 1 .
 4   4 

Step 6:

Steps 7: Graphing:

364
Copyright © 2015 Pearson Education, Inc.
Section 3.5: The Graph of a Rational Function

x 2 + x − 12 ( x + 4)( x − 3) x + 4
33. R( x) = = = p ( x) = x 2 + x − 12; q( x) = x 2 − x − 6; n = 2; m = 2
x 2 − x − 6 ( x − 3)( x + 2) x + 2

Step 1: Domain: {x x ≠ − 2, x ≠ 3}
02 + 0 − 12 −12
The y-intercept is R(0) = 2
= = 2 . Plot the point ( 0, 2 ) .
0 −0−6 −6

x+4
Step 2 & 3:In lowest terms, R( x) = , x ≠ 3 . Note: R ( x ) is still undefined at both 3 and −2 .
x+2
The x-intercept is the zero of y = x + 4 : –4 with odd multiplicity.
Plot ( −4, 0 ) . The graph will cross the x-axis at this point.

x+4
Step 4: In lowest terms, R( x) = , x ≠ 3 . The vertical asymptote is the zero of f ( x ) = x + 2 : x = − 2 ;
x+2
Graph this asymptote using a dashed line. The multiplicity of -2 is odd so the graph will approach plus
or minus infinity on either side of the asymptote. Note: x = 3 is not a vertical asymptote because the
 7
reduced form must be used to find the asymptotes. The graph has a hole at  3,  .
 5

Step 5: Since n = m , the line y = 1 is the horizontal asymptote. Graph this asymptote using a dashed line.
Solve to find intersection points:
x 2 + x − 12
=1
x2 − x − 6
x 2 + x − 12 = x 2 − x − 6
2x = 6
x=3
R( x) does not intersect y = 1 because R( x) is not defined at x = 3 .

Step 6:

365
Copyright © 2015 Pearson Education, Inc.
Chapter 3: Polynomial and Rational Functions

Steps 7: Graphing:

x 2 + 3 x − 10 ( x + 5)( x − 2) x − 2
34. R( x) = = = p( x) = x 2 + 3x − 10; q( x) = x 2 + 8 x + 15; n = 2; m = 2
x 2 + 8 x + 15 ( x + 5)( x + 3) x + 3

Step 1: Domain: {x x ≠ − 5, x ≠ −3}


02 + 3(0) − 10 −10 2  2
The y-intercept is R(0) = 2
= = − . Plot the point  0, −  .
0 + 8(0) + 15 15 3  3

x−2
Step 2 & 3:In lowest terms, R( x) = , x ≠ −5 . The x-intercept is the zero of y = x − 2 : 2 with odd
x+3
multiplicity.
Note: –5 is not a zero because reduced form must be used to find the zeros.
Plot the point ( 2,0 ) . The graph will cross the x-axis at this point.
x−2
Step 4: In lowest terms, R( x) = , x ≠ −5 . The vertical asymptote is the zero of f ( x ) = x + 3 : x = − 3 ;
x+3
Graph this asymptote using a dashed line. The multiplicity of -3 is odd so the graph will approach plus
or minus infinity on either side of the asymptote.
Note: x = −5 is not a vertical asymptote because reduced form must be used to find the asymptotes.
The graph has a hole at ( −5, 3.5 ) .

Step 5: Since n = m , the line y = 1 is the horizontal asymptote. Graph this asymptote using a dashed line.
Solve to find intersection points:
x 2 + 3 x − 10
=1
x 2 + 8 x + 15
x 2 + 3 x − 10 = x 2 + 8 x + 15
−5 x = 25
x = −5
R( x) does not intersect y = 1 because R( x) is not defined at x = −5 .

366
Copyright © 2015 Pearson Education, Inc.
Section 3.5: The Graph of a Rational Function

Step 6:

Steps 7: Graphing:

6 x 2 − 7 x − 3 (3 x + 1)(2 x − 3) 3x + 1
35. R( x) = = = p ( x) = 6 x 2 − 7 x − 3; q( x) = 2 x 2 − 7 x + 6; n = 2; m = 2
2 x 2 − 7 x + 6 (2 x − 3)( x − 2) x−2

 3 
Step 1: Domain:  x x ≠ , x ≠ 2 
 2 
6(0) 2 − 7(0) − 3 −3 1  1
The y-intercept is R (0) = 2
= = − . Plot the point  0, −  .
2(0) − 7(0) + 6 6 2  2

3x + 1 3 1
Step 2 & 3:In lowest terms, R( x) = , x ≠ . The x-intercept is the zero of y = 3 x + 1 : − with odd
x−2 2 3
multiplicity.
3
Note: x = is not a zero because reduced form must be used to find the zeros.
2
 1 
Plot the point  − , 0  . The graph will cross the x-axis at this point.
 3 

3x + 1 3
Step 4: In lowest terms, R( x) = , x ≠ . The vertical asymptote is the zero of f ( x ) = x − 2 : x = 2 ;
x−2 2
Graph this asymptote using a dashed line. The multiplicity of 2 is odd so the graph will approach plus
or minus infinity on either side of the asymptote.
3
Note: x = is not a vertical asymptote because reduced form must be used to find the asymptotes.
2
3 
The graph has a hole at  , −11 .
2 

367
Copyright © 2015 Pearson Education, Inc.
Chapter 3: Polynomial and Rational Functions

Step 5: Since n = m , the line y = 3 is the horizontal asymptote. Graph this asymptote using a dashed line.
Solve to find intersection points:
6 x2 − 7 x − 3
=3
2 x2 − 7 x + 6
6 x 2 − 7 x − 3 = 6 x 2 − 21x + 18
14 x = 21
3
x=
2
3
R( x) does not intersect y = 3 because R( x) is not defined at x = .
2

Step 6:

Steps 7: Graphing:

8 x 2 + 26 x + 15 (4 x + 3)(2 x + 5) 4 x + 3
36. R ( x) = 2
= = p ( x) = 8 x 2 + 26 x + 15; q( x) = 2 x 2 − x − 15; n = 2; m = 2
2 x − x − 15 (2 x + 5)( x − 3) x−3

 5 
Step 1: Domain:  x x ≠ − , x ≠ 3
 2 
2
8 ( 0 ) + 26 ( 0 ) + 15 15
The y-intercept is R(0) = = = −1 . Plot the point ( 0, −1) .
2 ( 0 ) − 0 − 15
2
−15

4x + 3 5 3
Step 2 & 3:In lowest terms, R( x) = , x ≠ − . The x-intercept is the zero of y = 4 x + 3 : − with odd
x −3 2 4
multiplicity.

368
Copyright © 2015 Pearson Education, Inc.
Section 3.5: The Graph of a Rational Function

5
Note: − is not a zero because reduced form must be used to find the zeros.
2
 3 
Plot the point  − , 0  . The graph will cross the x-axis at this point.
 4 

4x + 3 5
Step 4: In lowest terms, R ( x) = , x ≠ − . The vertical asymptote is the zero of f ( x ) = x − 3 : x = 3 ;
x −3 2
Graph this asymptote using a dashed line. The multiplicity of 3 is odd so the graph will approach plus
or minus infinity on either side of the asymptote.
5
Note: x = − is not a vertical asymptote because reduced form must be used to find the asymptotes.
2
 5 14 
The graph has a hole at  − ,  .
 2 11 

Step 5: Since n = m , the line y = 4 is the horizontal asymptote. Graph this asymptote using a dashed line.
Solve to find intersection points:
8 x 2 + 26 x + 15
=4
2 x 2 − x − 15
8 x 2 + 26 x + 15 = 8 x 2 − 4 x − 60
30 x = −75
5
x=−
2
5
R( x) does not intersect y = 4 because R( x) is not defined at x = − .
2

Step 6:

Steps 7: Graphing:

369
Copyright © 2015 Pearson Education, Inc.
Chapter 3: Polynomial and Rational Functions

x 2 + 5 x + 6 ( x + 2)( x + 3)
37. R( x) = = = x+2 p( x) = x 2 + 5 x + 6; q( x) = x + 3; n = 2; m = 1
x+3 x+3

Step 1: Domain: {x x ≠ −3}


02 + 5(0) + 6 6
The y-intercept is R(0) = = = 2 . Plot the point ( 0, 2 ) .
0+3 3

Step 2 & 3:In lowest terms, R( x) = x + 2, x ≠ −3 . The x-intercept is the zero of y = x + 2 : –2 with odd
multiplicity.
Note: –3 is not a zero because reduced form must be used to find the zeros.
Plot the point ( 0, −2 ) . The graph will cross the x-axis at this point.
Step 4:In lowest terms, R( x) = x + 2, x ≠ −3 . There are no vertical asymptotes. Note: x = −3 is not a
vertical asymptote because reduced form must be used to find the asymptotes. The graph has a hole at
( −3, −1) .
Step 5: Since n = m + 1 there is an oblique asymptote. The line y = x + 2 is the oblique asymptote. Solve to
find intersection points:
x2 + 5x + 6
= x+2
x+3
x 2 + 5 x + 6 = ( x + 2 )( x + 3)
x2 + 5x + 6 = x2 + 5x + 6
0=0
The oblique asymptote intersects R ( x) at every point of the form ( x, x + 2) except (–3, –1).

Step 6:

Steps 7: Graphing:

370
Copyright © 2015 Pearson Education, Inc.
Section 3.5: The Graph of a Rational Function

x 2 + x − 30 ( x + 6)( x − 5)
38. R ( x ) = = = x−5 p( x) = x 2 + x − 30; q( x) = x + 6; n = 2; m = 1
x+6 x+6

Step 1: Domain: {x x ≠ − 6}
02 + (0) − 30 −30
The y-intercept is R (0) = = = −5 . Plot the point ( 0, −5 ) .
0+6 6

Step 2 & 3:In lowest terms, R ( x ) = x − 5, x ≠ −6 . The x-intercept is the zero of y = x − 5 : 5 with odd
multiplicity.
Note: –6 is not a zero because reduced form must be used to find the zeros.
Plot the point ( 5, 0 ) . The graph will cross the x-axis at this point.

Step 4: In lowest terms, R ( x ) = x − 5, x ≠ −6 . There are no vertical asymptotes. Note: x = − 6 is not a vertical
asymptote because reduced form must be used to find the asymptotes. The graph has a hole at
( −6, −11) .
Step 5: Since n = m + 1 there is an oblique asymptote. The line y = x − 5 is the oblique asymptote.
Solve to find intersection points:
x 2 + x − 30
= x −5
x+6
x 2 + x − 30 = ( x + 6 )( x − 5 )
x 2 + x − 30 = x 2 + x − 30
0=0
The oblique asymptote intersects R ( x) at every point of the form ( x, x − 5) except (–6, –11).

Step 6:

Steps 7: Graphing:

371
Copyright © 2015 Pearson Education, Inc.
Chapter 3: Polynomial and Rational Functions

3x − 6 3x − 6 −3( x − 2)
39. H ( x) = = = p ( x ) = 3x − 6; q( x) = 4 − x 2 ; n = 1; m = 2
4 − x2 −( x 2 − 4) ( x + 2)( x − 2)

Step 1: Domain: {x x ≠ − 2, x ≠ 2}
3(0) − 6 −6 3 3
The y-intercept is H (0) = 2
= = − . Plot the point 0, − .
4−0 4 2 2

3x − 6 −3
Step 2 & 3: H ( x) = 2
= is in lowest terms. The x-intercept is the zero of p( x) : none
4− x x+2

3x − 6 −3
Step 4: H ( x) = 2
= is in lowest terms so R ( x) = x + 2 . The vertical asymptotes are the zeros of
4− x x+2
R ( x) : x = − 2 .
Graph the asymptote using a dashed line. The multiplicity of -2 is odd so the graph will approach plus
or minus infinity on either side of the asymptote.

Step 5: Since n < m , the line y = 0 is the horizontal asymptote. Graph this asymptote using a dashed line.
Solve to find intersection points:
3x − 6
=0
4 − x2
3x − 6 = 0
x=2
The function is not defined at x = 2 so there is no interection.

Step 6:

Steps 7: Graphing:

372
Copyright © 2015 Pearson Education, Inc.
Section 3.5: The Graph of a Rational Function

2 − 2x −2( x − 1)
40. H ( x) = = p ( x) = 2 − 2 x; q ( x ) = x 2 − 1; n = 1; m = 2
x2 − 1 ( x + 1)( x − 1)

Step 1: Domain: {x x ≠ − 1, x ≠ 1}
2 − 2(0) 2
The y-intercept is H (0) = 2
= = −2 . Plot the point ( 0, −2) .
0 −1 −1

−2( x − 1)
Step 2 & 3: H ( x) = is in lowest terms. The possible x-intercept is the zero of p ( x) : -1 but H ( −1) is
( x − 1)( x + 1)
not defined.

−2
Step 4: H ( x) = is in lowest terms so R ( x) = x + 1 . The vertical asymptotes are the zeros of R( x) :
x +1
x = − 1 . There is a hole at (1, −1)
Graph the asymptote using a dashed line. The multiplicity of -1 is odd so the graph will approach plus
or minus infinity on either side of the asymptote.

Step 5: Since n < m , the line y = 0 is the horizontal asymptote. Graph this asymptote using a dashed line.
Solve to find intersection points:
2 − 2x
=0
x2 − 1
2 − 2x = 0
x =1
H ( x) is not defined at x = 1 so there is no intersection.

Step 6:

Steps 7: Graphing:

373
Copyright © 2015 Pearson Education, Inc.
Chapter 3: Polynomial and Rational Functions

x2 − 5x + 4 ( x − 1)( x − 4) x−4
41. F ( x) = 2
= 2
= p ( x) = x 2 − 5 x + 4; q( x) = x 2 − 2 x + 1; n = 2; m = 2
x − 2x + 1 ( x − 1) x −1

Step 1: Domain: {x x ≠ 1}
02 − 5(0) + 4 4
The y-intercept is R (0) = 2
= = 4 . Plot the point ( 0, 4) .
0 − 2(0) + 1 1

x−4
Step 2 & 3:In lowest terms, F ( x) = , x ≠ 1 . The x-intercept is the zero of y = x − 4 : 4 with odd multiplicity.
x −1
Note: –5 is not a zero because reduced form must be used to find the zeros.
Plot the point ( 4,0) . The graph crosses the x-axis at this point.

x−4
Step 4: In lowest terms, F ( x) = , x ≠ 1 . The vertical asymptote is the zero of f ( x ) = x − 1 : x = 1 ; Graph
x −1
this asymptote using a dashed line. The multiplicity of 1 is odd so the graph will approach plus or
minus infinity on either side of the asymptote.

Step 5: Since n = m , the line y = 1 is the horizontal asymptote. Graph this asymptote using a dashed line.
Solve to find intersection points:
x2 − 5x + 4
=1
x2 − 2 x + 1
x2 − 5x + 4 = x2 − 2 x + 1
−3x = −3
x =1
F ( x ) does not intersect y = 1 because R ( x) is not defined at x = 1 .

Step 6:

Steps 7: Graphing:

374
Copyright © 2015 Pearson Education, Inc.
Section 3.5: The Graph of a Rational Function

x 2 − 2 x − 15 ( x − 5)( x + 3) x − 5
42. F ( x) = 2
= = p ( x) = x 2 − 2 x − 15; q( x) = x 2 + 6 x + 9; n = 2; m = 2
x + 6x + 9 ( x + 3)( x + 3) x + 3

Step 1: Domain: {x x ≠ −3}


02 − 2(0) − 15 −15 5 5
The y-intercept is F (0) = 2
= = − . Plot the point 0, − .
0 + 6(0) + 9 9 3 3

x−5
Step 2 & 3:In lowest terms, F ( x) = , x ≠ −3 . The x-intercept is the zero of y = x − 5 : 5 with odd
x+3
multiplicity.
Note: –3 is not a zero because reduced form must be used to find the zeros.
Plot the point (5,0) . The graph crosses the x-axis at this point.

x−5
Step 4: In lowest terms, F ( x) = , x ≠ −3 . The vertical asymptote is the zero of f ( x ) = x + 3 : x = − 3 ;
x+3
Graph this asymptote using a dashed line. The multiplicity of -3 is odd so the graph will approach plus
or minus infinity on either side of the asymptote.

Step 5: Since n = m , the line y = 1 is the horizontal asymptote. Graph this asymptote using a dashed line.
Solve to find intersection points:
x 2 − 2 x − 15
=1
x2 + 6 x + 9
x 2 − 2 x − 15 = x 2 + 6 x + 9
−8 x = 24
x = −3
R ( x) does not intersect y = 1 because F ( x ) is not defined at x = −3 .

Step 6:

Steps 7: Graphing:

375
Copyright © 2015 Pearson Education, Inc.
Chapter 3: Polynomial and Rational Functions

x
43. G ( x) = p( x) = x; q ( x) = ( x + 2) 2 ; n = 1; m = 2
( x + 2) 2

Step 1: Domain: { x x ≠ −2} .

x
Step 2: G ( x) = is in lowest terms.
( x + 2) 2

(0) 0
Step 3: The y-intercept is G (0) = 2
= = 0 . Plot the point ( 0, 0 ) .
(0 + 2) 4

The x-intercept is the zero of p( x) : 0 with odd multiplicity.


Plot the point ( 0, 0 ) . The graph crosses the x-axis at this point.

x
Step 4: G ( x) = is in lowest terms. The vertical asymptote is the zero of q( x) : x = −2 . Graph this
( x + 2) 2
asymptote. The multiplicity of -2 is even so the graph will approach the same infinity on both sides of
the asymptote.

Step 5: Since n < m , the line y = 0 is the horizontal asymptote. Graph this asymptote using a dashed line.
Solve to find intersection points:
x
=0
( x + 2) 2
x=0
G ( x) intersects y = 0 at (0, 0) .

Step 6:

Steps 7: Graphing:

376
Copyright © 2015 Pearson Education, Inc.
Section 3.5: The Graph of a Rational Function

2− x
44. G ( x) = p ( x ) = 2 − x; q ( x) = ( x − 1) 2 ; n = 1; m = 2
( x − 1) 2

Step 1: Domain: { x x ≠ 1} .

2− x
Step 2: G ( x) = is in lowest terms.
( x − 1) 2

2 − (0) 2
Step 3: The y-intercept is G (0) = 2
= = 2 . Plot the point ( 0, 2) .
(0 − 1) 1

The x-intercept is the zero of p( x) : 2 with odd multiplicity.


Plot the point ( 2, 0) . The graph crosses the x-axis at this point.

2− x
Step 4: G ( x) = is in lowest terms. The vertical asymptote is the zero of q( x) : x = 1 . Graph this
( x − 1) 2
asymptote. The multiplicity of 1 is even so the graph will approach the same infinity on both sides of
the asymptote.
Step 5: Since n < m , the line y = 0 is the horizontal asymptote. Graph this asymptote using a dashed line.
Solve to find intersection points:
2− x
=0
( x − 1) 2
2− x = 0
x=2
G ( x) intersects y = 0 at (2, 0) .

Step 6:

Steps 7: Graphing:

377
Copyright © 2015 Pearson Education, Inc.
Chapter 3: Polynomial and Rational Functions

1 x2 + 1
45. f ( x) = x + = p( x) = x 2 + 1; q ( x) = x; n = 2; m = 1
x x

Step 1: Domain: {x x ≠ 0}
There is no y-intercept because 0 is not in the domain.

x2 + 1
Step 2 & 3: f ( x) = is in lowest terms. There are no x-intercepts since x 2 + 1 = 0 has no real solutions.
x

x2 + 1
Step 4: f ( x) = is in lowest terms. The vertical asymptote is the zero of q( x) : x = 0 Graph this
x
asymptote using a dashed line. The multiplicity of 0 is odd so the graph will approach plus or minus
infinity on either side of the asymptote.

Step 5: Since n = m + 1 , there is an oblique asymptote.


x 1
f ( x) = x +
Dividing: x x2 + 1 x
x 2 The oblique asymptote is y =x.
1
Graph this asymptote using a dashed line. Solve to find intersection points:
x2 + 1
=x
x
x2 + 1 = x2
1≠ 0
The oblique asymptote does not intersect f ( x ) .

Step 6:

378
Copyright © 2015 Pearson Education, Inc.
Section 3.5: The Graph of a Rational Function

Steps 7: Graphing :

9 2x2 + 9
46. f ( x) = 2 x + = p( x) = 2 x 2 + 9; q ( x) = x; n = 2; m = 1
x x

Step 1: Domain: {x x ≠ 0}
There is no y-intercept because 0 is not in the domain.

2 x2 + 9
Step 2 & 3: f ( x) = is in lowest terms. There are no x-intercepts since 2 x 2 + 9 = 0 has no real solutions.
x

2 x2 + 9
Step 4: f ( x) = is in lowest terms. The vertical asymptote is the zero of q( x) : x = 0
x
Graph this asymptote using a dashed line. The multiplicity of 0 is odd so the graph will approach plus
or minus infinity on either side of the asymptote.

Step 5: Since n = m + 1 , there is an oblique asymptote. Dividing:


2x
9
x 2x2 + 9 f ( x) = 2 x +
x
2x2
9
The oblique asymptote is y = 2 x . Graph this asymptote using a dashed line. Solve to find intersection
points:
2 x2 + 9
= 2x
x
2 x2 + 9 = 2 x2
9≠0
The oblique asymptote does not intersect f ( x ) .

379
Copyright © 2015 Pearson Education, Inc.
Chapter 3: Polynomial and Rational Functions

Step 6:

Steps 7: Graphing:

47. 2 1 x3 + 1 ( x + 1) x − x + 1
f ( x) = x + = =
2
( ) p ( x) = x3 + 1; q ( x) = x; n = 3; m = 1 .
x x x

Step 1: Domain: {x x ≠ 0}
There is no y-intercept because 0 is not in the domain.

x3 + 1
Step 2 & 3: f ( x) = is in lowest terms. The x-intercept is the zero of p ( x) : –1 with odd multiplicity.
x
Plot the point ( −1, 0 ) . The graph crosses the x-axis at this point.

x3 + 1
Step 4: f ( x) = is in lowest terms. The vertical asymptote is the zero of q( x) : x = 0
x
Graph this asymptote using a dashed line. The multiplicity of 0 is odd so the graph will approach plus
or minus infinity on either side of the asymptote.

Step 5: Since n > m + 1 , there is no horizontal or oblique asymptote.

Step 6:

380
Copyright © 2015 Pearson Education, Inc.
Section 3.5: The Graph of a Rational Function

Steps 7: Graphing:

(−1, 0)

48. f ( x) = 2 x 2 +
3
16 2 x3 + 16 2 x + 8
= =
(
=
) (
2 ( x + 2) x2 − 2x + 4 ) p ( x) = 2 x3 + 16; q ( x) = x; n = 3; m = 1
x x x x

Step 1: Domain: {x x ≠ 0}
There is no y-intercept because 0 is not in the domain.

2 x3 + 16
Step 2 & 3: f ( x) = is in lowest terms. The x-intercept is the zero of p( x) : −2 with odd multiplicity.
x
Plot the point. ( −2, 0 ) . The graph crosses the x-axis at this point.

2 x3 + 16
Step 4: f ( x) = is in lowest terms. The vertical asymptote is the zero of q( x) : x = 0
x
Graph this asymptote using a dashed line. The multiplicity of 0 is odd so the graph will approach plus
or minus infinity on either side of the asymptote.

Step 5: Since n > m + 1 , there is no horizontal or oblique asymptote.

Step 6:

Interval ( −∞, −2 ) ( −2, 0 ) ( 0, ∞ )


Number Chosen −3 −1 1
Value of f f ( −3) = 16 f ( −1) = −14 f (1) = 18
Location of Graph Above x-axis Below x-axis Above x-axis
Point on Graph ( −3,16 ) ( −1, −14 ) (1,18)

381
Copyright © 2015 Pearson Education, Inc.
Chapter 3: Polynomial and Rational Functions

Steps 7: Graphing:

1 x4 + 1
49. f ( x) = x + 3
= 3
p ( x ) = x 4 + 1; q ( x ) = x3 ; n = 4; m = 3
x x

Step 1: Domain: {x x ≠ 0}
There is no y-intercept because 0 is not in the domain.

x4 + 1
Step 2 & 3: f ( x ) = is in lowest terms. There are no x-intercepts since x 4 + 1 = 0 has no real solutions.
x3

x4 + 1
Step 4: f ( x) = is in lowest terms. The vertical asymptote is the zero of q( x) : x = 0
x3
Graph this asymptote using a dashed line. The multiplicity of 0 is odd so the graph will approach plus
or minus infinity on either side of the asymptote.

Step 5: Since n = m + 1 , there is an oblique asymptote. Dividing:


x
1
x3 x 4 + 1 f ( x) = x +
x3
x4
1
The oblique asymptote is y = x. Graph this asymptote using a dashed line. Solve to find intersection
points:
x4 + 1
=x
x3
x4 + 1 = x4
1≠ 0
The oblique asymptote does not intersect f ( x ) .

382
Copyright © 2015 Pearson Education, Inc.
Section 3.5: The Graph of a Rational Function

Step 6:

Steps 7: Graphing:

9 2 x4 + 9
50. f ( x) = 2 x + = p ( x) = 2 x 4 + 9; q ( x) = x3 ; n = 4; m = 3
x3 x3

Step 1: Domain: {x x ≠ 0}
There is no y-intercept because 0 is not in the domain.

2 x4 + 9
Step 2 & 3: f ( x) = is in lowest terms. There are no x-intercepts since 2 x 4 + 9 = 0 has no real solutions.
x3

2 x4 + 9
Step 4: f ( x) = is in lowest terms. The vertical asymptote is the zero of q( x) : x = 0
x3
Graph this asymptote using a dashed line. The multiplicity of 0 is odd so the graph will approach plus
or minus infinity on either side of the asymptote.

Step 5: Since n = m + 1 , there is an oblique asymptote. Dividing:


2x
9
x 2 x4 + 9
3
f ( x) = 2 x + 3
x
2 x4
9
The oblique asymptote is y = 2 x . Graph this asymptote using a dashed line. Solve to find intersection
points:

383
Copyright © 2015 Pearson Education, Inc.
Chapter 3: Polynomial and Rational Functions

2 x4 + 9
= 2x
x3
2 x4 + 9 = 2 x4
9≠0
The oblique asymptote does not intersect f ( x ) .

Step 6:

Steps 7: Graphing:

1 9
51. f ( x) = x + , x > 0 52. f ( x) = 2x + , x > 0
x x

minimum value = 2.00 at x = 1.00 minimum value ≈ 8.49 at x ≈ 2.12

384
Copyright © 2015 Pearson Education, Inc.
Section 3.5: The Graph of a Rational Function

53.
1
f ( x ) = x2 + , x > 0 ( x − 1)( x − 3) ( x 2 + a )
x 59. One possibility: R ( x ) =
( x + 1) 2 ( x − 2) 2
(Using the point ( 0,1) leads to a = 4 / 3 .) Thus,
( x − 1)( x − 3) ( x 2 + 43 )
R( x) = .
( x + 1) 2 ( x − 2) 2

3( x + 2)( x − 1) 2
60. One possibility: R ( x ) =
minimum value ≈ 1.89 at x ≈ 0.79 ( x + 3)( x − 4) 2

9 61. a. The degree of the numerator is 1 and the


54. f ( x ) = 2 x2 + , x > 0 degree of the denominator is 2. Thus, the
x
horizontal asymptote is y = 0 . The
concentration of the drug decreases to 0 as
time increases.
b. Graphing:

minimum value ≈ 10.30 at x ≈ 1.31

1
55. f ( x) = x + ,x>0
x3

c. Using MAXIMUM, the concentration is


highest after t ≈ 0.71 hours.
62. a. The degree of the numerator is 1 and the
degree of the denominator is 2. Thus, the
horizontal asymptote is y = 0 . The
concentration of the drug decreases to 0 as
minimum value ≈ 1.75 at x ≈ 1.32 time increases.
9 b. Graphing:
56. f ( x) = 2 x + 3
,x>0
x

minimum value ≈ 5.11 at x ≈ 1.92 c. Using MAXIMUM, the concentration is


highest after t = 5 minutes.
x2
57. One possibility: R ( x) = 63. a. The cost of the project is the sum of the cost
x2 − 4 for the parallel side, the two other sides, and
the posts.
x
58. One possibility: R ( x) = − 2
x −1

385
Copyright © 2015 Pearson Education, Inc.
Chapter 3: Polynomial and Rational Functions

A = xy  727.4 
1000 = xy b. 620 = 600  
 772.4 − v
 s 
1000
y= 436, 440
x 620 =
772.4 − vs
If the length of a perpendicular side is x feet,
1000 620 ( 772.4 − vs ) = 436, 440
the length of the parallel side is y =
x 436, 440
772.4 − vs =
feet. Thus, 620
1000 436, 440
C ( x) = 2 ⋅8⋅ x + 5⋅ + 4 ( 25 ) vs = 772.4 − ≈ 68.5
x 620
5000 If f ' = 620 Hz , the speed of the ambulance
= 16 x + + 100
x is roughly 68.5 miles per hour.
b. The domain is x > 0 . Note that x is a length 436, 440
so it cannot be negative. In addition, if c. y=
772.4 − x
x = 0 , there is no rectangle (that is, the area
is 0 square feet).
5000
c. C ( x ) = 16 x + + 100
x

436, 440
d. Let Y1 = and Y2 = 620 , then find
772.4 − x
d. Using MINIMUM, the dimensions of the intersection point.
cheapest cost are about 17.7 feet by 56.6
feet (longer side parallel to river).

The graph agrees with our direct calculation.


65. a. The surface area is the sum of the areas of
the six sides.
2 53
Note: x = 17 = feet and S = xy + xy + xy + xy + x 2 + x 2 = 4 xy + 2 x 2
3 3
The volume is x ⋅ x ⋅ y = x 2 y = 10, 000 .
1000 3000
y= = feet . 10, 000
53 / 3 53 Thus, y = , so
x2
 772.4 − 45   10, 000 
f ' ( vs ) = 600 
2
64. a. S ( x) = 4 x   + 2x
 772.4 − v   x 2

 s 

 727.4  40, 000


= 2 x2 +
= 600 
 772.4 − v  x
 s 
2 x3 + 40, 000
=
x

386
Copyright © 2015 Pearson Education, Inc.
Section 3.5: The Graph of a Rational Function

b. Graphing: 5000
y= ≈ 10.78
( 21.54 )2
The dimensions of the box are: 21.54 in. by
21.54 in. by 10.78 in.
e. Answers will vary. One possibility is to save
costs or reduce weight by minimizing the
material needed to construct the box.

c. Using MINIMUM, the minimum surface 67. a. 500 = π r 2 h


area (amount of cardboard) is about 2784.95 500
square inches. h=
π r2
d. The surface area is a minimum when
x ≈ 21.54 inches. C (r ) = 6(2π r 2 ) + 4(2π rh)
10, 000  500 
y= ≈ 21.54 inches = 12π r 2 + 8π r  2 
( 21.544 )2  πr 
The dimensions of the box are: 21.54 in. by 4000
= 12π r 2 +
21.54 in. by 21.54 in. r
e. Answers will vary. One possibility is to save b. Graphing:
costs or reduce weight by minimizing the
material needed to construct the box.

66. a. The surface area is the sum of the areas of


the five sides.
S = xy + xy + xy + xy + x 2 = 4 xy + x 2
The volume is x ⋅ x ⋅ y = x 2 y = 5000.
5000 Using MINIMUM, the cost is least for
Thus, y = , so
x2 r ≈ 3.76 cm.
 5000 
S ( x) = 4 x  2  + x 2 68. a. 100 = π r 2 h
 x 
20, 000 100
= x2 + h=
x π r2
3
x + 20, 000 A(r ) = 2π r 2 + 2π rh
=
x  100 
= 2π r 2 + 2π r  2 
b. Graphing:
 πr 
200
= 2π r 2 +
r
200
b. A(3) = 2π⋅ 32 +
3
200
= 18π + ≈ 123.22 square feet
3
c. Using MINIMUM, the minimum surface 200
area (amount of cardboard) is about 1392.48 c. A(4) = 2π⋅ 42 +
4
square inches. = 32π + 50 ≈ 150.53 square feet
d. The surface area is a minimum when
x = 21.54 .

387
Copyright © 2015 Pearson Education, Inc.
Chapter 3: Polynomial and Rational Functions

200 b. Since R(x) is undefined and has a hole at


d. A(5) = 2π⋅ 52 +
5 5 14
− , , we can remove the
= 50π + 40 ≈ 197.08 square feet 2 11
discontinuity by defining the function as:
e. Graphing:
8 x 2 + 26 x + 15 5
2
if x ≠ −
R ( x) = 2 x − x − 15 2
14 5
if x = −
11 2

x2 − 1
71. y =
x −1
Using MINIMUM, the area is smallest when
r ≈ 2.52 feet.

69. a. Since R(x) is undefined and has a hole at


7
3, , we can remove the discontinuity by
5
defining the function as:
x3 − 1
2 y=
x + x − 12 x −1
if x ≠ 3
R ( x) = x2 − x − 6
7
if x = 3
5

b. Since R(x) is undefined and has a hole at


3
, −11 , we can remove the discontinuity
2 x4 − 1
y=
by defining the function as: x −1

6x2 − 7 x − 3 3
2
if x ≠
R( x) = 2 x − 7 x + 6 2
3
−11 if x =
2

70. a. Since R(x) is undefined and has a hole at x5 − 1


7 y=
−5, , we can remove the discontinuity x −1
2
by defining the function as:

x 2 + 3 x − 10
2
if x ≠ −5
R ( x ) = x + 8 x + 15
7
if x = −5
2 x = 1 is not a vertical asymptote because of the
following behavior:
When x ≠ 1 :
x 2 − 1 ( x + 1)( x − 1)
y= = = x +1
x −1 x −1
388
Copyright © 2015 Pearson Education, Inc.
Section 3.5: The Graph of a Rational Function

y=
x3 − 1 ( x − 1) x + x + 1
=
2
(
= x2 + x + 1
) y=
x4
x −1 x −1 x −1

y=
x4 − 1
=
(
x2 + 1 x2 − 1 )( )
x −1 x −1

=
(x 2
)
+ 1 ( x − 1)( x + 1)
x −1
3 2
= x + x + x +1 x6
y=
x 5 − 1 ( x 4 + x 3 + x 2 + x + 1)( x − 1) x −1
y= =
x −1 x −1
4 3 2
= x + x + x + x +1
In general, the graph of
xn − 1
y= , n ≥ 1, an integer,
x −1
will have a “hole” with coordinates (1, n ) . x8
y=
x −1
x2
72. y =
x −1

All four graphs have a vertical asymptote at 3x x−2


79. =
x2 3x + 1 x + 5
x =1. y = has an oblique asymptote at
x −1 3x ( x + 5) = ( x − 2)(3 x + 1)
y = x +1. 3x 2 + 15 x = 3 x 2 − 5 x − 2
15 x = −5 x − 2
73. Answers will vary.
1
20 x = −2 → x = −
74. Answers will vary. One example is 10
2
3 ( x − 2 )( x + 1)
R ( x) = . 80. The maximum value occurs at
( x + 5 )( x − 6 )2 b 6 9
x=− =− =
2a 2( − 23 ) 2
75. Answers will vary. One example is
2
2 ( x − 3)( x + 2 ) 9 2 9
2
9
R ( x) = 3
. f =− +6 −5
( x − 1) 2 3 2 2
2 81 54 17
76. Answers will vary. =− + −5 =
3 4 2 2
77. Answers will vary.
5 −3
81. ≈ −0.164
78. (4 x3 − 7 x + 1) − (5 x 2 − 9 x + 3) = 7+2
4 x3 − 7 x + 1 − 5 x 2 + 9 x − 3 = 4 x3 − 5 x 2 + 2 x − 2

389
Copyright © 2015 Pearson Education, Inc.
Chapter 3: Polynomial and Rational Functions

b. The graph of f is below the x-axis (so f is


negative) for x < 0 or 1 < x < 2 . Since the
Section 3.6
inequality is not strict, we include 0, 1, and 2
1. 3 − 4 x > 5 in the solution set. Therefore, the solution
−4 x > 2 set is {x x ≤ 0 or 1 ≤ x ≤ 2 } or, using
1 interval notation (−∞, 0] ∪ [1, 2] .
x<−
2
6. The x-intercepts of the graph of f are −1 , 1, and 2.
 1
The solution set is  x x < −  or, using interval a. The graph of f is below the x-axis (so f is
 2
negative) for −1 < x < 1 or x > 2 . Therefore,
 1
notation,  −∞, −  .
 2
the solution set is {x − 1 < x < 1 or x > 2 } or,
using interval notation, (−1, 1) ∪ (2, ∞) .
b. The graph of f is above the x-axis (so f is
positive) for x < −1 or 1 < x < 2 . Since the
inequality is not strict, we include 0, 1, and 2 in the
2. x 2 − 5 x ≤ 24 solution set. Therefore, the solution set is
x 2 − 5 x − 24 ≤ 0 {x x ≤ −1 or 1 ≤ x ≤ 2 } or, using interval notation
( x + 3)( x − 8) ≤ 0 (−∞, −1] ∪ [1, 2] .
2
f ( x) = x − 5 x − 24 = ( x + 3)( x − 8) 7. The x-intercept of the graph of f is 0.
x = −3, x = 8 are the zeros of f. a. The graph of f is below the x-axis (so f is
Interval (−∞, −3) (−3,8) (8, ∞) negative) for −1 < x < 0 or x > 1 . Therefore,
Number
−4 0 9
the solution set is {x − 1 < x < 0 or x > 1 } or,
Chosen using interval notation, (−1, 0) ∪ (1, ∞) .
Value of f 12 −24 12
b. The graph of f is above the x-axis (so f is
Conclusion Positive Negative Positive
positive) for x < −1 or 0 < x < 1 . Since the
The solution set is { x | −3 ≤ x ≤ 8} or, using inequality is not strict, we include 0 in the
solution set. Therefore, the solution set is
interval notation, [ −3, 8] .
{x x < −1 or 0 ≤ x < 1 } or, using interval
( ] notation (−∞, −1) ∪ [0, 1) .
−3 0 3 6 8
8. The x-intercepts of the graph of f are 1 and 3.
3. True
a. The graph of f is above the x-axis (so f is
4. False. The value 3 is not in the domain of f, so it positive) for x < −1 or −1 < x < 1 or x > 3 .
must be restricted from the solution. The Therefore, the solution set is
solution set would be { x | x ≤ 0 or x > 3} . {x x < −1 or − 1 < x < 1 or x > 3 } or, using
interval notation, (−∞, −1) ∪ (−1, 1) ∪ (3, ∞) .
5. The x-intercepts of the graph of f are 0, 1, and 2.
a. The graph of f is above the x-axis (so f is b. The graph of f is below the x-axis (so f is
positive) for 0 < x < 1 or x > 2 . Therefore, negative) for 1 < x < 2 or 2 < x < 3 . Since
the inequality is not strict, we include 1 and
the solution set is {x 0 < x < 1 or x > 2 } or, 3 in the solution set. Therefore, the solution
using interval notation, (0, 1) ∪ (2, ∞) .

390
Copyright © 2015 Pearson Education, Inc.
Section 3.6: Polynomial and Rational Inequalities

set is { x 1 ≤ x < 2 or 2 < x ≤ 3 } or, using


interval notation [1, 2) ∪ (2, 3] .

9. We graphed f ( x) = x 2 ( x − 3) in Problem 69 of
Section 4.1. The graph is reproduced below.

From the graph, we that f is above the x-axis (so f is


positive) for −4 < x < 2 or x > 2 . Since the
inequality is not strict, we include −4 and 2 in the
solution set. Therefore, the solution set is
{x x ≥ −4} or, using interval notation [−4, ∞) .

12. We graphed f ( x) = ( x − 1)( x + 3) 2 in Problem 72


From the graph, we see that f is below the x-axis
(so f is negative) for x < 0 or 0 < x < 3 . Thus, of Section 4.1. The graph is reproduced below.

the solution set is {x x < 0 or 0 < x < 3 } or,


using interval notation (−∞, 0) ∪ (0, 3) .

10. We graphed f ( x) = x( x + 2) 2 in Problem 70 of


Section 4.1. The graph is reproduced below.

From the graph, we that f is above the x-axis (so f


is positive) for x > 1 . Therefore, the solution set
is {x x > 1} or, using interval notation (1, ∞) .

13. We graphed f ( x) = −2( x + 2)( x − 2)3 in Problem


73 of Section 4.1. The graph is reproduced below.
From the graph, we see that f is below the x-axis
(so f is negative) for x < −2 or −2 < x < 0 .
Since the inequality is not strict, we include −2
and 0 in the solution set. Therefore, the solution
set is {x x ≤ 0} or, using interval notation
(−∞, 0] .

11. We graphed f ( x) = ( x + 4)( x − 2)2 in Problem 71


of Section 4.1. The graph is reproduced below.
From the graph, we see that f is below the x-axis
(so f is negative) for x < −2 or x > 2 . Since the
inequality is not strict, we include −2 and 2 in the
solution set. Therefore, the solution set is

391
Copyright © 2015 Pearson Education, Inc.
Chapter 3: Polynomial and Rational Functions

{x x ≤ −2 or x ≥ 2} or, using interval notation


(−∞, − 2] ∪ [2, ∞) .

1
14. We graphed f ( x) = − ( x + 4)( x − 1)3 in Problem
2
74 of Section 4.1. The graph is reproduced below.

From the graph, we that R is below the x-axis (so


R is negative) for x < −2 or 0 < x < 1 . Thus, the
solution set is { x x < −2 or 0 < x < 1} or, using
interval notation (−∞, −2) ∪ (0, 1) .

3x + 3
17. We graphed R ( x) = in Problem 9 of
2x + 4
From the graph, we see that f is below the x-axis Section 4.5. The graph is reproduced below.
(so f is negative) for x < −4 or x > 1 . Therefore,
the solution set is {x x < −4 or x > 1 } or, using
interval notation (−∞, −4) ∪ (1, ∞) .

x +1
15. We graphed R ( x ) = in Problem 7 of
x( x + 4)
Section 4.5. The graph is reproduced below.

From the graph, we that R is below the x-axis (so


R is negative) for −2 < x < −1 . Since the inequality
is not strict, we include −1 in the solution set.
Therefore, the solution set is { x − 2 < x ≤ −1} or,
using interval notation (−2, −1] .

2x + 4
18. We graphed R ( x) = in Problem 10 of
x −1
From the graph, we that R is above the x-axis (so Section 4.5. The graph is reproduced below.
R is positive) for −4 < x < −1 or x > 0 .
Therefore, the solution set is
{ x − 4 < x < −1 or x > 0} or, using interval
notation (−4, −1) ∪ (0, ∞) .

x
16. We graphed R ( x) = in Problem 8
( x − 1)( x + 2)
of Section 4.5. The graph is reproduced below.

From the graph, we that R is above the x-axis (so


R is positive) for x < −2 or x > 1 . Since the
inequality is not strict, we include −2 in the
solution set. Therefore, the solution set is
392
Copyright © 2015 Pearson Education, Inc.
Section 3.6: Polynomial and Rational Inequalities

{x x ≤ −2 or x > 1} or, using interval notation x = – 8, x = 0 are the zeros of f .


(−∞, −2] ∪ (1, ∞) . Interval (−∞, − 8) (−8, 0) (0, ∞)
Number
19. ( x − 5)2 ( x + 2) < 0 −9 −1 1
Chosen
f ( x) = ( x − 5) 2 ( x + 2) Value of f −81 7 9
x = 5, x = − 2 are the zeros of f . Conclusion Negative Positive Positive
Interval (−∞, − 2) (−2, 5) (5, ∞) The solution set is {x x < − 8} or, using interval
Number
−3 0 6 notation, ( −∞, −8 ) .
Chosen
Value of f −64 50 8 23. 2 x3 > −8 x 2
Conclusion Negative Positive Positive 2 x3 + 8 x 2 > 0
The solution set is {x x < −2 } or, using interval 2x2 ( x + 4) > 0
notation, ( −∞, −2 ) . f ( x ) = 2 x3 + 8 x 2
x = 0, x = −4 are the zeros of f.
20. ( x − 5)( x + 2) 2 > 0
Interval (−∞, −4) (−4, 0) (0, ∞)
f ( x) = ( x − 5)( x + 2) 2
Number
x = 5, x = − 2 are the zeros of f . −5 −1 1
Chosen
Interval (−∞, − 2) (−2, 5) (5, ∞) Value of f −50 6 10
Number Conclusion Negative Positive Positive
−3 0 6
Chosen
The solution set is { x | −4 < x < 0 or x > 0} or,
Value of f −8 −20 64
using interval notation, ( −4, 0 ) ∪ ( 0,∞ ) .
Conclusion Negative Negative Positive
The solution set is {x x > 5 } or, using interval 24. 3 x3 < −15 x 2
notation, ( 5, ∞ ) . 3x3 + 15 x 2 < 0
3x2 ( x + 5) < 0
21. x3 − 4 x 2 > 0
f ( x ) = 3 x3 + 15 x 2
x 2 ( x − 4) > 0
x = 0, x = −5 are the zeros of f.
f ( x) = x3 − 4 x 2 = x 2 ( x − 4 )
Interval (−∞, −5) (−5, 0) (0, ∞)
x = 0, x = 4 are the zeros of f .
Number
Interval (−∞, 0) (0, 4) (4, ∞) −6 −1 1
Chosen
Number Value of f −108 12 18
−1 1 5
Chosen Conclusion Negative Positive Positive
Value of f −5 −3 25
The solution set is { x | x < −5} or, using interval
Conclusion Negative Negative Positive
notation, ( −∞, −5 ) .
The solution set is {x x > 4 } or, using interval
notation, ( 4, ∞ ) . 25. ( x − 1)( x − 2)( x − 3) ≤ 0
f ( x) = ( x − 1)( x − 2)( x − 3)
22. x3 + 8 x 2 < 0 x = 1, x = 2, x = 3 are the zeros of f .
x 2 ( x + 8) < 0
f ( x) = x3 + 8 x 2 = x 2 ( x + 8 )

393
Copyright © 2015 Pearson Education, Inc.
Chapter 3: Polynomial and Rational Functions

Interval ( −∞ , 1) (1, 2) (2, 3) (3, ∞ ) Interval ( −∞, − 3) ( −3, 0) (0, 1) (1, ∞ )


Number Number
0 1.5 2.5 4 −4 −1 0.5 2
Chosen Chosen
Value of f −6 0.375 −0.375 6 Value of f −20 4 −0.875 10
Conclusion Negative Positive Negative Positive Conclusion Negative Positive Negative Positive

The solution set is {x x ≤ 1 or 2 ≤ x ≤ 3 } or, The solution set is {x − 3 < x < 0 or x > 1 } or,
using interval notation, ( −∞,1] ∪ [ 2,3] . using interval notation, ( −3, 0 ) ∪ (1, ∞ ) .

26. ( x + 1)( x + 2)( x + 3) ≤ 0 29. x4 > x2


f ( x) = ( x + 1)( x + 2)( x + 3) x4 − x2 > 0
x = −1, x = − 2, x = −3 are the zeros of f .
x 2 ( x 2 − 1) > 0
Interval ( −∞, − 3) ( −3, −2) ( −2, −1) ( −1, ∞ )
x 2 ( x − 1)( x + 1) > 0
Number
−4 −2.5 −1.5 0 f ( x) = x 2 ( x − 1)( x + 1)
Chosen
x = −1 , x = 0, x = 1 are the zeros of f
Value of f −6 0.375 −0.375 6
Interval ( −∞ , − 1) ( −1, 0) (0, 1) (1, ∞ )
Conclusion Negative Positive Negative Positive
Number
The solution set is −2 −0.5 0.5 2
{ x x ≤ −3 or − 2 ≤ x ≤ −1 } or, using interval Chosen
Value of f 12 −0.1875 −0.1875 12
notation, ( −∞, −3] ∪ [ −2, −1] .
Conclusion Positive Negative Negative Positive

27. 3 2
x − 2 x − 3x > 0 The solution set is {x x < − 1 or x > 1 } or,

(
x x2 − 2 x − 3 > 0 ) using interval notation, ( −∞, −1) ∪ (1,∞ ) .
x( x + 1)( x − 3) > 0
30. x 4 < 9 x 2
3 2
f ( x) = x − 2 x − 3 x
x4 − 9 x2 < 0
x = −1, x = 0, x = 3 are the zeros of f .
x 2 ( x 2 − 9) < 0
Interval ( −∞ , − 1) ( −1, 0) (0, 3) (3, ∞ )
x 2 ( x − 3)( x + 3) < 0
Number
−2 −0.5 1 4 f ( x) = x 2 ( x − 3)( x + 3)
Chosen
Value of f −10 0.875 −4 20 x = 0, x = 3, x = −3 are the zeros of f
Conclusion Negative Positive Negative Positive Interval ( −∞, − 3) ( −3, 0) (0, 3) (3, ∞ )

The solution set is {x − 1 < x < 0 or x > 3 } or,


Number
−4 −1 1 4
Chosen
using interval notation, ( −1, 0 ) ∪ ( 3,∞ ) .
Value of f 112 −8 −8 112
3 2 Conclusion Positive Negative Negative Positive
28. x + 2 x − 3x > 0
x( x 2 + 2 x − 3) > 0 The solution set is {x – 3 < x < 0 or 0 < x < 3 }
x( x + 3)( x − 1) > 0 or, using interval notation, ( −3, 0 ) ∪ ( 0,3) .
f ( x) = x( x + 3)( x − 1)
x = 0, x = −3, x = 1 are the zeros of f

394
Copyright © 2015 Pearson Education, Inc.
Section 3.6: Polynomial and Rational Inequalities

31. x4 > 1 x −3
34. >0
4
x −1 > 0 x +1
x −3
( x − 1)( x 2 + 1) > 0
2 f ( x) =
x +1
( x − 1)( x + 1)( x 2 + 1) > 0 The zeros and values where f is undefined are
f ( x) = ( x − 1)( x + 1)( x 2 + 1) x = − 1 and x = 3 .
x = 1, x = −1 are the zeros of f ; x 2 + 1 has no Interval (−∞, − 1) (−1, 3) (3, ∞)
real solution Number
−2 0 4
Interval ( −∞ , − 1) ( −1,1) (1, ∞ ) Chosen
Number Chosen −2 0 2 Value of f 5 −3 0.2
Value of f 15 −1 15 Conclusion Positive Negative Positive
Conclusion Positive Negative Positive
The solution set is {x x < − 1 or x > 3 } or,
The solution set is {x x < −1 or x > 1 } or, using using interval notation, ( −∞, −1) ∪ ( 3, ∞ ) .
interval notation, ( −∞, −1) ∪ (1, ∞ ) .
( x − 1)( x + 1)
3
35. ≤0
32. x >1 x
3
x −1 > 0 ( x − 1)( x + 1)
f ( x) =
2 x
( x − 1)( x + x + 1) > 0
The zeros and values where f is undefined are
f ( x) = ( x − 1)( x 2 + x + 1) x = − 1, x = 0 and x = 1 .
x = 1 is a zero of f; x 2 + x + 1 has no real solution. Interval ( −∞ , − 1) ( −1, 0) (0, 1) (1, ∞ )
Interval ( −∞, 1) (1, ∞ ) Number
Number Chosen 0 2 −2 −0.5 0.5 2
Chosen
Value of f −1 7 Value of f −1.5 1.5 −1.5 1.5
Conclusion Negative Positive Conclusion Negative Positive Negative Positive
The solution set is {x x > 1 } or, using interval The solution set is {x x ≤ −1 or 0 < x ≤ 1 } or,
notation, (1, ∞ ) . using interval notation, ( −∞, −1] ∪ ( 0,1] .

x +1 ( x − 3)( x + 2)
33. >0 36. ≤0
x −1 x −1
x +1 ( x − 3)( x + 2)
f ( x) = f ( x) =
x −1 x −1
The zeros and values where f is undefined are The zeros and values where f is undefined are
x = − 1 and x = 1 . x = − 2, x = 1 and x = 3 .
Interval (−∞, − 1) (−1,1) (1, ∞)
Number
−2 0 2
Chosen
1
Value of f −1 3
3
Conclusion Positive Negative Positive
The solution set is {x x < − 1 or x > 1 } or,
using interval notation, ( −∞, −1) ∪ (1, ∞ ) .

395
Copyright © 2015 Pearson Education, Inc.
Chapter 3: Polynomial and Rational Functions

Interval ( −∞ , − 2) ( −2, 1) (1, 3) (3, ∞ ) The solution set is {x x < − 2 or x > 2 } or,
Number using interval notation, ( −∞, −2 ) ∪ ( 2,∞ ) .
−3 0 2 4
Chosen
x+4
39. ≤1
Value of f −1.5 6 −4 2 x−2
Conclusion Negative Positive Negative Positive x+4
−1 ≤ 0
x−2
The solution set is {x x ≤ − 2 or 1 < x ≤ 3 } or, x + 4 − ( x − 2)
≤0
using interval notation, ( −∞, −2] ∪ (1,3] . x−2
6
≤0
( x − 2) 2 x−2
37. ≥0
x2 − 1 6
f ( x) =
( x − 2) 2 x−2
≥0 The value where f is undefined is x = 2 .
( x + 1)( x − 1)
( x − 2) 2 Interval (−∞, 2) (2, ∞)
f ( x) = Number Chosen 0 3
x2 − 1
The zeros and values where f is undefined are Value of f −3 6
x = − 1, x = 1 and x = 2 . Conclusion Negative Positive

Interval ( −∞, − 1) ( −1,1) (1, 2) (2, ∞ ) The solution set is {x x < 2 } or, using interval
Number notation, ( −∞, 2 ) .
−2 0 1.5 3
Chosen
x+2
16 40. ≥1
Value of f −4 0.2 0.125 x−4
3 x+2
Conclusion Positive Negative Positive Positive −1 ≥ 0
x−4
x + 2 − ( x − 4)
The solution set is {x x < − 1 or x > 1 } or, ≥0
x−4
using interval notation, ( −∞, −1) ∪ (1, ∞ ) . 6
≥0
x−4
( x + 5) 2
38. ≥0 6
x2 − 4 f ( x) =
x−4
( x + 5) 2 The value where f is undefined is x = 4 .
≥0
( x + 2)( x − 2)
Interval (−∞, 4) (4, ∞)
( x + 5) 2
f ( x) = Number Chosen 0 5
x2 − 4
Value of f −1.5 6
The zeros and values where f is undefined are
Conclusion Negative Positive
x = − 5, x = − 2 and x = 2 .
Interval ( −∞, − 5) ( −5, −2) ( −2, 2) (2, ∞ )
The solution set is {x x > 4 } or, using interval
Number notation, ( 4, ∞ ) .
−6 −3 0 3
Chosen
Value of f 0.03125 0.8 −6.25 12.8
Conclusion Positive Positive Negative Positive

396
Copyright © 2015 Pearson Education, Inc.
Section 3.6: Polynomial and Rational Inequalities

3x − 5 1 2
41. ≤2 43. <
x+2 x−2 3x − 9
3x − 5 1 2
−2≤ 0 − <0
x+2 x − 2 3x − 9
3x − 5 − 2( x + 2) 3x − 9 − 2( x − 2)
≤0 <0
x+2 ( x − 2)(3 x − 9)
x−9 x−5
≤0 <0
x+2 ( x − 2)(3x − 9)
x−9 x−5
f ( x) = f ( x) =
x+2 ( x − 2)(3x − 9)
The zeros and values where f is undefined are
The zeros and values where f is undefined are
x = − 2 and x = 9 .
x = 2, x = 3, and x = 5 .
Interval (−∞, − 2) (−2, 9) (9, ∞)
Interval ( −∞ , 2) (2, 3) (3, 5) (5, ∞ )
Number
−3 0 10 Number
Chosen 0 2.5 4 6
Chosen
1
Value of f 12 −4.5 5 10 1 1
12 Value of f − −
Conclusion Positive Negative Positive 18 3 6 36
Conclusion Negative Positive Negative Positive
The solution set is {x − 2 < x ≤ 9 } or, using
interval notation, ( −2,9] .
The solution set is {x x < 2 or 3 < x < 5 } or,
using interval notation, ( −∞, 2 ) ∪ ( 3,5 ) .
x−4
42. ≥1 5 3
2x + 4 44. >
x−4 x − 3 x +1
−1 ≥ 0 5 3
2x + 4 − >0
x − 4 − 2x − 4 x − 3 x +1
≥0 5 x + 5 − 3x + 9
2x + 4 >0
x+8 ( x − 3)( x + 1)
≤0
2 ( x + 2) 2 ( x + 7)
>0
x +8 ( x − 3)( x + 1)
f ( x) =
2 ( x + 2) 2 ( x + 7)
f ( x) =
The zeros and values where f is undefined are ( x − 3)( x + 1)
x = − 8 and x = − 2 . The zeros and values where f is undefined are
Interval (−∞, − 8) (−8, −2) (−2, ∞) x = −7, x = −1, and x = 3 .
Number Interval ( −∞, − 7) ( −7, −1) ( −1, 3) (3, ∞ )
−9 −3 0
Chosen Number
−8 −2 0 4
1 Chosen
Value of f −2.5 2
14 2 14 22
Conclusion Positive Negative Positive Value of f − 2 −
77 3 5
The solution set is {x − 8 ≤ x < − 2 } or, using Conclusion Negative Positive Negative Positive

interval notation, [ −8, −2 ) . The solution set is {x − 7 < x < −1 or x > 3 } or,
using interval notation, ( −7, −1) ∪ ( 3, ∞ ) .

397
Copyright © 2015 Pearson Education, Inc.
Chapter 3: Polynomial and Rational Functions

x 2 (3 + x)( x + 4) (3 − x)3 (2 x + 1)
45. ≥0 47. <0
( x + 5)( x − 1) x3 − 1
(3 − x)3 (2 x + 1)
x 2 (3 + x)( x + 4) <0
f ( x) = ( x − 1)( x 2 + x + 1)
( x + 5)( x − 1)
The zeros and values where f is undefined are (3 − x)3 (2 x + 1)
f ( x) =
x = −5, x = − 4, x = −3, x = 0 and x = 1 . ( x − 1)( x 2 + x + 1)
The zeros and values where f is undefined are
Number
Interval Value of f Conclusion 1
Chosen
x = 3, x = − , and x = 1 .
2
216 Number
( −∞, − 5 ) −6 Positive Interval Value of f Conclusion
7 Chosen
243
( −5, −4 ) −4.5 − Negative  −∞, − 1 
44   −1 32 Positive
 2
49
( −4, − 3 ) −3.5 Positive  − 1 ,1 
108   0 −27 Negative
 2 
(−3, 0) −1 −0.75 Negative
(1, 3 ) 2 5/7 Positive
63
(0, 1) 0.5 − Negative ( 3, ∞ ) 4 −1 / 7 Negative
44

120 The solution set is x − 1 < x < 1 or x > 3  or,
(1, ∞ ) 2 Positive  2 
7
 1 
The solution set is using interval notation,  − ,1 ∪ (3, ∞) .
 2 
{ x x < − 5 or − 4 ≤ x ≤ −3 or x = 0 or x > 1 }
or, using interval notation, ( 2 − x )3 ( 3 x − 2 )
48. <0
( −∞, −5) ∪ [ −4, −3] ∪ {0} ∪ (1, ∞ ) . x3 + 1
( 2 − x )3 ( 3 x − 2 )
2
x( x + 1)( x − 2) <0
46. ≥0 ( x + 1) ( x 2 − x + 1)
( x − 1)( x + 1)
x( x 2 + 1)( x − 2) ( 2 − x )3 ( 3 x − 2 )
f ( x) = f ( x) =
( x − 1)( x + 1) ( x + 1) ( x 2 − x + 1)
The zeros and values where f is undefined are The zeros and values where f is undefined are
x = −1, x = 0, x = 1 and x = 2 . 2
x = 2, x = , and x = −1 .
Interval ( −∞ , − 1) ( −1, 0) (0,1) (1, 2) (2, ∞ )
3
Number
Number
−2 −0.5 0.5 1.5 3
Interval Value of f Conclusion
Chosen
Chosen

Value of f
40

25
1.25 −1.95 3.75
( −∞, − 1) −2 512 / 7 Positive
3 12
 2
Conclusion Positive Negative Positive Negative Positive  −1, 3  0 −16 Negative
 
The solution set is
2 
{ x x < −1 or 0 ≤ x < 1 or x ≥ 2 } or, using  , 2
3 
1 1/ 2 Positive

interval notation, ( −∞, −1) ∪ [ 0,1) ∪ [ 2,∞ ) . ( 2, ∞ ) 3 −1/ 4 Negative



The solution set is x −1 < x < 2 or x > 2  or,
 3 
 2
using interval notation,  −1,  ∪ (2, ∞) .
 3

398
Copyright © 2015 Pearson Education, Inc.
Section 3.6: Polynomial and Rational Inequalities

49. ( x + 1)( x − 3)( x − 5) > 0 52. x 2 + 3 x ≥ 10


f ( x) = ( x + 1)( x − 3)( x − 5) x 2 + 3x − 10 ≥ 0
x = −1, x = 3, x = 5 are the zeros of f. ( x + 5)( x − 2) ≥ 0
Interval ( −∞, −1) ( −1, 3) (3, 5) (5, ∞ ) f ( x) = x 2 + 3x − 10
Number x = −5, x = 2 are the zeros of f.
−2 0 4 6
Chosen
Value of f −35 15 −5 21
Interval (−∞, −5) ( −5, 2 ) ( 2, ∞ )
Number
Conclusion Negative Positive Negative Positive −6 0 3
Chosen
We want to know where f ( x) > 0 , so the
Value of f 8 −10 8
solution set is {x − 1 < x < 3 or x > 5 } or, using Conclusion Positive Negative Positive
interval notation, (−1, 3) ∪ (5, ∞) . We want to know where f ( x) ≥ 0 , so the
50. (2 x − 1)( x + 2)( x + 5) < 0 solution set is { x x ≤ −5 or x ≥ 2} or, using
f ( x) = (2 x − 1)( x + 2)( x + 5) interval notation, ( −∞, −5] ∪ [ 2, ∞ ) .
x = 1 , x = −2, x = −5 are the zeros of f.
2 x +1
53. ≤2
 1 1  x −3
Interval ( −∞, −5) ( −5, −2)  −2,   , ∞  x +1
 2 2  −2≤ 0
Number x−3
−6 −4 0 1
Chosen x + 1 − 2( x − 3)
≤0
Value of f −52 18 −10 18 x−3
Conclusion Negative Positive Negative Positive x + 1 − 2 x + 6)
≤0
We want to know where f ( x) < 0 , so the x −3
−x + 7
 1 ≤0
solution set is  x x < −5 or − 2 < x <  or, x −3
 2
−x + 7
 1 f ( x) =
using interval notation, (−∞, −5) ∪  −2,  . x −3
2  
The zeros and values where f is undefined are
51. 7 x − 4 ≥ −2 x 2 x = 3 and x = 7 .
2 x2 + 7 x − 4 ≥ 0 Interval (−∞, 3) (3, 7) (7, ∞)
(2 x − 1)( x + 4) ≥ 0 Number
1 5 8
2
Chosen
f ( x) = 2 x + 7 x − 4 1
Value of f −3 1 −
x = 1 , x = −4 are the zeros of f. 5
2 Conclusion Negative Positive Negative
 1 1 
Interval (−∞, −4)  −4,   , ∞ We want to know where f ( x) ≤ 0 , so the
 2 2 
Number −5 0 1
solution set is {x x < 3 or x ≥ 7 } or, using
Chosen interval notation, (−∞, 3) ∪ [7, ∞) . Note that 3
Value of f 11 −4 5
Conclusion Positive Negative Positive is not in the solution set because 3 is not in the
domain of f.
We want to know where f ( x) ≥ 0 , so the
 1
solution set is  x x ≤ −4 or x ≥  or, using
 2
1 
interval notation, ( −∞, −4 ∪  , ∞  .
2 

399
Copyright © 2015 Pearson Education, Inc.
Chapter 3: Polynomial and Rational Functions

x −1 56. ( x − 3)( x + 2) < x 2 + 3x + 5


54. ≥ −2
x+2 x 2 + 2 x − 3x − 6 < x 2 + 3x + 5
x −1 x 2 − x − 6 < x 2 + 3x + 5
+2≥0
x+2 − x − 6 < 3x + 5
x − 1 + 2( x + 2)
≥0 −4 x − 6 < 5
x+2
−4 x < 11
x −1+ 2x + 4
≥0 11
x+2 x>−
3x + 3 4
≥0  11 
x+2 The solution set is  x x > −  or, using
3( x + 1)  4
≥0
x+2  11 
interval notation,  − , ∞  .
3( x + 1)  4 
f ( x) =
x+2
The zeros and values where f is undefined are 6
57. 6x − 5 <
x = −2 and x = −1 . x
Interval (−∞, −2) (−2, −1) (−1, ∞) 6
6x − 5 − <0
Number 3 x
−3 − 1 6x2 − 5x − 6
Chosen 2 <0
Value of f 6 −3 2 x
Conclusion Positive Negative Positive (2 x − 3)(3 x + 2)
<0
We want to know where f ( x) ≥ 0 , so the x
(2 x − 3)(3 x + 2)
solution set is {x x < −2 or x ≥ −1 } or, using f ( x) =
x
interval notation, (−∞, − 2) ∪ [−1, ∞) . Note that The zeros and values where f is undefined are
−2 is not in the solution set because −2 is not in 2 3
the domain of f. x = − , x = 0 and x = .
3 2
55. 3( x 2 − 2) < 2( x − 1) 2 + x 2  2  2   3 3 
Interval  −∞, −   − , 0   0,   , ∞
2 2 2  3  3   2 2 
3x − 6 < 2( x − 2 x + 1) + x
Number −1 −0.5 1 2
3x 2 − 6 < 2 x 2 − 4 x + 2 + x 2 Chosen
3x 2 − 6 < 3x 2 − 4 x + 2 Value of f −5 4 −5 4
Conclusion Negative Positive Negative Positive
−6 < −4 x + 2
We want to know where f ( x) < 0 , so the
4x − 6 < 2
4x < 8  2 3
solution set is  x x < − or 0 < x <  or,
x<2  3 2
The solution set is { x x < 2} or, using interval  2  3
using interval notation,  −∞, −  ∪  0,  .
 3  2
notation, (−∞, 2) .
12
58. x+ <7
x
12
x+ −7 < 0
x
x 2 − 7 x + 12
<0
x
( x − 3)( x − 4)
<0
x
400
Copyright © 2015 Pearson Education, Inc.
Section 3.6: Polynomial and Rational Inequalities

( x − 3)( x − 4) 61. f ( x) = 2 x 4 + 11x3 − 11x 2 − 104 x − 48


f ( x) = ; The zeros and values
x a. Step 1: f ( x) has at most 4 real zeros.
where f is undefined are x = 0, x = 3 and x = 4 .
Step 2:Possible rational zeros:
Interval ( −∞ , 0) (0, 3) (3, 4) (4, ∞ ) p : ±1, ± 2, ± 3, ± 4, ± 6, ± 8,
Number ± 12, ± 24, ±48
−1 1 3.5 5
Chosen q : ±1, ± 2
1 p
Value of f −20 6 − 0.4 = ±1, ± 2, ± 3, ± 4, ± 6, ± 8, ± 12,
14 q
Conclusion Negative Positive Negative Positive 1 3
± 24, ± , ± , ±48
2 2
We want to know where f ( x) < 0 , so the
Step 3:From the graph it appears that there
solution set is {x x < 0 or 3 < x < 4 } or, using are x-intercepts at –4, –0.5, and 3.
interval notation, ( −∞, 0 ) ∪ ( 3,4 ) . Using synthetic division:
−4 2 11 − 11 − 104 − 48
3
59. x − 9x ≤ 0 −8 − 12 92 48
x( x − 3)( x + 3) ≤ 0 2 3 − 23 − 12 0
3
f ( x) = x − 9 x Since the remainder is 0, x + 4 is a
x = −3, x = 0, x = 3 are the zeros of f . factor. Using synthetic division again:
−4 2 3 − 23 − 12
Interval ( −∞, −3) ( −3, 0) (0, 3) (3, ∞ )
−8 20 12
Number
−4 −1 1 4 2 −5 −3 0
Chosen
Value of f −28 8 −8 28 Since the remainder is 0, x + 4 is also a
factor again. Thus,
Conclusion Negative Positive Negative Positive
f ( x) = ( x + 4) 2 (2 x 2 − 5 x − 3)
We want to know where f ( x) ≤ 0 , so the
= ( x − 4) 2 (2 x + 1)( x − 3)
solution set is { x | x ≤ −3 or 0 ≤ x ≤ 3} or, using
1
interval notation, ( −∞, −3] ∪ [ 0,3] . The zeros are –4, − , and 3.
2
b. The factoring is:
60. x3 − x ≥ 0 f ( x) = ( x − 4)2 (2 x + 1)( x − 3)
x( x − 1)( x + 1) ≥ 0
c.
f ( x) = x3 − x
x = −1, x = 0, x = 1 are the zeros of f .
Interval ( −∞, −1) ( −1, 0) (0, 1) (1, ∞ )
Number 1 1
−2 − 2
Chosen 2 2
Value of f −6 0.375 −0.375 6
Conclusion Negative Positive Negative Positive

We want to know where f ( x) ≥ 0 , so the


d. Looking at the graph we have f ( x) < 0 at
solution set is { x | −1 ≤ x ≤ 0 or x ≥ 1} or, using
 1 
interval notation, [ −1, 0] ∪ [1, ∞ ) .  − ,3 
 2 

401
Copyright © 2015 Pearson Education, Inc.
Chapter 3: Polynomial and Rational Functions

62. f ( x) = 4 x5 − 19 x 4 + 32 x3 − 31x 2 + 28 x − 12 Since the remainder is 0, 4 x − 3 is a


factor. Using synthetic division again:
a. Step 1: f ( x ) has at most 5 real zeros.
Thus,
Step 2:Possible rational zeros: f ( x) = ( x − 2) 2 (4 x3 − 3 x 2 + 4 x − 3)
p : ±1, ± 2, ± 3, ± 4, ± 6, ± 12
= ( x − 2) 2 (4 x − 3)( x 2 + 1)
q : ±1, ± 2, ± 4
3
p The zeros are 2 and .
= ±1, ± 2, ± 3, ± 4, ± 6, ± 12, 4
q
b. The factoring is:
1 3 1 3
± ,± ,± ,± f ( x) = ( x − 2)2 (4 x − 3)( x 2 + 1)
2 2 4 4
Step 3:From the graph it appears that there c.
are x-intercepts at 3/4 and 2.
Using synthetic division:
2 4 − 19 32 − 31 28 − 12
8 − 22 20 − 22 12
4 − 11 10 − 11 6 0
Since the remainder is 0, x − 2 is a
factor. Using synthetic division again:
2 4 − 11 10 − 11 6
8 −6 8 −6
4 −3 4 −3 0 d. Looking at the graph we have f ( x) < 0 at
Since the remainder is 0, x − 2 is also a
 3
factor again.  −∞, 
Using synthetic division again:  4
3
4 −3 4 −3
4
3 0 3
4 0 4 0

x2 + 5x − 6 ( x + 6)( x − 1)
63. a. R( x) = = p( x) = x 2 + 5 x − 6; q( x) = x 2 − 4; n = 2; m = 2
x2 − 4 x + 4 ( x − 2)( x − 2)

Step 1: Domain: {x x ≠ 2}
(0) 2 + 5(0) − 6 −6 3  3
The y-intercept is R (0) = = = − . Plot the point  0, −  .
(0) 2 − 4(0) + 4 4 2  2

( x + 6)( x − 1)
Step 2 & 3:In lowest terms, R ( x ) = , x ≠ 2 . The x-intercepts are the zeros of y = x + 6 and
( x − 2)( x − 2)
y = x − 1 : −6,1 ;

( x + 6)( x − 1)
Step 4: In lowest terms, R ( x ) = , x ≠ 2 . The vertical asymptote is the zero of f ( x ) = x − 2 :
( x − 2)( x − 2)
x = 2;
Graph this asymptote using a dashed line.

402
Copyright © 2015 Pearson Education, Inc.
Section 3.6: Polynomial and Rational Inequalities

Step 5: Since n = m , the line y = 1 is the horizontal asymptote. Graph this asymptote using a dashed line.
Solve to find intersection points:
x2 + 5x − 6
=1
x2 − 4 x + 4
x2 + 5x − 6 = x2 − 4x + 4
5 x − 6 = −4 x + 4
9 x = 10
10
x=
9

Steps 6 & 7: Graphing:

( x + 6)( x − 1)
b. ≥0
( x − 2)( x − 2)
The zeros and values where f is undefined are x = −6, x = 1, and x = 2 .
Number
Interval Value of f Conclusion
Chosen
216
( −∞, − 6 ) −6 Positive
7
243
( −6, −1) −4.5 − Negative
44
49
( −1, 2 ) −3.5 Positive
108
120
(2, ∞ ) 2 Positive
7
The solution set is {x x < − 6 or 1 ≤ x ≤ 2 or x > 2 } or, using interval notation,
( −∞, −6] ∪ [1, 2 ) ∪ ( 2, ∞ )
2 x2 + 9 x + 9 (2 x + 3)( x + 3)
64. a. R( x) = 2
= p( x) = 2 x 2 + 9 x + 9; q ( x) = x 2 − 4; n = 2; m = 2
x −4 ( x + 2)( x − 2)

Step 1: Domain: {x x ≠ 2, −2}


2(0) 2 + 9(0) + 9 9 9  9
The y-intercept is R (0) = 2
= = − . Plot the point  0, −  .
(0) − 4 −4 4  4

403
Copyright © 2015 Pearson Education, Inc.
Chapter 3: Polynomial and Rational Functions

(2 x + 3)( x + 3)
Step 2 & 3:In lowest terms, R ( x) = , x ≠ 2, −2 . The x-intercepts are the zeros of y = 2 x + 3 and
( x + 2)( x − 2)
3
y = x + 3 : − , −3 ;
2
(2 x + 3)( x + 3)
Step 4: In lowest terms, R ( x) = , x ≠ 2, −2 . The vertical asymptotes are the zeros of
( x + 2)( x − 2)
f ( x ) = x − 2 and f ( x ) = x + 2 : x = 2 and x = −2 ;
Graph these asymptotes using dashed lines.

Step 5: Since n = m , the line y = 1 is the horizontal asymptote. Graph this asymptote using a dashed line.
Solve to find intersection points:
2 x2 + 9 x + 9
=1
x2 − 4
x2 + 9 x + 9 = x2 − 4
9 x = −13
13
x=−
9

Steps 6 & 7: Graphing:

(2 x + 3)( x + 3)
b. ≥0
( x + 2)( x − 2)
3
The zeros and values where f is undefined are x = − , x = − 3, x = −2 and x = 2 .
2
Number
Interval Value of f Conclusion
Chosen
( −∞, − 3) −4 5
12 Positive
( −3, −2) −2.5 − 49 Negative
3 2
−2, − −1.75 3 Positive
2
3
− ,2 0 − 94 Negative
2
(2, ∞ ) 3 54 Positive
5

404
Copyright © 2015 Pearson Education, Inc.
Section 3.6: Polynomial and Rational Inequalities

 3 
The solution set is  x x < − 3 or − 2 < x ≤ − or x > 2  or, using interval notation,
 2 

( −∞, −3) ∪  −2, −  ∪ ( 2, ∞ )


3
 2

x3 + 2 x 2 − 11x − 12 ( x − 3)( x + 4)( x + 1) ( x + 4)( x + 1)


65. a. R( x) = 2
= = p ( x) = x3 + 2 x 2 − 11x − 12;
x − x−6 ( x − 3)( x + 2) ( x + 2)
q( x) = x 2 − x − 6; n = 3; m = 2

Step 1: Domain: {x x ≠ −2,3}


(0)3 + 2(0) 2 − 11(0) − 12 −12
The y-intercept is R (0) = = = 2 . Plot the point ( 0, 2 ) .
(0) 2 − (0) − 6 −6

( x + 4)( x + 1)
Step 2 & 3:In lowest terms, R ( x) = , x ≠ −2 . The x-intercepts are the zeros of y = x + 4 and
( x + 2)
y = x + 1 : −4, −1 ;
Note: x = 3 is not a zero because reduced form must be used to find the zeros.

( x + 4)( x + 1)
Step 4: In lowest terms, R ( x) = , x ≠ −2 . The vertical asymptote is the zero of f ( x ) = x + 2 :
( x + 2)
x = −2 ;
Graph this asymptote using a dashed line.

Step 5: Since n = m + 1 , there is an oblique asymptote. Dividing:


x+3
−2 x + 6
x 2 − x − 6 x3 + 2 x 2 − 11x − 12 G ( x) = x + 3 +
x2 − x − 6
x3 − x 2 − 6 x
3x 2 − 5x − 12
3x 2 − 3x − 18
− 2x + 6
The oblique asymptote is y = x + 3 . Graph this asymptote with a dashed line.

Steps 6 & 7: Graphing:

405
Copyright © 2015 Pearson Education, Inc.
Chapter 3: Polynomial and Rational Functions

( x − 3)( x + 4)( x + 1)
b. ≥0
( x − 3)( x + 2)
The zeros and values where f is undefined are x = −4, x = − 2, and x = −1 .
Number
Interval Value of f Conclusion
Chosen
4
( −∞, − 4 ) −5 − Negative
3
( −4, −2 ) −3 2 Positive
5
( −2, − 1) −1.5 − Negative
2
( −1, ∞ ) 0 2 Positive

The solution set is {x − 4 ≤ x < −2 or − 1 ≤ x < 3 or x > 3 } or, using interval notation,
[ −4, −2 ) ∪ [ −1,3) ∪ ( 3, ∞ )
x3 − 6 x 2 + 9 x − 4 ( x − 1)( x − 1)( x − 4) ( x − 1)( x − 1)
66. a. R( x) = 2
= = p( x) = x3 − 6 x 2 + 9 x − 4;
x + x − 20 ( x + 5)( x − 4) ( x + 5)
q ( x) = x 2 + x − 20; n = 3; m = 2

Step 1: Domain: {x x ≠ −5, 4}


(0)3 − 6(0) 2 + 9(0) − 4 −4 1  1
The y-intercept is R (0) = = = . Plot the point  0,  .
(0) 2 + (0) − 20 −20 5  5

( x − 1)( x − 1)
Step 2 & 3:In lowest terms, R ( x) = , x ≠ −5 . The x-intercept is the zero of y = x − 1 : 1 ;
( x + 5)
Note: x = 4 is not a zero because reduced form must be used to find the zeros.

( x − 1)( x − 1)
Step 4: In lowest terms, R ( x) = , x ≠ −5 . The vertical asymptote is the zero of f ( x ) = x + 5 :
( x + 5)
x = −5 ;
Graph this asymptote using a dashed line.

Step 5: Since n = m + 1 , there is an oblique asymptote. Dividing:


x−7
−2 x + 6
x 2 + x − 20 x3 − 6 x 2 + 9 x − 4 G ( x) = x + 3 +
x2 − x − 6
x3 + x 2 − 20 x
− 7 x 2 + 29 x − 4
− 7 x 2 − 7 x + 140
36 x − 144
The oblique asymptote is y = x − 7 . Graph this asymptote with a dashed line.

406
Copyright © 2015 Pearson Education, Inc.
Section 3.6: Polynomial and Rational Inequalities

Steps 6 & 7: Graphing:

( x − 1)( x − 1)( x − 4)
b. ≥0
( x + 5)( x − 4)
The zeros and values where f is undefined are x = −5, x = 1, and x = 4 .
Number
Interval Value of f Conclusion
Chosen
( −∞, − 5 ) −6 −49 Negative
1
( −5,1) 0 Positive
5
1
(1, 4 ) 3 Positive
2
8
(4, ∞ ) 5 Positive
5
The solution set is {x − 5 < x < 4 or x > 4 } or, using interval notation, ( −5, 4 ) ∪ ( 4, ∞ )

x 4 − 16 ( x − 4)( x + 4)
67. R( x) = =
x2 − 9 ( x − 3)( x + 3)
x2 + 9
x2 − 9 x4 − 16
4 2
x − 9x
9 x 2 − 16
9 x 2 − 81
65
As x approaches negative and positive infinity, the end behavior is y = x 2 .
x 4 − 16
We need R ( x) = < x2
x2 − 9

407
Copyright © 2015 Pearson Education, Inc.
Chapter 3: Polynomial and Rational Functions

x 4 − 16
< x2
x2 − 9
x 4 − 16
− x2 < 0
x2 − 9
x 4 − 16 x4 − 9 x2
− <0
x2 − 9 x2 − 9
9 x 2 − 16
<0
x2 − 9
(3 x − 4)(3 x + 4)
<0
( x − 3)( x + 3)
The vertical asymptotes are at x = ±5 . So we need to test a number in each interval.
Interval ( −∞, −3) (−3, ) ( ) ( ,3)

4
3
4 4
− ,
3 3
4
3
(3, ∞)
test value −4 −2 0 2 4
3x - 4 − − − + +
3x + 4 − − + + +
x−3 − − − − +
x+3 − + + + +
Sign of expression + − + − +
4 4
So R(x) is below its end behavior on the interval: −3, − ∪ ,3
3 3

x2 − 4 ( x − 2)( x + 2)
68. R ( x) = =
x 2 − 25 ( x − 5)( x + 5)

1
2 2 21
x − 4 x − 25 R( x) = 1 +
x2 − 4
x2 − 4
21
As x approaches negative and positive infinity, the end behavior (horizontal asymptote) is y = 1 .
x2 − 4
We need R ( x) = >1
x 2 − 25
x2 − 4
>1
x 2 − 25
x2 − 4
−1 > 0
x 2 − 25
x 2 − 4 − ( x 2 − 25)
>0
x 2 − 25
21
>0
x 2 − 25
The vertical asymptotes are at x = ±5 . So we need to test a number in each interval.

408
Copyright © 2015 Pearson Education, Inc.
Section 3.6: Polynomial and Rational Inequalities

Interval ( −∞, − 5) ( −5,5) (5, ∞)


Number
−6 0 6
Chosen
Value 21 / 11 −21 / 25 21 / 11
Conclusion Positive Negative Positive
So the solution set is: ( −∞, −5) ∪ (5, ∞)

69. Let x be the positive number. Then


71. The domain of f ( x) = x 4 − 16 consists of all
x3 > 4 x 2 real numbers x for which
x3 − 4 x 2 > 0 x 4 − 16 ≥ 0
2
x ( x − 4) > 0 ( x 2 + 4)( x 2 − 4) ≥ 0
f ( x) = x 2 ( x − 4)
x = 0 and x = 4 are the zeros of f .
(x 2
)+ 4 ( x − 2 )( x + 2 ) ≥ 0

p( x) = ( x 2
)
+ 4 ( x − 2 )( x + 2 )
Interval (−∞, 0) (0, 4) (4, ∞)
Number x = – 2 and x = 2 are the zeros of p .
−1 1 5
Chosen Interval (−∞, − 2) (−2, 2) (2, ∞)
Value of f −5 −3 25 Number
−3 0 3
Conclusion Negative Negative Positive Chosen
Since x must be positive, all real numbers Value of p 65 −16 65
greater than 4 satisfy the condition. The solution Conclusion Positive Negative Positive
set is { x x > 4 } or, using interval notation, The domain of f will be where p( x) ≥ 0 . Thus,
( 4, ∞ ) . the domain of f is {x x ≤ − 2 or x ≥ 2 } or, using

70. Let x be the positive number. Then interval notation, ( −∞, −2] ∪ [ 2, ∞ ) .
3
x <x 72. The domain of f ( x) = x3 − 3 x 2 consists of all
3
x −x<0 real numbers x for which
x ( x − 1)( x + 1) < 0 x3 − 3x 2 ≥ 0
f ( x ) = x ( x − 1)( x + 1) x 2 ( x − 3) ≥ 0
x = −1, x = 0, and x = 1 are the zeros of f. p ( x) = x 2 ( x − 3)
Interval ( −∞ , − 1) ( −1, 0) (0,1) (1, ∞ )
x = 0 and x = 3 are the zeros of p .

Number
Interval (−∞, 0) (0,3) (3, ∞)
−2 −1 / 2 1/ 2 2 Number
Chosen −1 1 4
Chosen
Value of f −6 0.375 −0.375 6
Value of p −4 −2 16
Conclusion Negative Positive Negative Positive
Conclusion Negative Negative Positive
Since x must be positive, all real numbers
The domain of f will be where p( x) ≥ 0 . Thus,
between (but not including) 0 and 1 satisfy the
condition. The solution set is { x 0 < x < 1 } or, the domain of f is { x x = 0 or x ≥ 3 } or, using

using interval notation, ( 0,1) . interval notation, {0} ∪ [3,∞ ) .

409
Copyright © 2015 Pearson Education, Inc.
Chapter 3: Polynomial and Rational Functions

x−2 x = −1 and x = 1 are the zeros of h.


73. The domain of f ( x) = includes all Interval (−∞, − 1) (−1,1) (1, ∞)
x+4
x−2 Number
−2 0 2
values for which ≥0. Chosen
x+4
Value of h 21 −3 21
x−2 Conclusion Positive Negative Positive
R( x) =
x+4
The zeros and values where R is undefined are f ( x ) ≤ g ( x ) if −1 ≤ x ≤ 1 . That is, on the
x = – 4 and x = 2 . interval [ −1,1] .
Interval (−∞, − 4) (−4, 2) (2, ∞)
Number
−5 0 3
Chosen
Value of R 7 −1 1
2 7
Conclusion Positive Negative Positive
The domain of f will be where R ( x) ≥ 0 . Thus,
the domain of f is {x x < − 4 or x ≥ 2 } or,
using interval notation, ( −∞, −4 ) ∪ [ 2, ∞ ) .
76. f ( x) ≤ g ( x)
x4 − 1 ≤ x − 1
x −1
74. The domain of f ( x) = includes all
x+4 x4 − x ≤ 0
values for which
x −1
(
x x3 − 1 ≤ 0)
x ( x − 1) ( x + x + 1) ≤ 0
≥0 2
x+4
h ( x ) = x ( x − 1) ( x + x + 1)
x −1 2
R( x) =
x+4
The zeros and values where the expression is x = 0 and x = 1 are the zeros of h.
undefined are x = – 4 and x = 1 . Interval (−∞, 0) (0,1) (1, ∞)
Interval (−∞, − 4) (−4,1) (1, ∞) Number
−1 1/ 2 2
Number −5 0 2 Chosen
Chosen
1 1 Value of h 2 −7 /16 14
Value of R 6 −
4 6 Conclusion Positive Negative Positive
Conclusion Positive Negative Positive
f ( x ) ≤ g ( x ) if 0 ≤ x ≤ 1 . That is, on the
The domain of f will be where R ( x) ≥ 0 . Thus,
interval [ 0,1] .
the domain of f is {x x < − 4 or x ≥ 1 } or, using
interval notation, ( −∞, −4 ) ∪ [1, ∞ ) .

75. f ( x) ≤ g ( x)
x 4 − 1 ≤ −2 x 2 + 2
x4 + 2 x2 − 3 ≤ 0
( x + 3)( x − 1) ≤ 0
2 2

( x + 3) ( x − 1)( x + 1) ≤ 0
2

h ( x ) = ( x + 3) ( x − 1)( x + 1)
2

410
Copyright © 2015 Pearson Education, Inc.
Section 3.6: Polynomial and Rational Inequalities

77. f ( x) ≤ g ( x) interval [ −1,1] .


4 2
x − 4 ≤ 3x
x − 3x 2 − 4 ≤ 0
4

(x 2
)( )
− 4 x2 + 1 ≤ 0

( x − 2 )( x + 2 ) ( x + 1) ≤ 0
2

h ( x ) = ( x − 2 )( x + 2 ) ( x + 1)2

x = −2 and x = 2 are the zeros of h.


Interval (−∞, − 2) (−2, 2) (2, ∞)
Number
−3 0 3 79. We need to solve C ( x) ≤ 100 .
Chosen
80 x + 5000
Value of h 50 −4 50 ≤ 100
x
Conclusion Positive Negative Positive 80 x + 5000 100 x
− ≤0
f ( x ) ≤ g ( x ) if −2 ≤ x ≤ 2 . That is, on the x x
5000 − 20 x
interval [ −2, 2] . ≤0
x
20(250 − x)
≤0
x
20 ( 250 − x )
f ( x) =
x
The zeros and values where the expression is
undefined are x = 0 and x = 250 .
Interval (−∞, 0) (0, 250) (250, ∞)
Number
−1 1 260
Chosen
Value of f −5020 4980 −10 /13
Conclusion Negative Positive Negative
78. f ( x) ≤ g ( x) The number of bicycles produced cannot be
4
x ≤ 2− x 2
negative, so the solution is { x x ≥ 250 } or,
using interval notation, [ 250, ∞ ) . The company
4 2
x + x −2≤ 0
( x + 2)( x − 1) ≤ 0
2 2 must produce at least 250 bicycles each day to
keep average costs to no more than $100.
( x + 2) ( x − 1)( x + 1) ≤ 0
2

80. We need to solve C ( x ) ≤ 100 .


h ( x ) = ( x + 2 ) ( x − 1)( x + 1)
2
80 x + 6000
x = −1 and x = 1 are the zeros of h. ≤ 100
x
Interval (−∞, − 1) (−1,1) (1, ∞) 80 x + 6000 100 x
Number − ≤0
−2 0 2 x x
Chosen
6000 − 20 x
Value of h 18 −2 18 ≤0
x
Conclusion Positive Negative Positive
20 ( 300 − x )
f ( x ) ≤ g ( x ) if −1 ≤ x ≤ 1 . That is, on the ≤0
x

411
Copyright © 2015 Pearson Education, Inc.
Chapter 3: Polynomial and Rational Functions

20 ( 300 − x ) mmoon mobj mearth mobj


f ( x) = G >G
( 384, 400 − r )
2
x r2
The zeros and values where the expression is
mmoon mearth
undefined are x = 0 and x = 300 . >
( 384, 400 − r )
2
Interval (−∞, 0) (0,300) (300, ∞) r2
Number mmoon mearth
−1 1 310 − >0
Chosen
( 384, 400 − r ) r
2 2

Value of f −6020 5980 −20 / 31


r 2 mmoon − ( 384, 400 − r ) mearth
2
Conclusion Negative Positive Negative
>0
r 2 ( 384, 400 − r )
2
The number of bicycles produced cannot be
negative, so the solution is { x x ≥ 300 } or, The zeros and values where the left-hand side is
using interval notation, [300, ∞ ) . The company undefined are r = 0 , r ≈ 432,353 ,
must produce at least 300 bicycles each day to r ≈ 346, 022 , and r = 384, 400 . Since the
keep average costs to no more than $100. distance from Earth to the object will be greater
than 0 but less than the distance to the moon, we
81. a. K ≥ 16 can exclude some of these values.
2 (150 )( S + 42 ) Interval (0, 346022) (346022,384400)
≥ 16 Number Chosen 100, 000 350, 000
S2
300S + 12, 600 Value of left side −6 × 1014 1.3 × 1013
≥ 16 Conclusion Negative Positive
S2
300 S + 12, 600 The gravitational force on the object due to the
− 16 ≥ 0 moon will be greater than the force due to the
S2
Earth when the object is more than 346,022
300 S + 12, 600 − 16S 2 kilometers from Earth.
≥0
S2
83. Let x represent the number of student that attend
Solve −16 S 2 + 300S + 12, 600 = 0 and
the play. Then the discounted price per ticket is
S 2 = 0 . The zeros and values where the 40 − 0.20x . Each student’s share of the bus cost is
left-hand side is undefined are S = 0 , 500
S ≈ 39 , S ≈ −20 . Since the stretch cannot . Thus, each student’s total cost will be
x
be negative, we only consider cases where
500
S ≥0. C ( x) = 40 − 0.20 x + .
x
Interval (0, 39) (39, ∞)
We need to solve C ( x) ≤ 40 .
Number Chosen 1 40
500
Value of left side 12884 −0.625 40 − 0.20 x + ≤ 40
x
Conclusion Positive Negative 500
−0.20 x + ≤0
The cord will stretch less than 39 feet. x
b. The safe height is determined by the 500
0.20 x − ≥0
minimum clearance (3 feet), the free length x
of the cord (42 feet), and the stretch in the 2
0.20 x − 500)
cord (39 feet). Therefore, the platform must ≥0
x
be at least 3 + 42 + 39 = 84 feet above the
ground for a 150-pound jumper. 0.20( x 2 − 2500)
≥0
x
82. Let r = the distance between Earth and the object 0.20( x + 50)( x − 50)
in kilometers. Then 384, 400 − r = the distance ≥0
x
between the object and the moon. We want

412
Copyright © 2015 Pearson Education, Inc.
Chapter 3 Review Exercises

2=k 9
0.20( x + 50)( x − 50)
f ( x) = 2
x 2 = 3k so k =
3
The zeros and values where f is undefined are
2
x = −50, x = 0 and x = 50 . Since the number of So y = x
3
students cannot be negative, we only consider
cases where x ≥ 0 . 90. f ( x) = x 2 + 3 x − 2
Interval (0, 50) (50, ∞) f ( x − 2) = ( x − 2)2 + 3( x − 2) − 2
Number Chosen 25 100
= x2 − 4 x + 4 + 3x − 6 − 2
Value of f −15 15
= x2 − x − 4
Conclusion Negative Positive
We are looking for where f ( x) ≥ 0 . Thus, the 91.

solution is {x x ≥ 50 } or, using interval notation, 6 x 4 y 4 + 3 x3 y 5 − 18 x 2 y 6 = 3x 2 y 4 (2 x 2 + xy − 6 y 2 )


[50, ∞ ) . If at least 50 students attend the play, the = 3x 2 y 4 ( x + 2 y )(2 x − 3 y )
price per student will be at or below $40.

84. Answers will vary, for example, x 2 < 0 has no


real solution and x 2 ≤ 0 has exactly one real
Chapter 3 Review Exercises
solution.
1. f ( x) = 4 x5 − 3 x 2 + 5 x − 2 is a polynomial of
85. x 4 + 1 < −5 has no solution because the quantity degree 5.
x 4 + 1 is never negative. ( x 4 + 1 ≥ 1 )
3 x5
86. No, the student is not correct. For example, 2. f ( x) = is a rational function. It is not a
2x +1
x = −5 is in the solution set, but does not satisfy
polynomial because there are variables in the
the original inequality. denominator.
−5 + 4 −1 1
= = ≤0
−5 − 3 −8 8 3. f ( x) = 3 x 2 + 5 x1/ 2 − 1 is not a polynomial
When multiplying both sides of an inequality by 1
a negative, we must switch the direction of the because the variable x is raised to the power,
2
inequality. Since we do not know the sign of
which is not a nonnegative integer.
x + 3 , we cannot multiply both sides of the
inequality by this quantity. 4. f ( x) = 3 is a polynomial of degree 0.
x −5
87. Answers will vary. One example: ≤0 5. f ( x) = ( x + 2)3
x+3
Using the graph of y = x3 , shift left 2 units.
88. 9 − 2 x ≤ 4 x + 1
4
−6 x ≤ −8 → x ≥
3
4
So the solution is: ,∞
3

89. Since we have y varying directly with x then


y = k x . When x = 9, then y = 2 so we have

413
Copyright © 2015 Pearson Education, Inc.
Chapter 3: Polynomial and Rational Functions

6. f ( x) = − ( x − 1) 4 Near −2 :
f ( x ) ≈ −2 ( x + 2 )( −2 + 4 ) = −4 ( x + 2 )
Using the graph of y = x 4 , shift right 1 unit,
(a line with slope −4 )
then reflect about the x-axis.
Near 0: f ( x ) ≈ x ( 0 + 2 )( 0 + 4 ) = 8 x
(a line with slope 8)
Step 6: Graphing:

7. f ( x) = ( x − 1) 4 + 2
Using the graph of y = x 4 , shift right 1 unit,
then shift up 2 units.
9. f ( x) = ( x − 2) 2 ( x + 4)

Step 1: Degree is 3. The function resembles


y = x3 for large values of x .

Step 2: y-intercept:
2
f ( 0) = ( 0 − 2) ( 0 + 4 ) = 16
x-intercepts: solve f ( x) = 0 :
( x − 2) 2 ( x + 4) = 0  x = 2 or x = −4
Step 3: The graph crosses the x-axis at x = –4
8. f ( x) = x( x + 2)( x + 4) since this zero has multiplicity 1. The
Step 1: Degree is 3. The function resembles graph touches the x-axis at x = 2
since this zero has multiplicity 2.
y = x3 for large values of x .
Step 4: The polynomial is of degree 3 so the
Step 2: y-intercept: graph has at most 3 − 1 = 2 turning
f ( 0 ) = ( 0 )( 0 + 2 ) ( 0 + 4 ) = 0 points.
x-intercepts: solve f ( x) = 0 : Step 5: Near −4 :
x( x + 2)( x + 4) = 0 f ( x ) ≈ ( −4 − 2 )
2
( x + 4 ) = 36 ( x + 4 )
x = 0 or x = −2 or x = −4 (a line with slope 36)
Step 3: The graph crosses the x-axis at x =
Near 2:
−4 , x = −2 and x = 0 since each 2 2
zero has multiplicity 1. f ( x ) ≈ ( x − 2) ( 2 + 4) = 6 ( x − 2)
Step 4: The polynomial is of degree 3 so the (a parabola opening upward)
graph has at most 3 − 1 = 2 turning
points.
Step 5: Near −4 :
f ( x ) ≈ −4 ( −4 + 2 )( x + 4 ) = 8 ( x + 4 )
(a line with slope 8)

414
Copyright © 2015 Pearson Education, Inc.
Chapter 3 Review Exercises

Step 6: Graphing: Step 2: y-intercept:


2
f ( 0 ) = ( 0 − 1) ( 0 + 3) ( 0 + 1)
=3
x-intercepts: solve f ( x) = 0 :
( x − 1)2 ( x + 3)( x + 1) = 0
x = 1 or x = −3 or x = −1
Step 3: The graph crosses the x-axis at
x = −3 and x = −1 since each zero
has multiplicity 1.The graph touches
the x-axis at x = 1 since this zero has
multiplicity 2.
Step 4: The polynomial is of degree 4 so the
graph has at most 4 − 1 = 3 turning
10. f ( x) = −2 x3 + 4 x 2 = −2 x 2 ( x − 2 ) points.
Step 5: Near −3 :
Step 1: Degree is 3. The function resembles 2
f ( x ) ≈ ( −3 − 1) ( x + 3)( −3 + 1) = −32 ( x + 3)
y = −2 x3 for large values of x .
(a line with slope −32 )
Step 2: x-intercepts: 0, 2; y-intercept: 0
Near −1 :
Step 3: The graph crosses x axis at x = 2 2
f ( x ) ≈ ( −1 − 1) ( −1 + 3)( x + 1) = 8 ( x + 1)
since it has multiplicity 2 and touches
the x axis at x = 0 since it has (a line with slope 8)
multiplicity 1.
Near 1:
Step 4: The polynomial is of degree 3 so the 2 2
graph has at most 3 − 1 = 2 turning f ( x ) ≈ ( x − 1) (1 + 3)(1 + 1) = 8 ( x − 1)
points. (a parabola opening upward)
Step 5: Near 0: f ( x ) ≈ −2 x 2 ( 0 − 2 ) = 4 x 2
Step 6: Graphing:
(a parabola opening upward)
2
Near 2: f ( x ) ≈ −2 ( 2 ) ( x − 2 ) = −8 ( x − 2 )
(a line with slope −8 )

Step 6: Graphing:

12. f ( x) = 8 x3 − 3 x 2 + x + 4
Since g ( x) = x − 1 then c = 1 . From the
Remainder Theorem, the remainder R when
2 f ( x ) is divided by g ( x ) is f (c ) :
11. f ( x) = ( x − 1) ( x + 3) ( x + 1)
f (1) = 8(1)3 − 3(1) 2 + 1 + 4
Step 1: Degree is 4. The function resembles = 8 − 3 +1+ 4
y = x 4 for large values of x . = 10
So R = 10 and g is not a factor of f .

415
Copyright © 2015 Pearson Education, Inc.
Chapter 3 Polynomial and Rational Functions

13. f ( x) = x 4 − 2 x3 + 15 x − 2 Possible rational zeros:


p = ±1, ± 2, ± 4, ± 8; q = ±1;
Since g ( x) = x + 2 then c = −2 . From the
p
Remainder Theorem, the remainder R when = ±1, ± 2, ± 4, ± 8
f ( x ) is divided by g ( x ) is f (c ) : q

f (−2) = (−2) 4 − 2(−2)3 + 15(−2) − 2 Using synthetic division:


= 16 − 2(−8) − 30 − 2 We try x + 2 :
=0 −2 1 −3 −6 8
So R = 0 and g is a factor of f . − 2 10 − 8
1 −5 4 0
14. 4 12 0 −8 0 0 0 1
x + 2 is a factor. The other factor is the
48 192 736 2944 11,776 47,104
quotient: x 2 − 5 x + 4 .
12 48
f (4) = 47,105
184 736 2944 11,776 47,105
(
Thus, f ( x) = ( x + 2) x 2 − 5 x + 4 . )
= ( x + 2)( x − 1)( x − 4)
15. f ( x ) = 12 x − x + 8 x − 2 x + x + 3
8 7 4 3 The zeros are –2, 1, and 4, each of multiplicity 1.
There are 4 sign changes in f ( x ) , so there are 4 19. f ( x) = 4 x3 + 4 x 2 − 7 x + 2
positive real zeros, 2 positive real zeros, or no By Descartes’ Rule of Signs, there are two
positive real zeros. positive real zeros or no positive real zeros.
8 7 4 3 f (− x) = 4(− x)3 + 4(− x) 2 − 7(− x) + 2 ;
f ( − x ) = 12 ( − x ) − ( − x ) + 8 ( − x ) − 2 ( − x ) + ( − x ) + 3
= − 4 x3 + 4 x 2 + 7 x + 2
= 12 x8 + x 7 + 8 x 4 + 2 x3 − x + 3
thus, there is one negative real zero.
There are 2 sign changes in f ( − x ) , so there are
Possible rational zeros:
2 negative real zeros or no negative real zeros. p = ±1, ± 2; q = ±1, ± 2, ± 4;
16. f ( x ) = −6 x5 + x 4 + 5 x3 + x + 1 p 1 1
= ±1, ± 2, ± , ±
q 2 4
There is 1 sign change in f ( x ) so there is
exactly one positive real zero. Using synthetic division:
5 4 3 We try x + 2 :
f ( − x ) = −6 ( − x ) + ( − x ) + 5 ( − x ) + ( − x ) + 1
−2 4 4 −7 2
= 6 x5 + x 4 − 5 x3 − x + 1 −8 8 −2
There are 2 sign changes in f ( − x ) , so there are
4 −4 1 0
2 negative real zeros or no negative real zeros.
x + 2 is a factor. The other factor is the quotient:
17. a0 = −3 , a8 = 12 4 x2 − 4 x + 1 .
p = ±1, ±3
q = ±1, ±2, ±3, ±4, ±6, ±12
(
Thus, f ( x) = ( x + 2 ) 4 x 2 − 4 x + 1 . )
= ( x + 2 ) (2 x − 1)(2 x − 1)
p 1 3 1 1 3 1 1
= ±1, ±3, ± , ± , ± , ± , ± , ± , ± 1
q 2 2 3 4 4 6 12 The zeros are –2, of multiplicity 1 and , of
2
18. f ( x) = x3 − 3x 2 − 6 x + 8 multiplicity 2.
By Descartes’ Rule of Signs, there are two 20. f ( x) = x 4 − 4 x3 + 9 x 2 − 20 x + 20
positive real zeros or no positive real zeros.
By Descartes’ Rule of Signs, there are four
f (− x) = (− x)3 − 3(− x) 2 − 6(− x) + 8 , there is one positive real zeros or two positive real zeros or
= − x3 − 3 x 2 + 6 x + 8 no positive real zeros.
negative real zero.

416
Copyright © 2015 Pearson Education, Inc.
Chapter 3 Review Exercises

f (− x) = (− x) 4 − 4(− x)3 + 9(− x) 2 − 20(− x) + 20 Thus, f ( x) = ( x + 3)( x − 2) 2 x 2 + 1 .( )


= x 4 + 4 x3 + 9 x 2 + 20 x + 20; 2
Since 2 x + 1 = 0 has no real solutions, the
Thus, there are no negative real zeros. solution set is {−3, 2} .
Possible rational zeros:
p = ±1, ± 2, ± 4, ± 5, ± 10, ± 20; q = ±1; 22. 2 x 4 + 7 x3 + x 2 − 7 x − 3 = 0
p The solutions of the equation are the zeros of
= ±1, ± 2, ± 4, ± 5, ± 10, ± 20 f ( x) = 2 x 4 + 7 x3 + x 2 − 7 x − 3 .
q
By Descartes’ Rule of Signs, there is one
Using synthetic division:
positive real zero.
We try x − 2 :
f (− x) = 2(− x) 4 + 7(− x)3 + (− x) 2 − 7(− x) − 3
2 1 −4 9 − 20 20
= 2 x 4 − 7 x3 + x 2 + 7 x − 3
2 −4 10 − 20
Thus, there are three negative real zeros or there
1 −2 5 − 10 0 is one negative real zero.
x − 2 is a factor and the quotient is Possible rational zeros:
x3 − 2 x 2 + 5 x − 10 = x 2 ( x − 2 ) + 5 ( x − 2 ) p = ±1, ± 3; q = ±1, ± 2;

( )
= ( x − 2) x2 + 5
p
q
1
= ±1, ± 3, ± , ±
2
3
2
Thus, f ( x) = ( x − 2)( x − 2) ( x + 5 )
2
Using synthetic division:
= ( x − 2) ( x + 5 )
2 2 We try x + 3 :
−3 2 7 1 −7 −3
Since x 2 + 5 = 0 has no real solutions, the only
−6 −3 6 3
zero is 2, of multiplicity 2.
2 1 − 2 −1 0
21. 2 x 4 + 2 x3 − 11x 2 + x − 6 = 0 x + 3 is a factor and the quotient is
The solutions of the equation are the zeros of
2x3 + x 2 − 2 x − 1 = x 2 ( 2 x + 1) − 1( 2 x + 1) .
f ( x) = 2 x 4 + 2 x3 − 11x 2 + x − 6 .
By Descartes’ Rule of Signs, there are three (
= ( 2 x + 1) x 2 − 1 )
positive real zeros or there is one positive real
zero. Thus,
f ( x) = ( x + 3) ( 2 x + 1) x 2 − 1 ( )
f (− x) = 2(− x) 4 + 2(− x)3 − 11(− x) 2 + (− x) − 6 = ( x + 3)( 2 x + 1)( x − 1)( x + 1)
= 2 x 4 − 2 x3 − 11x 2 − x − 6  1 
The solution set is −3, − 1, − , 1 .
Thus, there is one negative real zero.  2 
Possible rational zeros:
23. f ( x) = x3 − x 2 − 4 x + 2
p = ±1, ± 2, ± 3, ± 6; q = ±1, ± 2;
p 1 3 r coeff of q(x) remainder
= ±1, ± 2, ± 3, ± 6, ± , ±
q 2 2 1 1 0 −4 −2
Using synthetic division: 2 1 1 −2 −2
We try x + 3 : 3 1 2 2 8
−3 2 2 − 11 1 −6 −1 1 −2 −2 2
−6 12 − 3 6 −2 1 −3 2 −2
2 −4 1 −2 0
For r = 3, the last row of synthetic division
x + 3 is a factor and the quotient is contains only numbers that are positive or 0, so
2x3 − 4 x 2 + x − 2 = 2 x 2 ( x − 2 ) + 1( x − 2 ) we know there are no zeros greater than 3. For r
= -2, the last row of synthetic division results in
(
= ( x − 2) 2 x2 + 1 ) alternating positive (or 0) and negative (or 0)
values, so we know that there are no zeros less

417
Copyright © 2015 Pearson Education, Inc.
Chapter 3 Polynomial and Rational Functions

than -2. The upper bound is 3 and the lower 28. Since complex zeros appear in conjugate pairs,
bound is -2. −i , the conjugate of i , and 1 − i , the conjugate
of 1 + i , are the remaining zeros of f .
24. f ( x) = 2 x3 − 7 x 2 − 10 x + 35
f ( x ) = ( x − i )( x + i )( x − 1 − i )( x − 1 + i )
r coeff of q(x) remainder
= x 4 − 2 x3 + 3x 2 − 2 x + 2
1 2 −5 −15 20
3 2 −1 −13 −4 29. f ( x) = x3 − 3 x 2 − 6 x + 8 .
5 2 3 5 60 By Descartes’ Rule of Signs, there are two
positive real zeros or no positive real zeros.
−1 2 −9 −1 36 3 2
f (− x) = ( − x ) − 3 ( − x ) − 6 ( − x ) + 8
−2 2 −11 12 11
−3 2 −13 29 −52 = − x3 − 3x 2 + 6 x + 8
thus, there is one negative real zero.
For r = 5, the last row of synthetic division
contains only numbers that are positive or 0, so Possible rational zeros:
we know there are no zeros greater than 5. For r p = ±1, ± 2, ± 4, ± 8; q = ±1;
= -3, the last row of synthetic division results in p
alternating positive (or 0) and negative (or 0) = ±1, ± 2, ± 4, ± 8
q
values, so we know that there are no zeros less
than -3. The upper bound is 5 and the lower Using synthetic division:
bound is -3. We try x − 1 :
1 1 −3 −6 8
25. f ( x) = 3x3 − x − 1; [0, 1] 1 − 2 −8
f (0) = −1 < 0 and f (1) = 1 > 0
1 − 2 −8 0
The value of the function is positive at one
endpoint and negative at the other. Since the x − 1 is a factor and the quotient is x 2 − 2 x − 8
function is continuous, the Intermediate Value Thus,
Theorem guarantees at least one zero in the
given interval. ( )
f ( x) = ( x − 1) x 2 − 2 x − 8 = ( x − 1)( x − 4)( x + 2) .
The complex zeros are 1, 4, and –2, each of
26. f ( x) = 8 x 4 − 4 x3 − 2 x − 1; [0, 1] multiplicity 1.
f (0) = −1 < 0 and f (1) = 1 > 0
The value of the function is positive at one 30. f ( x) = 4 x3 + 4 x 2 − 7 x + 2 .
endpoint and negative at the other. Since the By Descartes’ Rule of Signs, there are two
function is continuous, the Intermediate Value positive real zeros or no positive real zeros.
Theorem guarantees at least one zero in the 3 2
f (− x) = 4 ( − x ) + 4 ( − x ) − 7 ( − x ) + 2
given interval.
= −4 x3 + 4 x 2 + 7 x + 2
27. Since complex zeros appear in conjugate pairs, Thus, there is one negative real zero.
4 − i , the conjugate of 4 + i , is the remaining
zero of f . Possible rational zeros:
p = ±1, ± 2; q = ±1, ±2, ±4;
f ( x ) = ( x − 6 )( x − 4 − i )( x − 4 + i )
p 1 1
= x3 − 14 x 2 + 65 x − 102 = ±1, ± , ± , ±2
q 2 4

Using synthetic division:


We try x + 2 :
−2 4 4 −7 2
−8 8 − 2
4 −4 1 0
x + 2 is a factor and the quotient is 4x 2 − 4 x + 1 .

418
Copyright © 2015 Pearson Education, Inc.
Chapter 3 Review Exercises

Thus, Possible rational zeros:


( 2
f ( x) = ( x + 2) 4 x − 4 x + 1 ) . p = ±1, ± 2, ± 3, ± 6; q = ±1, ±2;
p 1 3
= ( x + 2 )( 2 x − 1)( 2 x − 1) = ±1, ± , ±2, ± 3, ± ± 6
q 2 2
( )
2 2
= ( x + 2 )( 2 x − 1) = 4 ( x + 2 ) x + 12 Using synthetic division:
The complex zeros are –2, of multiplicity 1, and We try x − 2 :
1 2 2 2 − 11 1 − 6
, of multiplicity 2.
2 4 12 2 6
2 6 1 3 0
31. f ( x) = x 4 − 4 x3 + 9 x 2 − 20 x + 20 .
x − 2 is a factor and the quotient is
By Descartes’ Rule of Signs, there are four
positive real zeros or two positive real zeros or 2 x3 + 6 x 2 + x + 3 .
no positive real zeros.
4 3 2
(
Thus, f ( x) = ( x − 2) 2 x3 + 6 x 2 + x + 3 . )
f (− x) = ( − x ) − 4 ( − x ) + 9 ( − x ) − 20 ( − x ) + 20
We can factor 2 x3 + 6 x 2 + x + 3 by grouping.
4 3 2
= x + 4 x + 9 x + 20 x + 20 2 x 3 + 6 x 2 + x + 3 = 2 x 2 ( x + 3) + ( x + 3)
thus, there are no negative real zeros.
Possible rational zeros:
( )
= ( x + 3) 2 x 2 + 1

p = ±1, ± 2, ± 4, ± 5, ±10, ±20; q = ±1; = ( x + 3) ( 2 x + i )( 2 x − i )


p
= ±1, ± 2, ± 4, ± 5, ±10, ±20 f ( x) = ( x − 2 )( x + 3) ( 2 x + i )( 2 x − i )
q

Using synthetic division: ( )(


= 2 ( x − 2 )( x + 3) x + 22 i x − 22 i )
We try x − 2 : 2 2
2 1 −4 9 − 20 20 The complex zeros are 2, –3, − i , and i,
2 2
2 −4 10 − 20 each of multiplicity 1.
1 − 2 5 − 10 0 x+2 x+2
x − 2 is a factor and the quotient is 33. R ( x) = = is in lowest terms.
x −9 2 ( x + 3)( x − 3)
x3 − 2 x 2 + 5 x − 10 . The denominator has zeros at –3 and 3. Thus,
(
Thus, f ( x) = ( x − 2) x3 − 2 x 2 + 5 x − 10 . ) the domain is { x x ≠ −3, x ≠ 3} . The degree of
3 2 the numerator, p( x) = x + 2, is n = 1 . The
We can factor x − 2 x + 5 x − 10 by grouping.
x3 − 2 x 2 + 5 x − 10 = x 2 ( x − 2 ) + 5 ( x − 2 ) degree of the denominator
q ( x) = x 2 − 9, is m = 2 . Since n < m , the line
( )
= ( x − 2) x2 + 5
y = 0 is a horizontal asymptote. Since the
= ( x − 2 ) ( x + 5i )( x − 5i ) denominator is zero at –3 and 3, x = –3 and x = 3
are vertical asymptotes.
f ( x) = ( x − 2) 2 ( x + 5i )( x − 5i )
The complex zeros are 2, of multiplicity 2, and x2 + 4
34. R ( x ) = is in lowest terms. The
5i and − 5i , each of multiplicity 1. x−2
denominator has a zero at 2. Thus, the domain is
32. f ( x) = 2 x 4 + 2 x3 − 11x 2 + x − 6 . { x x ≠ 2} . The degree of the numerator,
By Descartes’ Rule of Signs, there are 3 positive p( x) = x 2 + 4, is n = 2 . The degree of the
real zeros or there is one positive real zero.
4 3 2 denominator, q( x) = x − 2, is m = 1 . Since
f (− x) = 2 ( − x ) + 2 ( − x ) − 11( − x ) + ( − x ) − 6
n = m + 1 , there is an oblique asymptote.
= 2 x 4 − 2 x3 − 11x 2 − x − 6
thus, there is one negative real zero. Dividing:

419
Copyright © 2015 Pearson Education, Inc.
Chapter 3 Polynomial and Rational Functions

x+2 2
q( x) = ( x + 2 ) = x 2 + 4 x + 4, is m = 2 . Since
2
x−2 x + 4 1
2 n = m , the line y = = 1 is a horizontal
x − 2x 1
2x + 4 x +1
asymptote. Since the denominator of y =
2x − 4 x+2
is zero at −2 , x = −2 is a vertical asymptote.
8

8
R( x) = x + 2 +
x−2

Thus, the oblique asymptote is y = x + 2 . Since


the denominator is zero at 2, x = 2 is a vertical
asymptote.

x2 + 3x + 2 ( x + 2 )( x + 1) x + 1
35. R ( x) = = = is in
( x + 2) 2
( x + 2 )2 x+2
lowest terms. The denominator has a zero at –2.
Thus, the domain is { x x ≠ −2} . The degree of
the numerator, p ( x) = x 2 + 3 x + 2, is n = 2 . The
degree of the denominator,

2x − 6
36. R ( x) = p ( x) = 2 x − 6; q ( x) = x; n = 1; m = 1
x

Step 1: Domain: { x x ≠ 0}
There is no y-intercept because 0 is not in the domain.

2 x − 6 2 ( x − 3)
Step 2: R( x) = = is in lowest terms.
x x

Step 3: The x-intercept is the zero of p( x) : 3 with odd multiplicity.


Plot the point ( 3, 0 ) . The graph crosses the x-axis at this point.

2 x − 6 2 ( x − 3)
Step 4: R( x) = = is in lowest terms. The vertical asymptote is the zero of q( x) : x = 0 .
x x
Graph this asymptote using a dashed line.

2
Step 5: Since n = m , the line y = = 2 is the horizontal asymptote. Solve to find intersection points:
1
2x − 6
=2
x
2x − 6 = 2x
−6 ≠ 0
R ( x) does not intersect y = 2 . Plot the line y = 2 with dashes.

Step 6:

420
Copyright © 2015 Pearson Education, Inc.
Chapter 3 Review Exercises

Steps 7 & 8: Graphing:

x+2
37. H ( x) = p( x) = x + 2; q( x) = x( x − 2) = x 2 − 2 x; n = 1; m = 2
x( x − 2)

Step 1: Domain: { x x ≠ 0, x ≠ 2} .

x+2
Step 2: H ( x) = is in lowest terms.
x( x − 2)

Step 3: There is no y-intercept because 0 is not in the domain.

The x-intercept is the zero of p( x) : –2 with odd multiplicity.


Plot the point ( −2, 0 ) . The graph crosses the x-axis at this point.

x+2
Step 4: H ( x) = is in lowest terms. The vertical asymptotes are the zeros of q( x) : x = 0 and x = 2 .
x( x − 2)
Graph these asymptotes using dashed lines.
Step 5: Since n < m , the line y = 0 is the horizontal asymptote. Solve to find intersection points:
x+2
=0
x( x − 2)
x+2=0
x = −2
H ( x) intersects y = 0 at (–2, 0). Plot the line y = 0 using dashes.

421
Copyright © 2015 Pearson Education, Inc.
Chapter 3 Polynomial and Rational Functions

Step 6:

Steps 7 & 8: Graphing:

x2 + x − 6 ( x + 3)( x − 2)
38. R ( x) = = p( x) = x 2 + x − 6; q ( x) = x 2 − x − 6;
2
x − x−6 ( x − 3)( x + 2)

Step 1: Domain: { x x ≠ − 2, x ≠ 3} .

x2 + x − 6
Step 2: R( x) = is in lowest terms.
x2 − x − 6

02 + 0 − 6 −6
Step 3: The y-intercept is R(0) = = = 1 . Plot the point ( 0,1) .
2
0 −0−6 −6

The x-intercepts are the zeros of p ( x) : –3 and 2 each with odd multiplicity.
Plot the points ( −3, 0 ) and ( 2, 0 ) . The graph crosses the x-axis at these points.

x2 + x − 6
Step 4: R( x) = is in lowest terms. The vertical asymptotes are the zeros of q( x) :
x2 − x − 6
x = −2 and x = 3 . Graph these asymptotes with dashed lines.

422
Copyright © 2015 Pearson Education, Inc.
Chapter 3 Review Exercises

1
Step 5: Since n = m , the line y = = 1 is the horizontal asymptote. Solve to find intersection points:
1
2
x + x−6
=1
x2 − x − 6
x2 + x − 6 = x2 − x − 6
2x = 0
x=0
R ( x) intersects y = 1 at (0, 1). Plot the line y = 1 using dashes.

Step 6:

Steps 7 & 8: Graphing:

x3 x3
39. F ( x) = = p( x) = x3 ; q( x) = x 2 − 4; n = 3; m = 2
x2 − 4 ( x + 2) ( x − 2)
Step 1: Domain: { x x ≠ − 2, x ≠ 2} .

x3
Step 2: F ( x) = is in lowest terms.
x2 − 4

03 0
Step 3: The y-intercept is F (0) = = = 0 . Plot the point ( 0, 0 ) .
2
0 −4 −4

The x-intercept is the zero of p( x) : 0 with odd multiplicity.


Plot the point ( 0, 0 ) . The graph crosses the x-axis at this point.

423
Copyright © 2015 Pearson Education, Inc.
Chapter 3 Polynomial and Rational Functions

x3
Step 4: F ( x) = is in lowest terms. The vertical asymptotes are the zeros of q( x) : x = − 2 and x = 2 .
x2 − 4
Graph these asymptotes using dashed lines.
Step 5: Since n = m + 1 , there is an oblique asymptote. Dividing:
x
x3 4x
x − 4 x3
2
2
= x+ 2
x −4 x −4
x3 − 4x
4x
The oblique asymptote is y = x . Solve to find intersection points:
x3
=x
x2 − 4
x3 = x3 − 4 x
4x = 0
x=0
F ( x) intersects y = x at (0, 0). Plot the line y = x using dashed lines.

Step 6:

Steps 7 & 8: Graphing:

2 x4
40. R( x) = p( x) = 2 x 4 ; q( x) = ( x − 1) 2 ; n = 4; m = 2
( x − 1) 2

Step 1: Domain: { x x ≠ 1} .

424
Copyright © 2015 Pearson Education, Inc.
Chapter 3 Review Exercises

2x4
Step 2: R ( x) = is in lowest terms.
( x − 1) 2

2(0) 4 0
Step 3: The y-intercept is R(0) = 2
= = 0 . Plot the point ( 0, 0 ) .
(0 − 1) 1

The x-intercept is the zero of p( x) : 0 with even multiplicity.


Plot the point ( 0, 0 ) . The graph touches the x-axis at this point.

2x4
Step 4: R ( x) = is in lowest terms. The vertical asymptote is the zero of q( x) : x = 1 .
( x − 1) 2
Graph this asymptote using a dashed line.
Step 5: Since n > m + 1 , there is no horizontal asymptote and no oblique asymptote.

Step 6:

Steps 7 & 8: Graphing:

x2 − 4 ( x + 2)( x − 2) x + 2
41. G ( x) = = = p ( x) = x 2 − 4; q( x) = x 2 − x − 2;
2
x −x−2 ( x − 2)( x + 1) x + 1

Step 1: Domain: { x x ≠ − 1, x ≠ 2} .

x+2
Step 2: In lowest terms, G ( x) = , x≠2.
x +1

425
Copyright © 2015 Pearson Education, Inc.
Chapter 3 Polynomial and Rational Functions

02 − 4 −4
Step 3: The y-intercept is G (0) = = = 2 . Plot the point ( 0, 2 ) .
2
0 −0−2 −2

The x-intercept is the zero of y = x + 2 : – 2 with odd multiplicity; Note: 2 is not a zero because
reduced form must be used to find the zeros.
Plot the point ( −2, 0 ) . The graph crosses the x-axis at this point.

x+2
Step 4: In lowest terms, G ( x) = , x ≠ 2 . The vertical asymptote is the zero of f ( x) = x + 1 : x = −1 ;
x +1
Graph this asymptote using a dashed line. Note: x = 2 is not a vertical asymptote because reduced form
 4
must be used to find the asymptotes. The graph has a hole at  2,  .
 3
1
Step 5: Since n = m , the line y = = 1 is the horizontal asymptote. Solve to find intersection points:
1
2
x −4
2
=1
x −x−2
x2 − 4 = x2 − x − 2
x=2
G ( x) does not intersect y = 1 because G ( x) is not defined at x = 2 . Plot the line y = 1 using dashes.

Step 6:

Steps 7 & 8: Graphing:

42. The x-intercepts of the graph of f are -3 and 2. b. The graph of f is above the x-axis (so f is
a. f ( x) = 0 for x = −3 and 2 . positive) for −3 < x < 2 or x > 2 .

426
Copyright © 2015 Pearson Education, Inc.
Chapter 3 Review Exercises

Therefore, the solution set is


{ x − 3 < x < 2 or x > 2 } or, using interval 44. x3 + x 2 < 4 x + 4
notation, (−3, 2) ∪ (2, ∞) . x3 + x 2 − 4 x − 4 < 0
c. The graph of f is below the x-axis (so f is x 2 ( x + 1) − 4 ( x + 1) < 0
negative) for x < −3 . Since the inequality is
not strict, we include -3 and 2 in the solution (x 2
)
− 4 ( x + 1) < 0
set. Therefore, the solution set is
( x − 2 )( x + 2 )( x + 1) < 0
{ x x ≤ −3 or x = 2 } or, using interval
f ( x) = ( x − 2 )( x + 2 )( x + 1)
notation (−∞, − 3] ∪ {2} .
x = − 2, x = −1, and x = 2 are the zeros of f .
d. The graph crosses the x-axis at x = −3 and Interval ( −∞, −2) ( −2, −1) ( −1, 2) (2, ∞ )
touches at x = 2 . Thus, -3 has odd Number
multiplicity while 2 has an even −3 −3 / 2 0 3
Chosen
multiplicity. Using one for the odd
multiplicity and two for the even Value of f −10 0.875 −4 20
multiplicity, a possible function is Conclusion Negative Positive Negative Positive
f ( x) = a ( x − 2) 2 ( x + 3) . Since the y-
The solution set is { x | x < −2 or − 1 < x < 2} , or,
intercept is 12, we know f (0) = 12 . Thus,
using interval notation, ( −∞, −2 ) ∪ ( −1, 2 ) .
a=1. Using a = 1 , the function is
f ( x) = ( x − 2) 2 ( x + 3) .
−4 −2 0 2 4
43. The x-intercepts of the graph of f are −3 and −1 .
6
a. The horizontal asymptote is y = 0.25 . 45. ≥1
x+3
b. The vertical asymptotes are x = − 2 and x = 2 6 6 − 1( x + 3) −x + 3
−1 ≥ 0  ≥0 ≥0
c. The graph of f is below the x-axis (so f is x+3 x+3 x+3
negative) for −3 < x < −2 or −1 < x < 2 . − ( x − 3)
f ( x) =
Therefore, the solution set is x+3
{ x − 3 < x < −2 or − 1 < x < 2 } or, using The zeros and values where the expression is
interval notation, (−3, −2) ∪ (−1, 2) . undefined are x = 3 and x = −3 .

d. The graph of f is above the x-axis (so f is


Interval ( −∞, − 3) ( −3, 3) ( 3, ∞ )
positive) for Number
−4 0 4
−∞ < x < −3 or − 2 < x < −1 or x > 2 . Since Chosen
the inequality is not strict, we include -3 and -1 1
Value of f −7 1 −
in the solution set. Therefore, the solution set 7
is { x − ∞ < x < −3 or − 2 < x < −1 or x > 2 } Conclusion Negative Positive Negative
or, using interval notation The solution set is {x − 3 < x ≤ 3 } , or, using
(−∞, −3] ∪ (−2, −1] ∪ ( 2, ∞) .
interval notation, ( −3,3] .
( x + 3)( x + 1)
e. One possibility: R( x) = a
( x + 2)( x − 2) −4 −2 0 2 4
 3
(Using the point  0, −  leads to
 16 
a = 1 / 4 .) Thus,
( x + 3)( x + 1) x 2 + 4 x + 3
R( x) = = .
4( x + 2)( x − 2) 4 x 2 − 16

427
Copyright © 2015 Pearson Education, Inc.
Chapter 3 Polynomial and Rational Functions

2x − 6 x 2 − 8 x + 12
46. <2 48. >0
1− x x 2 − 16
2x − 6
−2< 0 x 2 − 8 x + 12
1− x f ( x) =
x 2 − 16
2 x − 6 − 2(1 − x)
<0 ( x − 2)( x − 6)
1− x >0
( x + 4)( x − 4)
4x − 8
<0 The zeros and values where the expression is
1− x
undefined are x = − 4, x = 2, x = 4, and x = 6 .
4 ( x − 2)
f ( x) = Number
1− x Interval Value of f Conclusion
Chosen
The zeros and values where the expression is
undefined are x = 1, and x = 2 . 77
( −∞, − 4 ) −5 Positive
9
Interval ( −∞, 1) (1, 2 ) ( 2, ∞ )
3
Number ( −4, 2 ) 0 − Negative
0 1.5 3 4
Chosen 3
Value of f −8 4 −2 ( 2, 4 ) 3 Positive
7
Conclusion Negative Positive Negative 1
( 4, 6 ) 5 − Negative
The solution set is {x x < 1 or x > 2 } , or, 3
5
using interval notation, ( −∞,1) ∪ ( 2, ∞ ) . ( 6, ∞ ) 7 Positive
33
The solution set is
−4 −2 0 2 4
{ x x < − 4 or 2 < x < 4 or x > 6 } , or, using
( x − 2)( x − 1) interval notation, ( −∞, −4 ) ∪ ( 2, 4 ) ∪ ( 6, ∞ ) .
47. ≥0
x−3
( x − 2)( x − 1) −8 −4 0 4 8
f ( x) =
x−3
The zeros and values where the expression is 250
49. a. 250 = πr 2 h  h = ;
undefined are x = 1, x = 2, and x = 3 . πr 2
( −∞, 1) (1, 2 ) ( 2, 3 )  250 
Interval (3, ∞ ) A(r ) = 2π r 2 + 2πrh = 2πr 2 + 2πr  2 
Number  πr 
0 1.5 2.5 4 500
Chosen = 2πr 2 +
r
2 1 3
Value of f − − 6
3 6 2 500
Conclusion Negative Positive Negative Positive
b. A(3) = 2π ⋅ 32 +
3
The solution set is { x 1≤ x ≤ 2 or x > 3 } , or, = 18π +
500
≈ 223.22 square cm
3
using interval notation, [1, 2] ∪ ( 3,∞ ) .
500
c. A(5) = 2π ⋅ 52 +
5
−2 0 2 4 6
= 50π + 100 ≈ 257.08 square cm

428
Copyright © 2015 Pearson Education, Inc.
Chapter 3 Test

d. Use MINIMUM on the graph of then shift down 2 units.


500
y1 = 2πx 2 +
x

The area is smallest when the radius is


approximately 3.41 cm.
2. a. The maximum number of real zeros is the
degree, n = 3 .

b. g ( x ) = 2 x3 + 5 x 2 − 28 x − 15
We list all integers p that are factors of
50. a. c. a0 = −15 and all the integers q that are
factors of a3 = 2 .
b. P (t ) = 0.0095t 3 − 0.2195t 2 + 1.185t + 10.008
p : ± 1, ± 3, ± 5, ± 15
P (16) = 0.0095(16)3 − 0.2195(16) 2
q : ± 1, ±2
+ 1.185(16) + 10.008 = 11.7% p
Now we form all possible ratios q :
51 - 22. Answers will vary.
p 1 3 5 15
: ± , ±1, ± , ± , ±3, ±5, ± , ±15
53. a. Since the graph points upward for large q 2 2 2 2
values of x , the degree is even. If g has a rational zero, it must be one of the
b. Likewise, the leading coefficient is positive. 16 possibilities listed.
c. The graph is symmetric about the y-axis, so c. We can find the rational zeros by using the
the function is even. fact that if ris a zero of g, then g ( r ) = 0 .
d. The graph touches the x-axis at x = 0, which That is, we evaluate the function for
means 0 is a zero of the function with even different values from our list of rational
multiplicity. Thus, x 2 must be a factor. zeros. If we get g ( r ) = 0 , we have a zero.
e. The graph has 7 turning points, so the Then we use long division to reduce the
degree of the polynomial must be at least 8. polynomial and start again on the reduced
f. Answers will vary. polynomial.
We will start with the positive integers:
g (1) = 2 (1) + 5 (1) − 28 (1) − 15 ,
3 2

Chapter 3 Test
= 2 + 5 − 28 − 15 = −36
g (3) = 2 (3) + 5 (3) − 28 (3) − 15
4 3 2
1. f ( x) = ( x − 3) − 2
Using the graph of y = x 4 , shift right 3 units, = 54 + 45 − 84 − 15 = 0
So, we know that 3 is a zero. This means
that ( x − 3) must be a factor of g. Using
long division we get

429
Copyright © 2015 Pearson Education, Inc.
Chapter 3 Polynomial and Rational Functions

2 x 2 + 11x + 5 f. The power function that the graph of g


x − 3 2 x3 + 5 x 2 − 28 x − 15 resembles for large values of x is given by
the term with the highest power of x. In this
(
− 2 x3 − 6 x 2 ) case, the power function is y = 2 x3 . So, the
graph of g will resemble the graph of
11x 2 − 28 x
y = 2 x3 for large values of x .
(
− 11x 2 − 33 x )
g. We could first evaluate the function at
5 x − 15 several values for x to help determine the
− ( 5 x − 15 ) scale.
Putting all this information together, we
0
obtain the following graph:
Thus, we can now write
(
g ( x ) = ( x − 3) 2 x 2 + 11x + 5 )
The quadratic factor can be factored so we
get:
g ( x ) = ( x − 3)( 2 x + 1)( x + 5 )
To find the remaining zeros of g, we set the
last two factors equal to 0 and solve.
2x +1 = 0 x+5 = 0
2 x = −1 x = −5
1
x=− 3. x 3 − 4 x 2 + 25 x − 100 = 0
2
1 x 2 ( x − 4 ) + 25 ( x − 4 ) = 0
Therefore, the zeros are −5 , − , and 3.
2 ( x − 4 ) ( x 2 + 25) = 0
Notice how these rational zeros were all in
the list of potential rational zeros. x − 4 = 0 or x 2 + 25 = 0
d. The x-intercepts of a graph are the same as x=4 x 2 = −25
the zeros of the function. In the previous x = ± −25
1
part, we found the zeros to be −5 , − , and x = ±5i
2
The solution set is {4, −5i, 5i} .
1
3. Therefore, the x-intercepts are −5 , − ,
2 4. 3 x3 + 2 x − 1 = 8 x 2 − 4
and 3.
3x3 − 8 x 2 + 2 x + 3 = 0
To find the y-intercept, we simply find If we let the left side of the equation be f ( x ) ,
g ( 0) .
then we are simply finding the zeros of f.
3 2
g ( 0 ) = 2 ( 0 ) + 5 ( 0 ) − 28 ( 0 ) − 15 = −15 We list all integers p that are factors of a0 = 3
So, the y-intercept is −15 . and all the integers q that are factors of a3 = 3 .
p : ± 1, ±3 ; q : ±1, ±3
e. Whether the graph crosses or touches at an
x-intercept is determined by the multiplicity. p 1
Now we form all possible q : ± , ±1, ±3
Each factor of the polynomial occurs once, 3
so the multiplicity of each zero is 1. For odd
multiplicity, the graph will cross the x-axis
at the zero. Thus, the graph crosses the x-
axis at each of the three x-intercepts.

It appears that there is a zero near x = 1 .

430
Copyright © 2015 Pearson Education, Inc.
Chapter 3 Test

3 2
f (1) = 3 (1) − 8 (1) + 2 (1) + 3 = 0 ( x + 3) ( x − 1)
r ( x) =
Therefore, x=1 is a zero and ( x − 1) is a factor of x +1
The domain of the function is { x | x ≠ −1} .
f ( x ) . We can reduce the polynomial expression
by using synthetic division. Asymptotes:
Since the function is in lowest terms, the graph
1 3 −8 2 3 has one vertical asymptote, x = −1 .
The degree of the numerator is one more than the
3 −5 −3 degree of the denominator so the graph will have
3 −5 −3 0 an oblique asymptote. To find it, we need to use
long division (note: we could also use synthetic
( )
Thus, f ( x ) = ( x − 1) 3 x 2 − 5 x − 3 . We can find division in this case because the dividend is
the remaining zeros by using the quadratic linear).
formula. x +1
3x 2 − 5 x − 3 = 0 x + 1 x2 + 2 x − 3
a = 3, b = −5, c = −3
2
(
− x2 + x )
− ( −5 ) ± ( −5 ) − 4 ( 3)( −3)
x= x−3
2 ( 3)
− ( x + 1)
5 ± 25 + 36 5 ± 61
= = −4
6 6 The oblique asymptote is y = x + 1 .
 5 − 61 5 + 61 
Thus, the solution set is 1, , . 7. From problem 6 we know that the domain is
 6 6 
{ x | x ≠ −1} and that the graph has one vertical
5. We start by factoring the numerator and asymptote, x = −1 , and one oblique asymptote,
denominator. y = x +1.
2 x 2 − 14 x + 24 2 ( x − 3) ( x − 4 ) x-intercepts:
g ( x) = 2 =
x + 6 x − 40 ( x + 10 ) ( x − 4 ) To find the x-intercepts, we need to set the
numerator equal to 0 and solve the resulting
The domain of f is { x | x ≠ −10, x ≠ 4} .
equation.
2 ( x − 3) ( x + 3)( x − 1) = 0
In lowest terms, g ( x ) = with x ≠ 4 . x + 3 = 0 or x − 1 = 0
x + 10
The graph has one vertical asymptote, x = −10 , x = −3 x =1
since x + 10 is the only factor of the The x-intercepts are −3 and 1.
denominator of g in lowest terms. The graph is The points ( −3, 0 ) and (1, 0 ) are on the graph.
still undefined at x = 4 , but there is a hole in the y-intercept:
graph there instead of an asymptote.
02 + 2 ( 0 ) − 3
Since the degree of the numerator is the same as r ( 0) = = −3
0 +1
the degree of the denominator, the graph has a
horizontal asymptote equal to the quotient of the The y-intercept is −3 . The point ( 0, −3) is on
leading coefficients. The leading coefficient in the graph.
the numerator is 2 and the leading coefficient in Test for symmetry:
the denominator is 1. Therefore, the graph has
( − x )2 + 2 ( − x ) − 3 x 2 − 2 x − 3
the horizontal asymptote y = 12 = 2 . r (−x) = =
(−x) +1 −x +1
Since r ( − x ) ≠ r ( x ) , the graph is not symmetric
x2 + 2 x − 3
6. r ( x ) = with respect to the y-axis.
x +1
Start by factoring the numerator.

431
Copyright © 2015 Pearson Education, Inc.
Chapter 3 Polynomial and Rational Functions

Since r ( − x ) ≠ −r ( x ) , the graph is not the following function:


symmetric with respect to the origin. f ( x ) = a ( x − ( −2 ) ) ( x − 0 ) ( x − ( 3 − i ) ) ( x − ( 3 + i ) )
Behavior near the asymptotes: where a is any real number. If we let a = 1 , we
To determine if the graph crosses the oblique get
asymptote, we solve the equation f ( x) = ( x + 2) ( x ) ( x − 3 + i ) ( x − 3 − i )
r ( x) = x +1
2
( )
= x2 + 2 x ( x − 3 + i ) ( x − 3 − i )
x + 2x − 3
= x + 1, x ≠ −1 = (x + 2 x )( x )
2 2
x +1 − 6 x + 10

x2 + 2 x − 3 = x2 + 2 x + 1 4 3 22
= x − 6 x + 10 x + 2 x − 12 x + 20 x
3

−3 = 1 false = x 4 − 4 x3 − 2 x 2 + 20 x
The result is a contradiction so the graph does
not cross the oblique asymptote. 9. Since the domain excludes 4 and 9, the
denominator must contain the factors ( x − 4 )
The zeros of the numerator and denominator,
−3 , −1 , and 1, divide the x-axis into four and ( x − 9 ) . However, because there is only one
subintervals. vertical asymptote, x = 4 , the numerator must
( −∞, −3) , ( −3, −1) , ( −1,1) , (1, ∞ ) also contain the factor ( x − 9 ) .
We can check a point in each subinterval to
determine if the graph is above or below the x- The horizontal asymptote, y = 2 , indicates that
axis. the degree of the numerator must be the same as
Interval ( −∞, −3) ( −3, −1) ( −1,1) (1, ∞ ) the degree of the denominator and that the ratio
Number −5 −2 0 3 of the leading coefficients needs to be 2. We can
accomplish this by including another factor in
Value of r −3 3 −3 3
the numerator, ( x − a ) , where a ≠ 4 , along with
Location below above below above
a factor of 2.
Point ( −5, −3) ( −2,3) ( 0, −3) ( 3,3)
2 ( x − 9) ( x − a )
Therefore, we have r ( x ) = .
y = x +1
( x − 4 )( x − 9 )
y
If we let a = 1 , we get
5 2 ( x − 9 ) ( x − 1) 2 x 2 − 20 x + 18
r ( x) = = .
( x − 4 ) ( x − 9 ) x 2 − 13x + 36
10. Since we have a polynomial function and
(−3, 0)
polynomials are continuous, we simply need to
−5 (1, 0) 5 x show that f ( a ) and f ( b ) have opposite signs
(where a and b are the endpoints of the interval).
(0, −3) 2
f ( 0 ) = −2 ( 0 ) − 3 ( 0 ) + 8 = 8
−5 2
f ( 4 ) = −2 ( 4 ) − 3 ( 4 ) + 8 = −36
x = −1 Since f ( 0 ) = 8 > 0 and f ( 4 ) = −36 < 0 , the

8. Since the polynomial has real coefficients, we Intermediate Value Theorem guarantees that
can apply the Conjugate Pairs Theorem to find there is at least one real zero between 0 and 4.
the remaining zero. If 3 + i is a zero, then its
conjugate, 3 − i , must also be a zero. Thus, the x+2
11. <2
four zeros are −2 , 0, 3 − i , and 3 + i . The Factor x −3
Theorem says that if f ( c ) = 0 , then ( x − c ) is a We note that the domain of the variable consists
of all real numbers except 3.
factor of the polynomial. This allows us to write

432
Copyright © 2015 Pearson Education, Inc.
Chapter 3 Cumulative Review

x = 0, x = 1 are the zeros of f .


Rearrange the terms so that the right side is 0.
Interval (−∞, 0) (0, 1) (1, ∞)
x+2
−2< 0 Number
x −3 −1 0.5 2
x+2 Chosen
For f ( x ) = − 2 , we find the zeros of f and Value of f 2 −0.25 2
x−3
the values of x at which f is undefined. To do Conclusion Positive Negative Positive
this, we need to write f as a single rational
expression. The solution set is {x x ≤ 0 or x ≥ 1 } or
x+2
f ( x) = −2 ( −∞, 0 or 1, ∞ ) in interval notation.
x −3
x+2 x −3
= − 2⋅
x −3 x −3
x + 2 − 2x + 6
=
x−3 3. x 2 − 3x < 4
−x + 8
= x 2 − 3x − 4 < 0
x−3
The zero of f is x = 8 and f is undefined at x = 3 . ( x − 4 ) ( x + 1) < 0
We use these two values to divide the real f (x ) = x 2 − 3x − 4
number line into three subintervals.
3 8 x = −1, x = 4 are the zeros of f .

Interval (−∞,3 ) (3,8 ) (8, ∞ ) Interval (−∞, − 1) (−1, 4) (4, ∞)


Num. ch osen 0 4 9 Number
−2 0 5
Value of f − 8 4 −1 Chosen
3 6 Value of f 6 −4 6
C onclus ion negative pos itive negative
Conclusion Positive Negative Positive
Since we want to know where f ( x ) is negative,
we conclude that values of x for which x < 3 or The solution set is {x −1 < x < 4 } or ( −1, 4 )
x > 8 are solutions. The inequality is strict so in interval notation.
the solution set is { x | x < 3 or x > 8} . In
interval notation we write ( −∞,3) or ( 8, ∞ ) .

4. Slope –3, Containing the point (–1, 4)


Chapter 3 Cumulative Review Using the point-slope formula yields:
y − y1 = m ( x − x1 )
1. P = (1, 3) , Q = ( −4, 2 )
y − 4 = −3 ( x − ( −1) )
d P ,Q = ( −4 − 1)2 + ( 2 − 3)2
y − 4 = −3 x − 3
2 2
= ( −5 ) + ( −1) = 25 + 1 y = −3 x + 1
= 26 Thus, f ( x ) = −3 x + 1 .

2. x2 ≥ x
x2 − x ≥ 0
x( x − 1) ≥ 0
f ( x) = x 2 − x

433
Copyright © 2015 Pearson Education, Inc.
Chapter 3 Polynomial and Rational Functions

The solution set is {0, 2, 4} .

9. 3x + 2 ≤ 5 x − 1
3 ≤ 2x
3
≤x
2
3
x≥
2
 3 3 
The solution set is  x x ≥  or  , ∞  in
 2 2 
5. Parallel to y = 2 x + 1 ; Slope 2, Containing the interval notation.
point (3, 5)
Using the point-slope formula yields:

y − y1 = m ( x − x1 )
10. x2 + 4x + y2 − 2 y − 4 = 0
y − 5 = 2 ( x − 3)
( x 2 + 4 x + 4) + ( y 2 − 2 y + 1) = 4 + 4 + 1
y − 5 = 2x − 6
y = 2x −1 ( x + 2) 2 + ( y − 1) 2 = 9
( x + 2) 2 + ( y − 1) 2 = 32
Center: (–2, 1)
Radius 3

6. y = x3

11. y = x 3 − 9 x

x-intercepts: 0 = x3 − 9 x
(
0 = x x2 − 9 )
0 = x ( x + 3)( x − 3)
7. This relation is not a function because the
ordered pairs (3, 6) and (3, 8) have the same first x = 0, −3, and 3
element, but different second elements. ( 0, 0 ) , ( −3, 0 ) , ( 3, 0 )
8. x3 − 6 x 2 + 8 x = 0 y-intercepts: y = 03 − 9 ( 0 ) = 0  ( 0, 0 )
( 2
x x − 6x + 8 = 0 ) Test for symmetry:
x ( x − 4) ( x − 2) = 0
x = 0 or x = 4 or x = 2

434
Copyright © 2015 Pearson Education, Inc.
Chapter 3 Cumulative Review

x-axis: Replace y by − y : − y = x3 − 9 x , which f ( x + h) − f ( x)


e.
3
is not equivalent to y = x − 9 x . h

3
( x + h )2 + 5 ( x + h ) − 2 − ( x 2 + 5 x − 2 )
y-axis: Replace x by − x : y = ( − x ) − 9 ( − x ) =
h
= − x3 + 9 x x + 2 xh + h + 5 x + 5h − 2 − x 2 − 5 x + 2
2 2
=
h
which is not equivalent to y = x3 − 9 x .
2 xh + h 2 + 5h
=
Origin: Replace x by − x and y by − y : h
3
− y = (−x) − 9 (−x) = 2x + h + 5

y = − x3 + 9 x x+5
15. f ( x) =
which is equivalent to y = x3 − 9 x . Therefore, x −1
the graph is symmetric with respect to origin.
a. Domain { x x ≠ 1} .
12. 3x − 2 y = 7
−2 y = −3 x + 7 2+5 7
b. f (2) = = =7 ≠ 6;
3 7 2 −1 1
y = x− ( 2,6 ) is not on the graph of f .
2 2
3 The point ( 2, 7 ) is on the graph.
The given line has slope . Every line that is
2
perpendicular to the given line will have slope 3+5 8
c. f (3) = = =4;
2 3 −1 2
− . Using the point (1, 5 ) and the point-slope
3 ( 3,4 ) is on the graph of f .
formula yields:
y − y1 = m ( x − x1 ) d. Solve for x
x+5
2 =9
y −5 = − ( x − 1) x −1
3
2 2 x + 5 = 9 ( x − 1)
y −5 = − x +
3 3 x + 5 = 9x − 9
2 17 14 = 8 x
y = − x+
3 3 14 7
x= =
13. Not a function, since the graph fails the Vertical 8 4
Line Test, for example, when x = 0 . 7 
Therefore,  ,9  is on the graph of f .
4 
14. f ( x) = x 2 + 5 x − 2
e. f ( x ) is a rational function since it is in the
2
a. f (3) = 3 + 5(3) − 2 = 9 + 15 − 2 = 22 p( x)
form .
q( x)
2
b. f (− x) = ( − x ) + 5 ( − x ) − 2 = x 2 − 5 x − 2
16. f ( x ) = −3x + 7
c. ( )
− f ( x) = − x 2 + 5 x − 2 = − x 2 − 5 x + 2 The graph is a line with slope –3 and y-intercept
(0, 7).
2
d. f (3x) = ( 3x ) + 5 ( 3 x ) − 2 = 9 x 2 + 15 x − 2

435
Copyright © 2015 Pearson Education, Inc.
Chapter 3 Polynomial and Rational Functions

17. f ( x) = 2 x 2 − 4 x + 1

a = 2, b = −4, c = 1. Since a = 2 > 0, the 18. f ( x ) = x2 + 3x + 1


graph opens up.
average rate of change of f from 1 to 2 :
The x-coordinate of the vertex is f ( 2 ) − f (1) 11 − 5
b −4 = = 6 = msec
x=− =− =1. 2 −1 1
2a 2 ( 2)
f ( 2 ) = 11 so the point ( 2,11) is on the graph.
The y-coordinate of the vertex is Using this point and the slope m = 6 , we can
 b  2 obtain the equation of the secant line:
f  −  = f (1) = 2 (1) − 4 (1) + 1 = −1 .
 2a  y − y1 = m ( x − x1 )
Thus, the vertex is (1, –1). y − 11 = 6 ( x − 2 )
The axis of symmetry is the line x = 1 . y − 11 = 6 x − 12
The discriminant is: y = 6x −1
2
b 2 − 4ac = ( −4 ) − 4 ( 2 ) (1) = 8 > 0 , so the graph 19. a. x-intercepts: ( −5, 0 ) ; ( −1, 0 ) ; ( 5, 0 ) ;
has two x-intercepts. y-intercept: (0,–3)
The x-intercepts are found by solving: b. The graph is not symmetric with respect to
2 x2 − 4 x + 1 = 0 the origin, x-axis or y-axis.
− ( −4 ) ± 8 c. The function is neither even nor odd.
x=
2 ( 2)
d. f is increasing on (−∞, − 3) and (2, ∞) ; f is
4±2 2 2± 2
= = decreasing on ( −3, 2 ) ;
4 2
e. f has a local maximum at x = −3, and the
2− 2 2+ 2
The x-intercepts are and . local maximum is f ( −3) = 5 .
2 2

The y-intercept is f (0) = 1 . f. f has a local minimum at x = 2, and the


local minimum is f ( 2 ) = −6 .

5x
20. f ( x) = 2
x −9
5 (−x) −5 x
f (− x) = 2
= 2 = − f ( x ) , therefore
(−x) − 9 x − 9
f is an odd function.

436
Copyright © 2015 Pearson Education, Inc.
Chapter 3 Cumulative Review

2 x + 1 if − 3 < x < 2 a. ( f + g )( x) = x 2 − 5 x + 1 + ( −4 x − 7 )
21. f ( x) = 
−3 x + 4 if x ≥ 2 = x2 − 9 x − 6
The domain is: { x x is a real number} .
a. Domain: { x x > −3} or ( −3, ∞ )

 f  f ( x ) x2 − 5x + 1
 1  b.   ( x) = =
b. x-intercept:  − , 0  g ( x) −4 x − 7
 2  g
y-intercept: (0,1)  7
The domain is:  x x ≠ −  .
c.  4

24. a. R ( x) = x ⋅ p
 1 
= x  − x + 150 
 10 
1
= − x 2 + 150 x
10

1
b. R (100) = − (100) 2 + 150(100)
10
= −1000 + 15, 000
= $14, 000
d. Range: {y y < 5 } or ( −∞,5 )
1 2
c. Since R( x) = − x + 150 x is a quadratic
22.
2
f ( x) = −3 ( x + 1) + 5 10
1
function with a = − < 0 , the vertex will
Using the graph of y = x 2 , shift left 1 unit, 10
vertically stretch by a factor of 3, reflect about be a maximum point. The vertex occurs
the x-axis, then shift up 5 units. b 150
when x = − =− = 750 .
2a 2 ( −1/10 )
Thus, the revenue is maximized when
x = 750 units sold.
The maximum revenue is given by

1
R (750) = − (750) 2 + 150(750) .
10
= −56, 250 + 112, 500
= $56, 250

1
d. p=− (750) + 150 = −75 + 150 = $75 is
10
the selling price that maximizes the revenue.
23. f ( x) = x 2 − 5 x + 1 g ( x ) = −4 x − 7

437
Copyright © 2015 Pearson Education, Inc.
Solutions Manual for College Algebra: Concepts Through Functions 3rd Sullivan III 0321925726

Chapter 3 Polynomial and Rational Functions

Chapter 3 Projects e. f ( x) = x 4 + bx3 + cx 2 + dx + e


Project I – Internet-based Project f ( x) = ( x − r1 )( x − r2 )( x − r3 )( x − r4 )
f ( x) = ( x3 − (r1 + r2 + r3 ) x 2
Answers will vary
+ (r1r2 + r1r3 + r2 r3 ) x − r1r2 r3 )( x − r3 )
Project II f ( x) = x 4 − (r1 + r2 + r3 + r4 ) x3
a. x 2 + 8 x − 9 = 0 + ( r1r2 + r1r3 + r2 r3 + r1r4 + r2 r4 + r3 r4 ) x 2
( x + 9)( x − 1) = 0 −(r1r2 r4 + r1r3 r4 + r2 r3 r4 + r1r2 r3 ) x + r1r2 r3 r4
sum = −9 + 1 = −8 , product = ( −9 )(1) = −9
b = −(r1 + r2 + r3 + r4 )
x = −9 or x = 1
c = r1r2 + r1r3 + r2 r3 + r1r4 + r2 r4 + r3 r4
2
b. x + bx + c = 0 d = −(r1r2 r4 + r1r3 r4 + r2 r3 r4 + r1r2 r3 )
( x − r1 )( x − r2 ) = 0 e = r1r2 r3 r4
2
x − r1 x − r2 x + r1r2 = 0
f. The coefficients are sums, products, or sums of
x 2 − (r1 + r2 ) x + r1r2 = 0 products of the zeros.
b = −(r1 + r2 ) If f ( x) = x n + an −1 x n −1 + an − 2 x n − 2 + ... + a1 x + a0 ,
c = r1r2 then:
an −1 will be the negative of the sum of the zeros.
c. f ( x) = x3 − x 2 − 10 x − 8
an − 2 will be the sum of the double products.
f ( x) = ( x + 2)( x 2 − 3 x − 4)
a1 will be the negative (if n is even) or positive
f ( x) = ( x + 2)( x − 4)( x + 1)
(if n is odd) of the sum of (n-1) products.
zeros: −2 . 4. −1
a0 will be the negative (if n is odd) or positive
sum = −2 + 4 − 1 = 1 , product = (−2)(4)(−1) = 8
(if n is even) product of the zeros.
sum of double products
These will always hold. These would be useful if
= −2(4) + (−2)(−1) + 4(−1) = −8 + 2 − 4 = −10 you needed to multiply a number of binomials in
The coefficient of x 2 is the negative sum. The x − c form together and you did not want to have
coefficient of x is the sum of the double to do the multiplication out. These formulas
products. The constant term is the negative would help same time.
product.

d. f ( x) = x3 + bx 2 + cx + d
f ( x) = ( x − r1 )( x − r2 )( x − r3 )
f ( x) = ( x 2 − (r1 + r2 ) x + r1r2 )( x − r3 )
f ( x) = x3 − (r1 + r2 + r3 ) x 2
+(r1r2 + r1r3 + r2 r3 ) x − r1r2 r3
b = −(r1 + r2 + r3 )
c = r1r2 + r1r3 + r2 r3
d = −r1r2 r3

438
Copyright © 2015 Pearson Education, Inc.

Visit TestBankBell.com to get complete for all chapters


Another random document with
no related content on Scribd:
[205]

[Contents]

NOTES AND CORRESPONDENCE.

I.—I publish the following legal documents—the first articles for which I have to
expend any of St. George’s money,—intact: venturing not so much as the
profanity of punctuation. The Memorandum is drawn up by one of our leading
counsel, from my sketch of what I wanted. The points on which it may need
some modification are referred to in my added notes; and I now invite farther
criticism or suggestion from the subscribers to the Fund.

“2, Bond Court, Walbrook, London, E.C.,


“June 15th, 1875.

“St. George’s Company.

“Dear Sir,—According to the promise in our Mr. Tarrant’s letter of the 11th, we
now beg to send you what Mr. Wm. Barber, after reading your sketch, has
approved of as the written fundamental laws of the Company,—though we shall
be quite prepared to find that some alterations in it are still necessary to
express your views correctly.
“We are,
“Dear Sir,
“Yours faithfully,
“Tarrant & Mackrell.

“Professor Ruskin, Corpus Ch. Coll., Oxford.”

[206]

[Contents]

MEMORANDUM AND STATUTES OF THE COMPANY OF


ST. GEORGE.

The Company is constituted with the object of determining and instituting in


practice the wholesome laws of agricultural life and economy and of instructing
the agricultural labourer in the science art and literature of good husbandry. (a)
With this object it is proposed to acquire by gift purchase or otherwise plots or
tracts of land in different parts of the country which will be brought into such
state of cultivation or left uncultivated or turned into waste or common land and
applied to such purposes as having regard to the nature of the soil and other
surrounding circumstances may in each case be thought to be most generally
useful.

The members of the Company shall be styled Companions of the Company of


St. George (b) Any person may become a Companion by subscribing not less
than £ in money to the funds of the Company or by making a gift to the
Company of land not less than £ in value (c) and by having his name entered
on the Roll of Companions with due solemnity.

The name of every Companion shall be entered on the Roll of Companions


either by himself in the presence of two witnesses of full age who shall attest
such entry or if the Companion shall so desire by the Master of the Company
with the same formalities. The Roll of Companions shall be kept in safe custody
within the walls of the College of Corpus Christi in Oxford or at such other safe
and commodious place as the Companions shall from time to time direct.

Each Companion shall by virtue of the entry of his name on the Roll be deemed
to have bound himself by a solemn vow and promise as strict as if the same
had been ratified by oath to be true and loyal to the Company and to the best of
his power and might so far as in him lies to forward and advance the
[207]objects and interests thereof and faithfully to keep and obey the statutes
and rules thereof yet so nevertheless that he shall not be bound in any way to
harass annoy injure or inconvenience his neighbour.

Chief among the Companions of the Company shall be the Master thereof who
so long as he shall hold office shall have full and absolute power at his will and
pleasure to make and repeal laws and byelaws (d) and in all respects to rule
regulate manage and direct the affairs of the Company and receive apply and
administer funds and subscriptions in aid of its objects and to purchase acquire
cultivate manage lease sell or otherwise dispose of the estates and properties
of the Company and generally direct and control the operations thereof.

The Master shall be elected and may from time to time and at any time be
deposed by the votes of a majority in number of the Companions in General
Meeting assembled but except in the event of his resignation or deposition
shall hold office for life. The first Master of the Company shall be John Ruskin
who shall however (subject to re-election) only hold office until the first General
Meeting of the Companions.

The Master shall render to each Companion and shall be at liberty if he shall so
think fit to print for public circulation a monthly report and account of the
operations and financial position of the Company.

No Master or other Companion of the Company shall either directly or indirectly


receive any pay profit emolument or advantage whatsoever from out of by or by
means of his office or position as a member of the Company.

The practical supervision and management of the estates and properties of the
Company shall subject to the direction and control of the Master be entrusted
to and carried out by land agents tenants and labourers who shall be styled
Retainers of the Company. [208]
The name of each Retainer in the permanent employ of the Company shall be
entered in a Register to be called the Roll of Retainers and to be kept at the
same place as the Roll of Companions. Such entry shall be made either by the
Retainer himself in the presence of one witness of full age who shall attest the
entry or if the Retainer shall so desire by the Master with the same formalities.

No pecuniary liability shall attach to any Retainer of the Company by virtue of


his position as such but each Retainer shall by virtue of the entry of his name
on the Roll be deemed to have bound himself by a solemn vow and promise as
strict as if the same had been ratified by oath to be true and loyal to the
Company and faithfully to keep and obey the statutes and rules thereof and the
orders and commands of the officers of the Company who from time to time
may be set over him.

Each land agent and labourer being a Retainer of the Company shall receive
and be paid a fixed salary in return for his services and shall not by perquisites
commissions or any other means whatever either directly or indirectly receive
or acquire any pay profit emolument or advantages whatever other than such
fixed salary from out of or by means of his office or position as a Retainer of the
Company.

The rents and profits to be derived from the estates and properties of the
Company shall be applied in the first instance in the development of the land
(e) and the physical intellectual moral social and religious improvement of the
residents thereon in such manner as the Master shall from time to time direct or
approve and the surplus rents and profits if any shall be applied in reduction of
the amount paid by the tenants in proportion to their respective skill and
industry either by a gradual remission of rent towards the close of the tenancy
or in such other way as may be thought best but in no case shall the
Companions personally derive any rents or profits from the property of the
Company. [209]

All land and hereditaments for the time being belonging to the Company shall
be conveyed to and vested in any two or more of the Companions whom the
Master may from time to time select for the office as Trustees of the Company
and shall be dealt with by them according to the directions of the Master. (f)

The property of the Company shall belong to the Companions in the shares
and proportions in which they shall have respectively contributed or by
succession or accruer become entitled to the same.

Each Companion shall be entitled by writing under his hand during his lifetime
or by will or codicil to appoint one person as his successor in the Company and
such person shall on entry of his name on the Roll of Companions in
compliance with the formalities hereinbefore prescribed become a Companion
of the Company and become entitled to the share of his appointor in the
property of the Company. (g)

Each Companion shall at any time be entitled to resign his position by giving to
the Master a Notice under his hand of his desire and intention so to do.

If any Companion shall resign his position or die without having appointed a
successor or if the person so appointed shall for calendar months after the
date when notice of such resignation shall have been received by the Master or
after the date of such death as the case may be fail to have his name entered
on the Roll of Companions in compliance with the formalities hereinbefore
prescribed his share in the property of the Company shall forthwith become
forfeited and shall accrue to the other Companions in the shares and
proportions in which they shall inter se be for the time being entitled to the
property of the Company. (h)

The Company may at any time be dissolved by the Votes of three-fourths of the
Companions in General Meeting assembled and in the event of the Company
being so dissolved or being [210]dissolved by any other means not hereinbefore
specially provided for the property of the Company shall subject to the debts
liabilities and engagements thereof become divisible among the Companions
for the time being in the shares and proportions in which they shall for the time
being be entitled thereto yet so nevertheless that all leases agreements for
leases and other tenancies for the time being subsisting on the property of the
Company shall bind the persons among whom the property comprised therein
shall so become divisible and shall continue as valid and effectual to all intents
and purposes as if the Company had not been dissolved.

Notes on the above Memorandum.

(a) This sentence must be changed into: “such science art and literature as are
properly connected with husbandry.”
(b) In my sketch, I wrote Companions of St. George. But as the existence of St.
George cannot be legally proved or assumed, the tautologically legal phrase
must be permitted.

(c) This clause cannot stand. The admission into the Company must not be
purchaseable; also many persons capable of giving enthusiastic and wise help
as Companions, may be unable to subscribe money. Nothing can be required
as a condition of entrance, except the consent of the Master, and signature
promising obedience to the laws.

(d) This clause needs much development. For though the Master must be
entirely unrestrained in action within the limits of the Laws of the Company, he
must not change or add to them without some manner of consultation with the
Companions. Even in now founding the Society, I do not venture to write a
constitution for it without inviting the help of its existing members; and when
once its main laws are agreed upon, they must be inabrogable without the
same concurrence of the members which would be necessary to dissolve the
Society altogether. [211]

(e) To the development, and enlargement, of the Society’s operations, also.

(f) I do not think the Master should have the power of choosing the Trustees. I
was obliged to do so, before any Society was in existence; but the Trustees
have to verify the Master’s accounts, and otherwise act as a check upon him.
They must not, therefore, be chosen by him.

(g) A questionable clause, which I have not at present time to discuss.

(h) Partly the corollary of (g). The word ‘forfeited’ is morally, if not legally,
objectionable. No idea of forfeiture ought to attach to the resolved surrender of
transferable claim; or to the accidental inability to discover a fitting successor.

Reserving, therefore, the above clauses for future modification, the rest of the
Memorandum fully expresses what seems to me desirable for the first basis of
our constitution; and I shall be glad to hear whether any of the present
subscribers to St. George’s Fund will join me on these conditions.

II.—I should willingly have printed the letter from which the following extracts
are taken, (with comments,) as a ‘Fors’ by itself; but having other matters
pressing, must content myself to leave it in the smaller print. The more
interesting half of it is still reserved for next month.

“What long years have passed since my eyes first saw the calm sweet scene
beyond Wakefield Bridge! I was but a small creature then, and had never been
far from my mother’s door. It was a memorable day for me when I toddled a full
mile from the shady up-town street where we lived, past strange windows, over
unfamiliar flags, to see the big weir and the chapel on the Bridge. Standing on
tiptoe, I could just see over the parapet and look down-stream.

“That was my first peep into fair, green England, and destined [212]never to be
forgotten. The gray old chapel, the shining water below, the far-winding green
banks spangled with buttercups, the grove-clad hills of Heath and Kirkthorpe,—
all seemed to pass into my heart for ever.

“There was no railway then, only the Doncaster coach careering over the
Bridge with a brave sound of horn; fields and farmsteads stood where the
Kirkgate station is; where the twenty black throats of the foundry belch out
flame and soot, there were only strawberry grounds and blossoming pear-
orchards, among which the throstles and blackbirds were shouting for
gladness.

“The chapel lay neglected in a nest of wild willows, and a peaceful cobbler
dwelt in it. As I looked at it, Duke Richard and King Edward became living
realities to me; the dry bones of Pinnock’s Catechism started suddenly into life.
That was the real old chapel of the fifteenth century. Some years after, they
ousted the cobbler, pulled down the old stones, restored it, and opened it for
ritualistic worship; but the cheap stonework has crumbled away again, and it
now looks as ancient as in days of yore. Only, as I remember it, it had a white
hoariness: the foundry smoke has made it black at the present day.

“Some of my companions had been farther out in the world than myself. They
pointed out the dusky shape of Heath Hall, seen through the thinly-clad elm-
trees, and told me how old Lady ——‘s ghost still walked there on stormy
nights. Beyond was Kirkthorpe, where the forlorn shapes of the exiled Spanish
nuns had been seen flitting about their graves in the churchyard.

“There on the right was the tree-crowned mound of Sandal Castle, which
Cromwell had blown down; the dry ditch was full of primroses, they told me;
those woods bounded Crofton, famous for its cowslip fields; and in Heath wood
you would see the ground white with snowdrops in March.

“I do not think that it is the partiality of a native that makes me think you could
hardly find a fairer inland pastoral scene [213]than the one I beheld from
Wakefield Bridge the first time I stood there. On the chapel side there was the
soft green English landscape, with woods and spires and halls, and the brown
sails of boats silently moving among the flowery banks; on the town side there
were picturesque traffic and life; the thundering weir, the wide still water
beyond, the big dark-red granaries, with balconies and archways to the water,
and the lofty white mills grinding out their cheering music.

“But there were no worse shapes than honest, dusty millers’ men, and browned
boatmen, decent people; no open vileness and foul language were rampant in
our quiet clean town in those days. I can remember how clean the pavement
used to look there, and at Doncaster. Both towns are incredibly dirty now. I
cannot bear to look at the filthy beslavered causeway, in places where I
remember to have never seen anything worse than the big round thunder-
drops I used to watch with gleeful interest.

“In those days we were proud of the cleanness and sweet air and gentility of
Wakefield. Leeds was then considered rather vulgar, as a factory town, and
Bradford was obscure, rough, and wild; but Wakefield prided itself in refined
living on moderate means, and cultured people of small income were fond of
settling there.

“Market day used to be a great event for us all.

“I wish that you could have seen the handsome farmers’ wives ranged round
the church walls, with their baskets of apricots and cream cheese, before
reform came, and they swept away my dear old school-house of the
seventeenth century, to make an ugly barren desert of a market ground. You
might have seen, too, the pretty cottagers’ daughters, with their bunches of
lavender and baskets of fruit, or heaps of cowslips and primroses for the wine
and vinegar Wakefield housewives prided themselves upon. On certain days
they stood to be hired as maid-servants, and were prized in the country round
as neat, clean, modest-spoken girls. [214]
“I do not know where they are gone to now,—I suppose to the factories.
Anyhow, Wakefield ladies cry out that they must get servants from London, and
Stafford, and Wales. So class gets parted from class.

“Things were different then. Well-to-do ladies prided themselves on doing their
marketing in person, and kindly feeling and acquaintanceship sprang up
between town and country folk. My Wakefield friends nowadays laugh at the
idea of going to market. They order everything through the cook, and hardly
know their own tradespeople by sight. We used to get delicious butter at
tenpence a pound, and such curds and cream-cheese as I never taste now.
‘Cook’ brings in indifferent butter mostly, at near two shillings.

“As for the farmers’ wives, they would not like to be seen with a butter-basket.
They mostly send the dairy produce off by rail to people whom they never see,
and thus class is more sundered from class every day, even by the very
facilities that railways afford. I can remember that the townspeople had simple
merry-makings and neighbourly ways that this generation would scorn. Many a
pleasant walk we had to the farms and halls that belted the old town; and
boating parties on the Calder, and tea-drinkings and dances—mostly
extempore,—in the easy fashion of Vicar Primrose’s days.

“But pleasure must be sought farther off now. Our young folks go to London or
Paris for their recreation. People seem to have no leisure for being neighbourly,
or to get settled in their houses. They seem to be all expecting to make a heap
of money, and to be much grander presently, and finally to live in halls and
villas, and look down on their early friends.

“But I am sorry for the young people. They run through everything so soon, and
have nothing left to hope for or dream of in a few years. They are better
dressed than we were, and have more accomplishments; but I cannot help
thinking that we young [215]folks were happier in the old times, though shillings
were not half so plentiful, and we had only two frocks a year.

“Tradespeople were different, too, in old Wakefield.

“They expected to live with us all their lives; they had high notions of honour as
tradesmen, and they and their customers respected each other.
“They prided themselves on the ‘wear’ of their goods. If they had passed upon
the housewives a piece of sized calico or shoddy flannel, they would have
heard of it for years after.

“Now the richer ladies go to Leeds or Manchester to make purchases; the town
tradesmen are soured and jealous. They put up big plate-glass fronts, and
send out flaming bills; but one does not know where to get a piece of sound
calico or stout linen, well spun and well woven.

“Give me back our dingy old shops where everything was genuine, instead of
these glass palaces where we often get pins without points, needles without
eyes, and sewing thread sixty yards to the hundred—which I actually heard a
young Quaker defend the other day as an allowable trade practice.”

III.—I venture to print the following sentences from “a poor mother’s” letter, that
my reply may be more generally intelligible. I wish I could say, useful; but the
want of an art-grammar is every day becoming more felt:—

“I am rather ashamed to tell you how young he is (not quite eleven), fearing you
will say I have troubled you idly; but I was sincerely anxious to know your views
on the training of a boy for some definite sort of art-work, and I have always
fancied such training ought to begin very early,—[yes, assuredly,]—also, there
are reasons why we must decide early in what direction we shall look out for
employment for him.”

(I never would advise any parents to look for employment in art as a means of
their children’s support. It is only when [216]the natural bias is quite
uncontrollable, that future eminence, and comfort of material circumstances,
can be looked for. And when it is uncontrollable, it ceases to be a question
whether we should control it. We have only to guide it.)

“But I seem to dread the results of letting him run idle until he is fourteen or
fifteen years old—[most wisely]—and a poor and busy mother like me has not
time to superintend the employment of a boy as a richer one might. This makes
me long to put him to work under a master early. As he does so little at book-
learning, would the practical learning of stone-cutting under the village
stonemason (a good man) be likely to lead to anything further?”
I do not know, but it would be of the greatest service to the boy meanwhile. Let
him learn good joiners’ work also, and to plough, with time allowed him for
drawing. I feel more and more the need of a useful grammar of art for young
people, and simple elementary teaching in public schools. I have always hoped
to remedy this want, but have been hindered hitherto. [217]

Moutard—not -arde; but I can’t give better than this English for it. ↑
1
Fate, and the good novelist, thus dismiss poor grandmamma in a passing sentence,—
2
just when we wanted her so much to live a little longer, too! But that is Fors’s way, and
Gotthelf knows it. A bad novelist would have made her live to exactly the proper moment, and
then die in a most instructive manner, and with pathetic incidents and speeches which would
have filled a chapter. ↑
This paragraph implies, of course, the existence of all modern abuses,—the story dealing
3 only with the world as it is. ↑
A minute Evangelical fragment—dubitable enough. ↑
4
Primarily, because it is untrue. The respect of a child for its parent depends on the
5
parent’s own personal character; and not at all, irrespective of that, on his religious
behaviour. Which the practical good sense of the reverend novelist presently admits. ↑
We keep the metaphor in the phrase, to ‘give a dressing,’ but the short verb is better. ↑
6
Untranslateable. ↑
7
It was unworthy of Gotthelf to spoil his story by this vulgar theatrical catastrophe; and
8his object (namely, to exhibit the character of Hansli in riches as well as poverty,) does
not justify him; for, to be an example to those in his own position, Hansli should have remained
in it. We will, however, take what good we can get: several of the points for the sake of which I
have translated the whole story, are in this part of it. ↑
“Fidèle à toute épreuve.” ↑
9
“Patraque,”—machine out of repair, and useless. ↑
10
[Contents]
FORS CLAVIGERA.
LETTER LVI.

I believe my readers will scarcely thank me for printing, this month,


instead of the continuation of the letter from Wakefield, a theological
essay by Mr. Lyttel. But it is my first business, in Fors, to be just,—
and only my second or third to be entertaining; so that any person
who conceives himself to have been misrepresented must always
have my types at his command. On the other side, I must point out,
before entering further into controversy of any kind, the constant
habit in my antagonists of misrepresenting me. For instance; in an
article forwarded to me from a local paper, urging what it can in
defence of the arrangements noticed by me as offensive, at Kirby
Lonsdale and Clapham, I find this sentence:

“The squire’s house does not escape, though one can see no reason
for the remark unless it be that Mr. Ruskin dislikes lords, squires, and
clergymen.”

Now I have good reason for supposing this article [218]to have been
written by a gentleman;—and even an amiable gentleman,—who,
feeling himself hurt, and not at all wishing to hurt anybody, very
naturally cries out: and thinks it monstrous in me to hurt him; or his
own pet lord, or squire. But he never thinks what wrong there may be
in printing his own momentary impression of the character of a man
who has been thirty years before the public, without taking the
smallest pains to ascertain whether his notion be true or false.

It happens, by Fors’ appointment, that the piece of my early life


which I have already written for this month’s letter, sufficiently
answers the imputation of my dislike to lords and squires. But I will
preface it, in order to illustrate my dislike of clergymen, by a later bit
of biography; which, at the rate of my present progress in giving
account of myself, I should otherwise, as nearly as I can calculate,
reach only about the year 1975.

Last summer, in Rome, I lodged at the Hotel de Russie; and, in the


archway of the courtyard of that mansion, waited usually, in the
mornings, a Capuchin friar, begging for his monastery.

Now, though I greatly object to any clergyman’s coming and taking


me by the throat, and saying ‘Pay me that thou owest,’ I never pass
a begging friar without giving him sixpence, or the equivalent
fivepence of foreign coin;—extending the charity even occasionally
as far as tenpence, if no fivepenny-bit chance to be in my purse. And
this particular begging friar having a [219]gentle face, and a long white
beard, and a beautiful cloak, like a blanket; and being altogether the
pleasantest sight, next to Sandro Botticelli’s Zipporah, I was like to
see in Rome in the course of the day, I always gave him the extra
fivepence for looking so nice; which generosity so worked on his
mind,—(the more usual English religious sentiment in Rome
expending itself rather in buying poetical pictures of monks than in
filling their bellies),—that, after some six or seven doles of
tenpences, he must needs take my hand one day, and try to kiss it.
Which being only just able to prevent, I took him round the neck and
kissed his lips instead: and this, it seems, was more to him than the
tenpences, for, next day, he brought me a little reliquary, with a
certificated fibre in it of St. Francis’ cloak, (the hair one, now
preserved at Assisi); and when afterwards I showed my friend Fra
Antonio, the Assisi sacristan, what I had got, it was a pleasure to see
him open his eyes, wider than Monsieur the Syndic at Hansli’s fifty
thousand crowns. He thought I must have come by it dishonestly; but
not I, a whit,—for I most carefully explained to the Capuchin, when
he brought it me, that I was more a Turk than a Catholic;—but he
said I might keep the reliquary, for all that.
Contenting myself, for the moment, with this illustration of my
present dislike of clergymen, I return to earlier days. [220]

But for the reader’s better understanding of such further progress of


my poor little life as I may trespass on his patience in describing, it is
now needful that I give some account of my father’s mercantile
position in London.

The firm of which he was head partner may be yet remembered by


some of the older city houses, as carrying on their business in a
small counting-house on the first floor of narrow premises, in as
narrow a thoroughfare of East London,—Billiter Street, the principal
traverse from Leadenhall Street into Fenchurch Street.

The names of the three partners were given in full on their brass
plate under the counting-house bell,—Ruskin, Telford, and Domecq.

Mr. Domecq’s name should have been the first, by rights, for my
father and Mr. Telford were only his agents. He was the sole
proprietor of the estate which was the main capital of the firm,—the
vineyard of Macharnudo, the most precious hillside, for growth of
white wine, in the Spanish peninsula. The quality of the Macharnudo
vintage essentially fixed the standard of Xeres ‘sack’ or ‘dry’—secco
—sherris, or sherry, from the days of Henry the Fifth to our own;—
the unalterable and unrivalled chalk-marl of it putting a strength into
the grape which age can only enrich and darken,—never impair.

Mr. Peter Domecq was, I believe, Spanish born; and partly French,
partly English bred: a man of strictest [221]honour, and kindly
disposition; how descended, I do not know; how he became
possessor of his vineyard, I do not know; what position he held,
when young, in the firm of Gordon, Murphy, and Company, I do not
know; but in their house he watched their head-clerk, my father,
during his nine years of duty, and when the house broke up, asked
him to be his own agent in England. My father saw that he could fully
trust Mr. Domecq’s honour, and feeling;—but not so fully either his
sense, or his industry: and insisted, though taking only his agent’s
commission, on being both nominally, and practically, the head-
partner of the firm.

Mr. Domecq lived chiefly in Paris; rarely visiting his Spanish estate,
but having perfect knowledge of the proper processes of its
cultivation, and authority over its labourers almost like a chief’s over
his clan. He kept the wines at the highest possible standard; and
allowed my father to manage all matters concerning their sale, as he
thought best. The second partner, Mr. Henry Telford, brought into the
business what capital was necessary for its London branch. The
premises in Billiter Street belonged to him; and he had a pleasant
country house at Widmore, near Bromley; a quite far-away Kentish
village in those days.

He was a perfect type of an English country gentleman of moderate


fortune;—unmarried, living with three unmarried sisters—who, in the
refinement of their highly [222]educated, unpretending, benevolent,
and felicitous lives, remain in my memory more like the figures in a
beautiful story than realities. Neither in story, nor in reality, have I
ever again heard of, or seen, anything like Mr. Henry Telford;—so
gentle, so humble, so affectionate, so clear in common sense, so
fond of horses,—and so entirely incapable of doing, thinking, or
saying, anything that had the slightest taint in it of the racecourse or
the stable.

Yet I believe he never missed any great race; passed the greater
part of his life on horseback; and hunted during the whole
Leicestershire season;—but never made a bet, never had a serious
fall, and never hurt a horse. Between him and my father there was
absolute confidence, and the utmost friendship that could exist
without community of pursuit. My father was greatly proud of Mr.
Telford’s standing among the country gentlemen; and Mr. Telford was
affectionately respectful to my father’s steady industry and infallible
commercial instinct. Mr. Telford’s actual part in the conduct of the
business was limited to attendance in the counting-house during two
months at Midsummer, when my father took his holiday, and
sometimes for a month at the beginning of the year, when he
travelled for orders. At these times Mr. Telford rode into London daily
from Widmore, signed what letters and bills needed signature, read
the papers, and rode home again: any matters needing deliberation
were referred to my father, [223]or awaited his return. All the family at
Widmore would have been limitlessly kind to my mother and me, if
they had been permitted any opportunity; but my mother always felt,
in cultivated society,—and was too proud to feel with patience,—the
defects of her own early education, and therefore (which was the
true and fatal sign of such defect) never familiarly visited any one
whom she did not feel to be, in some sort, her inferior.

Nevertheless, Mr. Telford had a singularly important influence in my


education. By, I believe, his sister’s advice, he gave me, as soon as
it was published, the illustrated edition of Rogers’ ‘Italy.’ This book
was the first means I had of looking carefully at Turner’s work: and I
might, not without some appearance of reason, attribute to the gift
the entire direction of my life’s energies. But it is the great error of
thoughtless biographers to attribute to the accident which introduces
some new phase of character, all the circumstances of character
which gave the accident importance. The essential point to be noted,
and accounted for, was that I could understand Turner’s work when I
saw it; not by what chance or in what year it was first seen.

Poor Mr. Telford, nevertheless, was always held by papa and


mamma primarily responsible for my Turner insanities.
In a more direct, though less intended way, his help to me was
important. For, before my father thought it right to hire a carriage for
the above mentioned [224]Midsummer holiday, Mr. Telford always lent
us his own travelling chariot.

Now the old English chariot is the most luxurious of travelling


carriages, for two persons, or even for two persons and so much of
third personage as I possessed at three years old. The one in
question was hung high, so that we could see well over stone dykes
and average hedges out of it; such elevation being attained by the
old-fashioned folding-steps, with a lovely padded cushion fitting into
the recess of the door,—steps which it was one of my chief travelling
delights to see the hostlers fold up and down; though my delight was
painfully alloyed by envious ambition to be allowed to do it myself:—
but I never was,—lest I should pinch my fingers.

The ‘dickey,’—(to think that I should never till this moment have
asked myself the derivation of that word, and now be unable to get at
it!)—being, typically, that commanding seat in her Majesty’s mail,
occupied by the Guard; and classical, even in modern literature, as
the scene of Mr. Bob Sawyer’s arrangements with Sam,—was
thrown far back in Mr. Telford’s chariot, so as to give perfectly
comfortable room for the legs (if one chose to travel outside on fine
days), and to afford beneath it spacious area to the boot, a
storehouse of rearward miscellaneous luggage. Over which—with all
the rest of forward and superficial luggage—my nurse Anne
presided, both as guard and [225]packer; unrivalled, she, in the
flatness and precision of her in-laying of dresses, as in turning of
pancakes; the fine precision, observe, meaning also the easy wit and
invention of her art; for, no more in packing a trunk than commanding
a campaign, is precision possible without foresight.
Posting, in those days, being universal, so that at the leading inns in
every country town, the cry “Horses out!” down the yard, as one
drove up, was answered, often instantly, always within five minutes,
by the merry trot through the archway of the booted and bright-
jacketed rider, with his caparisoned pair,—there was no driver’s seat
in front: and the four large, admirably fitting and sliding windows,
admitting no drop of rain when they were up, and never sticking as
they were let down, formed one large moving oriel, out of which one
saw the country round, to the full half of the horizon. My own
prospect was more extended still, for my seat was the little box
containing my clothes, strongly made, with a cushion on one end of
it; set upright in front (and well forward), between my father and
mother. I was thus not the least in their way, and my horizon of sight
the widest possible. When no object of particular interest presented
itself, I trotted, keeping time with the postboy—on my trunk cushion
for a saddle, and whipped my father’s legs for horses; at first
theoretically only, with dexterous motion of wrist; but ultimately in a
quite practical and [226]efficient manner, my father having presented
me with a silver-mounted postillion’s whip.

The Midsummer holiday, for better enjoyment of which Mr. Telford


provided us with these luxuries, began usually on the fifteenth of
May, or thereabouts;—my father’s birthday was the tenth; on that
day I was always allowed to gather the gooseberries for his first
gooseberry pie of the year, from the tree between the buttresses on
the north wall of the Herne Hill garden; so that we could not leave
before that festa. The holiday itself consisted in a tour for orders
through half the English counties; and a visit (if the counties lay
northward) to my aunt in Scotland.

The mode of journeying was as fixed as that of our home life. We


went from forty to fifty miles a day, starting always early enough in
the morning to arrive comfortably to four-o’clock dinner. Generally,

You might also like